Basic Pathology, Fifth Edition: An introduction to the mechanisms of disease 9781482264210, 1482264218

An accessible and enjoyable introduction to pathology and the mechanisms of disease, this book puts pathology into its h

1,249 308 144MB

English Pages 392 [591] Year 2016

Report DMCA / Copyright

DOWNLOAD FILE

Polecaj historie

Basic Pathology, Fifth Edition: An introduction to the mechanisms of disease
 9781482264210, 1482264218

Table of contents :
Cover
Half Title
Title Page
Copyright Page
Table of Contents
Preface
Acknowledgements
PART 1: Disease, health and medicine
Introduction: Disease, health and medicine
Chapter 1: What causes disease?
Chapter 2: What are the common mechanisms of disease?
PART 2:
Defence against disease
Introduction: The role of epidemiology in disease
Chapter 3: The body’s response to infection
Chapter 4: The acute inflammatory response
Chapter 5: Healing and repair, chronic and granulomatous inflammation
Chapter 6: Chronic inflammation and the adaptive immune response
Part 3: Features of cardiovascular disorders
Introduction: Features of cardiovascular disorders
Chapter 7: Vascular occlusion and thrombosis
Chapter 8: Atherosclerosis and hypertension
Chapter 9: Circulatory failure
Part 4: Cell growth and its disorders
Introduction: A brief history of cancer
Chapter 10: Benign growth disorders
Chapter 11: Malignant neoplasms
Chapter 12: What causes cancer?
Chapter 13: Molecular genetics of cancer
Chapter 14: The behaviour of tumours
Chapter 15: The clinical effects of tumours
Epilogue
Index
PART 1: Symptoms
Case 1: Intermittent chest pain
Case 2: Shortness of breath
Case 3: Difficulty in passing urine
Case 4: Difficulty in swallowing
Case 5: Easy bruising
Case 6: Coughing up blood
Case 7: Fever and cough
Case 8: Abdominal pain and weight loss
Case 9: A mixture of problems
Case 10: The ancient mimic
PART 2:
Signs
Case 11: Intermittent neurological signs
Case 12: Permanent neurological signs
Case 13: A cervical mass
Case 14: Haematuria clinic
Case 15: Reduced air entry
Case 16: Enlarged lymph nodes
Case 17: More enlarged lymph nodes
Case 18: Tachycardia and tachypnoea
Case 19: A social drinker
Case 20: Indigestion
Case 21: The diabetic clinic
PART 3:
Investigations
Case 22: Abnormal full blood count
Case 23: Complex blood results
Case 24: An abnormal smear test
Case 25: An abnormal mammogram
Case 26: An abnormal baby
Case 27: The genetic clinic
Case 28: Fracture clinic
Case 29: Acute back pain
Case 30: MRI diagnosis
Case 31: Investigation after death
Case 32: Sudden death of a child
Case 33: Jaundice
Case 34: A tutorial on glomerulonephritis
PART 4:
Treatment
Case 35: Novel chemotherapy
Case 36: Supporting haematopoiesis
Case 37: A prophylactic resection
Case 38: A chronic autoimmune disease
Case 39: Keeping the pump pumping
Case 40: Radiologist or surgeon?
Case 41: Organ transplantation
PART 5:
Complex management
Case 42: Immobility and its complications
Case 43: A life-threatening rash
Case 44: A first seizure
Case 45: Febrile and unconscious
Case 46: Multi-organ disease
Case 47: Mixed social and medical problems
Case 48: Certifying the cause of death
Case 49: Referring cases to the coroner
Case 50: Your worst nightmare!

Citation preview

Basic Pathology

Fifth edition

Basic Pathology An introduction to the mechanisms of disease

Sunil R Lakhani BSc MBBS MD FRCPath FRCPA Professor and Head, Molecular and Cellular Pathology, School of Medicine, University of Queensland and State Director, Anatomical Pathology, Pathology Queensland, Brisbane, Australia Susan A Dilly BSc MBBS FRCPath Emeritus Professor of Pathological Sciences, Barts and The London Medical School, Queen Mary University of London, UK Caroline J Finlayson MBBS FRCPath Formerly Honorary Senior Lecturer and Consultant in Histopathology, St George’s Hospital Medical School, London, UK Mitesh Gandhi MBBS MRCP FRCR FRANZCR Princess Alexandra Hospital and Queensland X-ray, Woolloongabba, Brisbane, Australia

CRC Press Taylor & Francis Group 6000 Broken Sound Parkway NW, Suite 300 Boca Raton, FL 33487-2742 © 2016 by Sunil R. Lakhani, Caroline Finlayson, Susan A. Dilly, and Mitesh Gandhi CRC Press is an imprint of Taylor & Francis Group, an Informa business No claim to original U.S. Government works Printed on acid-free paper Version Date: 20160201 International Standard Book Number-13: 978-1-4822-6419-7 (Pack - Book and Ebook) This book contains information obtained from authentic and highly regarded sources. While all reasonable efforts have been made to publish reliable data and information, neither the author[s] nor the publisher can accept any legal responsibility or liability for any errors or omissions that may be made. The publishers wish to make clear that any views or opinions expressed in this book by individual editors, authors or contributors are personal to them and do not necessarily reflect the views/opinions of the publishers. The information or guidance contained in this book is intended for use by medical, scientific or health-care professionals and is provided strictly as a supplement to the medical or other professional’s own judgement, their knowledge of the patient’s medical history, relevant manufacturer’s instructions and the appropriate best practice guidelines. Because of the rapid advances in medical science, any information or advice on dosages, procedures or diagnoses should be independently verified. The reader is strongly urged to consult the relevant national drug formulary and the drug companies’ and device or material manufacturers’ printed instructions, and their websites, before administering or utilizing any of the drugs, devices or materials mentioned in this book. This book does not indicate whether a particular treatment is appropriate or suitable for a particular individual. Ultimately it is the sole responsibility of the medical professional to make his or her own professional judgements, so as to advise and treat patients appropriately. The authors and publishers have also attempted to trace the copyright holders of all material reproduced in this publication and apologize to copyright holders if permission to publish in this form has not been obtained. If any copyright material has not been acknowledged please write and let us know so we may rectify in any future reprint. Except as permitted under U.S. Copyright Law, no part of this book may be reprinted, reproduced, transmitted, or utilized in any form by any electronic, mechanical, or other means, now known or hereafter invented, including photocopying, microfilming, and recording, or in any information storage or retrieval system, without written permission from the publishers. For permission to photocopy or use material electronically from this work, please access www.copyright.com (http://www.copyright. com/) or contact the Copyright Clearance Center, Inc. (CCC), 222 Rosewood Drive, Danvers, MA 01923, 978-750-8400. CCC is a not-forprofit organization that provides licenses and registration for a variety of users. For organizations that have been granted a photocopy license by the CCC, a separate system of payment has been arranged. Trademark Notice: Product or corporate names may be trademarks or registered trademarks, and are used only for identification and explanation without intent to infringe. Visit the Taylor & Francis Web site at http://www.taylorandfrancis.com and the CRC Press Web site at http://www.crcpress.com

CONTENTS Preface Acknowledgements PART 1 Disease, health and medicine Introduction: Disease, health and medicine Chapter 1: What causes disease? Chapter 2: What are the common mechanisms of disease?

1 3 7 40

PART 2 Defence against disease Introduction: The role of epidemiology in disease Chapter 3: The body’s response to infection Chapter 4: The acute inflammatory response Chapter 5: Healing and repair, chronic and granulomatous inflammation Chapter 6: Chronic inflammation and the adaptive immune response

87 89 95 109

Part 3

Features of cardiovascular disorders Introduction: Features of cardiovascular disorders Chapter 7: Vascular occlusion and thrombosis Chapter 8: Atherosclerosis and hypertension Chapter 9: Circulatory failure

209 211 215 235 258

Part 4

Cell growth and its disorders Introduction: A brief history of cancer Chapter 10: Benign growth disorders Chapter 11: Malignant neoplasms Chapter 12: What causes cancer? Chapter 13: Molecular genetics of cancer Chapter 14: The behaviour of tumours Chapter 15: The clinical effects of tumours

291 293 295 305 320 332 354 361 373 374

Epilogue Index

147 178

PREFACE ‘What is the use of a book’, thought Alice, ‘without pictures or conversations.’ Lewis Carroll

Any artist will tell you that in drawing objects, you cannot ignore the spaces in between: the picture ceases to exist when only one aspect is viewed in isolation. Musical pieces composed entirely of notes and without pauses would be nothing more than a noise and an irritation. Yet when it comes to teaching, we may ignore this fact and fail to put our own specialty into the context of the whole curriculum. Over the last decade, there has been a trend towards a more integrated approach to medical education. We are delighted that such an approach, which we have always used in our teaching, is now widely adopted throughout the world. Our aim in this book has been to create a tutorial on the mechanisms of disease over a background of history, science and clinical relevance. The goal is to give the student a sense of belonging to a movement, the movement from past to present and from cell to patient. This book has been written in the hope that the student will read the text fully and at leisure. This not only contains detail about the disease processes but also historical anecdotes and clinical scenarios. The cartoons are intended to amuse as well as illustrate the importance of certain topics, and we sincerely hope that students reading the book will be able to shed the dull, dreary image of pathology that they all seem to be born with. Pathology is one of the most fascinating and fun subjects students are likely to encounter during their undergraduate training. If you understand the basic principles of disease, then the interpretation of clinical symptoms and signs, the rationale behind investigation and treatment, and the unravelling of complex cases

become more logical. Time spent building a framework of mechanisms will assist your clinical practice. This fifth edition of Basic Pathology can be used in conjunction with a companion volume called Pathology in Clinical Practice: 50 Case Studies, which is designed to help you use your pathology knowledge in clinical settings. Some extracts from Pathology in Clinical Practice are included in this book as well as links to the full cases highlighted with a ‘link’ symbol. Read more about bacterial infection in Pathology in Clinical Practice Case 7

When you access the e-book of Basic Pathology, either via the unique code printed in your hard copy or if you have purchased the e-book only, you will have access to the full text of both titles, and hyperlinks from Basic Pathology will take you directly to relevant cases in Pathology in Clinical Practice. We hope that you will find these stimulating, fun and complementary to this book because they contain more advanced, clinically relevant information. This edition introduces radiological images in place of some of the traditional autopsy-based photographs of diseased organs. We recognise how significantly technology has changed everyday clinical practice and students are expected to be familiar with the increasing repertoire of scanned images. Over the next decades, the same may become true for genomic information, and we have extensively revised and expanded these sections. This book is primarily intended for medical students, but should also be useful to students of dentistry, human biology and other health professions. Postgraduate students studying for pathology or surgical exams may also wish to consult these books.

ACKNOWLEDGEMENTS When we set out more than 20 years ago to write a textbook in which we would take the students on a journey, from past to present and from patient to the cell and back again, the phrase ‘integrated curriculum’ was not in common usage. Today, most medical schools have switched to the new-style curriculum in which pathology is taught as an integrated subject with other disciplines. With each version of this book, there has been a move towards greater integration, especially with our colleagues in radiology who share our key role in diagnosing and monitoring disease in patients. We have also been influenced considerably by the many clinicians who attend joint clinico–pathological meetings to discuss individual patients and the best approach for their clinical management. All these should be acknowledged as informing and shaping this book. In addition, our endeavours to produce this fifth edition have been considerably aided by help, comments and constructive criticism from a number of people, and we would like to acknowledge their time and effort. In particular, thanks to Peter Riley, Alison Milne and Alex Roche for their useful comments. This book

has evolved through each edition and all contributors to earlier editions have influenced this latest version. They include Professor Philip Butcher, Professor Mike Davies and Dr Grant Robinson at St George’s Hospital; Dr Ahmet Dogan, one of our co-authors from the third edition, and Dr Barry Newell, the lead author on the companion volume of Pathology in Clinical Practice. Dr Peter Simpson and Dr Amy McCart Reed from Brisbane made a useful contribution to the molecular sections of the current edition. We would like to thank the staff of Queensland X-ray and the radiology department of the Princess Alexandra Hospital, Brisbane for their help in providing the radiology images which have been used. Finally, and most importantly, we would like to thank our families for their encouragement and support. But for their understanding, it would be difficult to get away with the disproportionate amount of time that such projects consume. SRL, SAD, CJF, MG 2016

PArT 1 DISEASE, HEALTH AND MEDICINE

Introduction: Disease, health and medicine

3

Chapter 1: What causes disease?

7

Chapter 2: What are the common mechanisms of disease?

40

1

INTroDuCTIoN Medicine, to produce health, has to examine disease. Plutarch (c. 46–120) Greek biographer and essayist

Medicine is the science and practice of diagnosing, treating and preventing diseases and it depends on understanding the mechanisms of disease – the topic of this book. Diseases have causes (aetiology) and mechanisms (pathogenesis). They may produce symptoms (experienced by the patient) and signs (elicited by the physician). There may be structural changes that are visible to the naked eye (gross or macroscopical appearances), or only detectable down a microscope (microscopical appearances). Functional changes may be apparent to the patient or only detected by clinical or laboratory tests. All of this is pathology, and pathology is the study (logos) of suffering (pathos). Our approach to investigating mechanisms has, however, changed significantly over the last 25 years from a simple linear cause-and-effect approach (reductionism) to an appreciation of the significance of complex interactions (holistic approach). Huge advances in computer technology have allowed modelling of complex systems that would not previously have been possible and this is opening up whole fields of new research (the ‘omics’). The problem for a doctor or student is that this is mind-boggling and, in day-to-day life, we rely on simpler concepts that have practical application to the patient in front of us, so we acknowledge the complexity but adopt the simplistic. Generally in this book we use a mechanistic model that regards the body as a machine with repairable or replaceable parts. It places a high emphasis on the scientific evidence base for untangling cause and effect in both the disease and its treatment, because this is important for patient care and prognosis. We also describe ‘associations’ that have been identified between various

factors and diseases but where no clear mechanism has been discovered to explain the link. These are areas of intense research and worth reading about in research journals to learn the latest ideas. So how would you define ‘disease’? In fact, this is fraught with problems because disease definitions change with time and across cultures. Do you, in fact, need to suffer from a disease as part of the definition? Does a person have the disease before the symptoms appear? This is particularly important now that there is screening for many diseases and people may have abnormal ‘disease markers’ without any symptoms, or the ‘disease marker’ occurs in people who will never have the disease. This can lead to over-treating people who have lesions considered as premalignant but who would never develop invasive cancer, or people who have raised blood pressure but might not have a stroke or heart attack. With those caveats, we offer a biomedical definition of disease – ‘Disease occurs when homeostasis fails’. Homeostasis is the concept of a regulated system that maintains equilibrium (a balanced state) within the body, despite changes in the internal or external environment. If you accept this definition then understanding the mechanisms of disease will involve understanding the processes for maintaining homeostasis, identifying the agents and events that disrupt homeostasis, trying to determine why homeostatic mechanisms fail, and testing treatments that can alter the sequence of events and restore health. A disease should have the potential to produce some impairment of function, but may be detected while asymptomatic. It may also be treated or heal through the body’s normal processes so that no permanent damage is produced. Let us take the example of lobar pneumonia (see Chapter 4, fig 4.25) and the key concept of homeostasis and introduce a diagram that helps to illustrate the various components of the disease process (Fig. 1).

Introduction: Disease, health and medicine

DISEASE, HEALTH AND MEDICINE

3

Introduction: Disease, health and medicine

4

Disease agent

Cell Tissue Organ Person

Resolution Bacteria Adaptive response involving internal factors

Structural changes Functional changes

Treatment

Symptoms Signs (a)

Acute inflammatory response Consolidation (structural change) Reduced gas transfer in alveoli (functional change)

Resolution

Symptoms: Cough, breathlessness, haemoptysis Signs: Reduced chest movements, dull to percussion, X-ray changes

Improved gas transfer with new equilibrium achieved

Part 1: Disease, health and medicine

Oxygen therapy

Inflammatory cells produce Reduced gas transfer in alveoli (functional change)

Pyrogens (internal factors)

Response = tachycardia

Hypoxia (internal factor)

(b)

Figure 1 Disease occurs when homeostasis fails: (a) homeostatic mechanisms restore equilibrium; (b) some homeostatic mechanisms in lobar pneumonia (green dotted arrow = homeostatic mechanism).

Introduction: Disease, health and medicine

Risk factor A risk factor confers an increased risk of developing a disease, e.g. smoking and lung cancer.

Pathogenesis The pathological mechanism which results in clinically evident disease, e.g. the way in which the interaction between M. tuberculosis and the host immune system produces the caseating granulomatous lesion of TB.

Aetiology The aetiology is the cause of a disease, e.g. Mycobacterium tuberculosis casuses tuberculosis (TB).

5

Predisposition Patients have an increased susceptibility to develop a disease - usually inherited e.g. Familial Adenomatous Polyposis patients have a mutated APC gene and risk developing colorectal carcinoma if a succession of mutations occurs in one or more of their polyps. Premalignant Premalignant is a term for a pathological lesion or process which will probably, if untreated, transform to invasive malignancy, e.g. high-grade dysplasia in the cervix. (The aetiological agent is Human Papilloma Virus.) Disease mechanism This is the way in which disease-causing agents disturb homeostatic mechanisms. Understanding how a disease evolves can help in prevention or treatment (e.g. vaccination against HPV virus to prevent cervical carcinoma).

Figure 2 Don’t be confused by terminology!

In its simplest form, an intrinsic or external factor (the cause) acts on a cell, tissue, organ or whole person to produce structural or functional changes and a response. If an adaptive response is 100% successful, then homeostasis is maintained and no symptoms or signs result. If unsuccessful, then the disease manifests and the structural and functional changes may have an impact on another cell, tissue or organ to produce another set of reactions. Thus, in lobar pneumonia, the extrinsic cause is the bacterium, Streptococcus pneumoniae, which affects the lung. This stimulates an acute inflammatory response that can produce the structural changes of consolidation and functional changes of reduced gas transfer in alveoli. The patient has symptoms of a cough, breathlessness and, even, haemoptysis (coughing up blood).

The physician may detect the signs of reduced chest movements, an area that is dull to percussion and radiological opacity reflecting lung solidification. However, this is not the end of the story. We can produce another ‘disease sequence’ where the reduced gas transfer results in hypoxia (reduced oxygen saturation in the blood), which acts as an internal factor that leads to the heart responding with an increased heart rate (tachycardia). More blood is pumped through the part of the lung, which is not consolidated, so increasing gas transfer, and this may successfully compensate so that the hypoxia is corrected, i.e. a new equilibrium is achieved. But before you relax, look at page 139 where we explain about pyrogens that may accompany the inflammatory response to bacteria and the effect that they may have

Introduction: Disease, health and medicine

Disease Disease is a consequence of failure of homeostasis. It should have the potential to produce impairment of function, even though it may be diagnosed while still symptomatic (e.g. breast cancer discover in a screening mammogram).

6

Introduction: Disease, health and medicine

Part 1: Disease, health and medicine

on the heart. Yes, that is another route for producing a tachycardia and so our diagram could become almost infinitely complex as we add the different pathways. By now, you will have realised why we started by comparing reductionist and holistic approaches for understanding mechanisms. In the simplest situation, we just identify the primary mechanism, treat the patient with antibiotics against S. pneumoniae, remove the primary cause and return the patient to health. If the patient is very unwell and the secondary mechanisms are operating, we may need to provide oxygen by facemask to reduce the hypoxia, i.e. treat the secondary factor. We could consider using drugs such as paracetamol that have an antipyretic action and will help counteract the effect of the pyrogens; and so it goes on! Where should we stop? Are we able to analyse intelligently all the processes that may be operating? The answer is no but we are making huge leaps forward. We mentioned the ‘omics’, which is the term used for studying the totality of activities. Genomics is about analysing the functions and structure of the genome (i.e. the interactions of all the genes). Proteomics studies the interactions of all the proteins in a cell, lipidomics all the lipid-based actions. If you start off healthy and successfully maintain homeostasis, surely you should remain healthy. But

do you start off healthy? That leads us to the concept of predisposition. We each have around 25 000 genes present in every cell but expressed differently, depending on the cell type and its response to its environment. The likelihood of our suffering from a disease depends on interactions between our genes and the environment, so called G×E interactions. The environment, however, starts working from the moment of conception so even genetically identical twins may have different predispositions for diseases by the time they are born. This continues on through life, with our personal ‘exposome’ being the life-long impact of the various environmental exposures on our genome and our health. However, things are rarely inevitable and so you may be able to identify your higher predisposition for disease, understand the risk factors that will make matters worse, avoid those risk factors and so not get the disease! Preventive medicine in the future may be much more tailored to individual circumstances and susceptibility than was previously possible. Drug treatments may be used only where they will be truly beneficial. This could be the beginning of an era of personalised medicine based on individual genome analysis so understanding the mechanisms of disease is fundamental – read on!

CHAPTEr 1

WHAT CAuSES DISEASE?

8 8 10 13 15 16

Those who are enamoured of practice without science are like a pilot who goes into a ship without rudder or compass and never has any certainty where he is going. Practice should always be based upon a sound knowledge of theory. Leonardo da Vinci (1452–1519) Italian artist, sculptor, architect and engineer

Every day we eat food, breathe air, walk, talk and perform the tasks of daily life before going to bed unscathed. Some days, however, we may encounter and be harmed by viruses, bacteria, unusual antigens, extremes of temperature, chemical pollutants or fast-moving vehicles. We are harmed by environmental (or extrinsic) factors. The cause is usually obvious if it happens immediately but may be hard to discern when the effect takes days or years to manifest, e.g. tobacco smoking causing lung cancer. In some situations, the environment plays no part and the cause is entirely genetic (intrinsic), e.g. the anaemias related to abnormal haemoglobin (see page 279). There is no particular way of classifying the causes of diseases but it is useful to have a checklist that you find helpful when talking with patients and thinking about their ‘differential diagnosis’, i.e. the possible conditions and the causes that would explain their current symptoms. Table 1.1 shows such a list of causes, which are presented in an order that moves from the relatively simple to the more complex, so that you could

Biological agents as causes of diseases Inflammatory conditions Case study: coeliac disease Nutritional diseases Genetic causes of disease Case study: an abnormal baby

16 22 24 26 27 38

Table 1.1 Categorising the causes of disease Category

Example of causes and diseases

Physical

Heat, cold, radiation, electrical, mechanical

Chemical

Tobacco smoke, alcohol, drugs, poisons, air pollution

Structural

Congenital neural tube defects, vascular occlusion, bowel perforation

Infectious

Bacteria, viruses, fungi, protozoa, prions

Inflammatory

Peanut anaphylaxis, dust-mite asthma, autoimmune conditions

Nutritional

obesity, malnutrition, diabetes

Degenerative

osteoarthritis, dementia

Genetic

identify and treat the straightforward cases quickly. Depending on your location and specialty, you could use this approach to construct a flowchart of the most likely causes for your patients. We have put genetic causes at the bottom of the list because the genes interact with most of the other causes in complex ways. It is possible to produce a GxE (genes x environment) card to illustrate this, as shown on

Chapter 1: What causes disease?

Physical causes Clinical scenario: frostbite Chemical causes Case study: coughing up blood Structural causes Which are the most important diseases today?

7

8

What causes disease?

page 215. Mostly these causes are based on statistically derived associations rather than precise knowledge of the mechanisms but we attempt to highlight the main mechanisms through which genes affect health.

PHYSICAL CAuSES Let us start with a physical cause of disease and look at extremes of temperature.

Part 1: Disease, health and medicine

CLINICAL SCENArIo: FroSTBITE

A 29-year-old skier lost his way after drinking a few beers in a mountain restaurant. He was discovered, unconscious, without a hat, 20 hours later in an exposed location on the mountain face. He had obvious frostbite to his nose and fingertips, manifest as hard white areas. A weak, thready, but regular pulse was present and he was breathing shallowly. He was wrapped in a space blanket and rushed by helicopter to hospital. His rectal (core) temperature was found to be 30°C. He was warmed with blankets and hot water bottles. He suffered a cardiac arrest but was immediately resuscitated. His mild metabolic acidosis was corrected. As he regained consciousness he began to shiver violently and his limbs became reddened and swollen. He tried to raise himself up but promptly fainted. Over the next couple of days his fingertips and nose turned black and the skin began to slough off. At one point it appeared that he would lose the tips of most of his fingers, but after the skin had fallen off only two fingertips were lost and his nose was saved (Fig 1.1). What has happened here?

Our patient was hypothermic, i.e. his core temperature was 12°C • Numbness: membrane sodium pump inactivated, nerves and muscles inexcitable • Capillary endothelium damaged by cold 36.5 Data not available

Part 1: Disease, health and medicine

Figure 1.7 Percentage of tobacco use among adults. (From WHO global observatory map. © WHO 2008. All rights reserved.)

Figure 1.8 Axial CT scan showing calcified pleural plaques (blue arrows) and a fibrotic process with increased interstitial thickening at the bases (red arrow). The latter is also known as asbestosis and is associated with progressive respiratory dysfunction.

Figure 1.9 Amiodarone and the liver: an iatrogenic chemical cause of liver disease. Amiodarone is a drug used in the treatment of cardiac arrhythmias. It can potentially have side effects on the liver, ranging from abnormal liver function tests to, rarely, cirrhosis. Over the long term amiodarone is deposited in the liver, resulting in a change in the density of the liver on a CT scan. The normal liver should be the same density as the spleen (red arrow). However, this patient’s liver has a higher density (blue arrows) in keeping with amiodarone accumulation.

Structural causes

STruCTurAL CAuSES Structural causes include conditions that are congenital and produce obvious physical disability (Fig. 1.10), such as neural tube defect, or are acquired, such as acute obstruction or rupture of blood vessels or parts of the bowel. The cardiovascular structural conditions are covered in Chapters 7–8 which describe thrombosis, embolism and atherosclerosis. The congenital conditions are often of unknown cause but some have clear genetic or infective causes.

15

Genes are extremely important during normal development and mistakes that occur during the production of gametes, fertilisation of the ovum and formation of the embryo can result in malformation or death. A key concept is ‘body patterning’, which provides a basic anatomical organisation through creating gradients of gene products. For example, the four HOX gene clusters create a gradient along the anteroposterior axis of the embryo. Not all of our organs are paired and symmetrical; as an example we only have one heart and this should be situated on the left. To achieve this requires a right–left differentiation which is dependent on functioning cilia creating an asymmetrical flow of fluid (Fig. 1.11). Figure 1.10 (a) Postmortem CT and (b) MrI on a fetus born with a large occipital encephalocele – a defect in the posterior skull bones with herniation of the brain and meninges through the defect (yellow arrows). This is one of the types of neural tube defect. others include spina bifida and anencephaly.

b

Figure 1.11 Kartagener’s syndrome is due to abnormal cilia that affect embryonic development – chest radiograph showing the situs inversus or ‘mirror imaging’ component of the syndrome, with the apex of the heart pointing to the right (blue arrow), a right-sided aortic arch (red arrow) and a gastric air bubble (yellow arrow) on the right side.

Chapter 1: What causes disease?

a

A systems-biology approach to understanding the ciliopathy diso...

https://genomemedicine.biomedcentral.com/articles/10.1186/gm275

Cilia maintain separate cytoplasmic and membrane compartments, but are completely lacking vesicles. Instead cilia rely on specialized modes of transportation called intraflagellar transport (IFT) to deliver cargo proteins and lipids along the axoneme. IFT can operate in the anterograde direction (towards the ciliary tip) using complex B factors, or in the retrograde direction (towards the ciliary base) using complex A factors; together these factors regulate the transport speed and net cargo flux. An important, recently emerged aspect of cilia is the gatekeeper role of the septin family of proteins, regulating initial entry and exit of ciliary factors at the base of the cilium [1]. The transition zone, where the gatekeeper functions, is an adjacent structure at the base of the cilium, forming the linkers between microtubule and ciliary membrane; it is probably required for unloading ciliary-directed cargo and sorting ciliary-based signaling mechanisms [2] so that the cell can interpret their context.

Emerging genetics of the ciliopathies

16

What causes disease?

Although ciliopathies are individually rare disorders, an amazing spectrum of what were previously disparate syndromes is now recognized as part of the ciliopathy spectrum. Ciliopathies can be subdivided into 'motile ciliopathies' and 'non-motile ciliopathies', although we usually define ciliopathies as disorders that result from aplasia and/or disrupted function of primary cilia. Motile ciliopathies comprise a class of disorders displaying prominent situs inversus (a condition in which the normal positions of organs are reversed). Non-motile ciliopathies show prominent but mixed features in several vital organs, including the brain, kidney and liver, and others, such as the eye and digit. Ciliopathies range from largely organ-specific disorders, such as polycystic kidney disease (PKD), to pleiotropic disorders, such as cerebello-oculo-renal syndrome (CORS), Bardet-Biedl syndrome (BBS), Jeune asphyxiating thoracic dystrophy (JATD) and MeckelGruber syndrome (MKS) (Figure 2). All ciliopathies known so far show a recessive mode of inheritance, either autosomal or X-linked, with strong evidence of genetic modifiers that determine expressivity. Moreover, clinically distinguishable ciliopathies often result from mutations in a single gene (Table 1), suggesting complex genetic networks.

Figure 2 Almost every organ in the body shows vulnerability in the ciliopathies. Most ciliopathies have overlapping clinical features in multiple organs. Cystic kidney and retinal defects are frequently observed. Skeletal dysplasia is predominantly seen in JATD, OFD1 and EVC. ALMS, Alström syndrome; BBS, Bardet-Biedl syndrome; CORS, cerebello-oculo-renal syndrome; EVC, Ellis-van Creveld syndrome; JATD, Jeune asphyxiating thoracic dystrophy; JBTS, Joubert syndrome; LCA, Leber congenital amaurosis; MKS, Meckel syndrome; NPHP, nephronophthisis; OFD1, oral-facial-digital syndrome type 1; PCD, primary ciliary dyskinesia; PKD, polycystic kidney disease.

Just to give you an idea of the complexity, Fig. 1.12 is a picture of some congenital abnormalities associated with ciliopathies and the genes involved. Please don’t attempt to memorise these; just be grateful that you can look them up on genetic databases when required.

Table 1

Ciliopathies, genes and subcellular functions of the proteins

Part 1: Disease, health and medicine

3 of 10

WHICH ArE THE MoST IMPorTANT DISEASES ToDAY? The World Health Organization (WHO) 2012 mortality statistics show that, of the 56 million people who died in 2012, ‘non-communicable diseases’ caused 68% of all deaths: cardiovascular diseases (CVDs – mainly ischaemic heart disease and strokes), cancers, diabetes and chronic lung diseases were the most important of these. ‘Communicable’, i.e. ‘infectious’, maternal, neonatal and nutritional conditions together caused 23% of global deaths, and injuries caused 9%. The use of tobacco is thought to be at the root of 10% of all deaths, largely from

Figure 1.12 Almost every organ in the body shows vulnerability in the ciliopathies. Most ciliopathies have overlapping clinical features in multiple organs. Cystic kidney and retinal defects are frequently observed. Skeletal dysplasia is predominantly seen in JATD, oFD1 and EVC. ALMS, Alström’s syndrome; BBS, Bardet–Biedl syndrome; CorS, cerebro-oculo-renal syndrome; EVC, Ellis–van Creveld syndrome; JATD, Jeune asphyxiating thoracic dystrophy; JBTS, Joubert’s syndrome; LCA, Leber’s congenital amaurosis; MKS, Meckel’s syndrome; NPHP, nephronophthisis; oFD1, oral– facial–digital syndrome type 1; PCD, primary ciliary dyskinesia; PKD, polycystic kidney disease. (reproduced from Lee and Gleeson. Genomic Medicine 2011;3:59.)

respiratory tract (and other) cancers and cardiovascular disease. What makes troubling reading is the fact that, in 22/04/2016, 10:18 low- and middle-wealth countries, the average age at which deaths occurred is considerably lower than in the wealthy countries. Of deaths in affluent countries 87% are from CVDs and cancers, compared with 37% in low-income countries. Deaths due to infective causes and poor hygiene are far more common in low- and mid-income countries, of which tuberculosis, diarrhoeal diseases, malaria and HIV/AIDS are major contributors. Tuberculosis (TB) is second only to HIV/ AIDS as the greatest killer worldwide due to a single infectious agent and TB causes a quarter of all deaths in HIV/AIDS patients. BIoLoGICAL AGENTS AS CAuSES oF DISEASES Infectious causes are especially important because of the number of people affected worldwide and the fact that many are treatable with antimicrobials.

Biological agents as causes of diseases

changes in the classification of some medically important microbes.

read more about bacterial infection in Pathology in Clinical Practice Case 7

Bacteria are generally classified according to their shape and whether they stain blue–purple with the Gram stain (Fig. 1.13 and Table 1.3). Endotoxins and exotoxins are important mechanisms by which they cause damage and these are described in detail in Chapter 2 page 72. Advances in molecular biological techniques have led to some significant changes in medical microbiology. Most importantly there have been DNA in nucleus

17

TAXoNoMY

Microbes were traditionally classified on the basis of their microscopical morphology, culture requirements, biochemical and serological properties – in other words their phenotypic characteristics. Nucleic acid amplification and genome sequencing mean that organisms can now be speciated by their genotypic properties. In bacteriology this has resulted in the reclassification of many

Membrane protein

The peptidoglycan wall contains techoic acid and can stimulate an acute inflammatory response Organelles Gram-positive bacteria stain blue with crystal violet, which lodges in their thick outer wall. Inside they have a double-layered phospholipid membrane, with membrane proteins, surrounding cytoplasm with organelles and a circular nucleus composed of double-stranded DNA Outer coat contains lipopolysaccharide (LPS), a component of which causes endotoxic shock, fever and diarrhoea. Porin protein allows nutrient transfer

LPS

Periplasmic space contains enzymes and proteins in a gel

LPS

Thin peptidoglycan coat

Gram-negative bacteria have an extra, double-layered coat that traps crystal violet before it can reach the peptidoglycan coat (which is thinner than in G+ bacteria).The violet crystals are washed out by alcohol as part of the Gram staining process and the bacterium is visualized using a red counter-stain

Cocci

Staphylococci

Other Corkscrew

Diplococci Coccus

Vibrio Streptococci

Filamentous

Bacilli Coccobacillus

Bacillus

Helical form

Spirochaete

Bacterial shapes – bacteria are classified by their shape and groupings as well as their Gram staining. Bacteria can be circular, rod shaped, spiral, curved and/or have flagellae. They can occur singly, in pairs (diplo), clusters (staphlo) and strings (strepto); eg streptococci.

Figure 1.13 Gram-negative and Gram-positive bacteria.

Chapter 1: What causes disease?

Bacterial shapes

What causes disease?

18

Table 1.3 Simple summary of the ways to classify bacteria Gram-positive

Gram-negative

Acid fast

Mycobacteria

Examples of aerobic and anaerobic bacteria obligate aerobes

Bacillus cereus

Neisseria spp.

Facultative anaerobes

Bacillus anthracis

Escherichia coli

Staphylococci

Salmonellae

Microaerophilic bacteria Streptococci

Spirochaetes

obligate anaerobes

Bacteroides spp.

Clostridia

Examples of morphologically distinct Gram-positive and Gram-negative bacteria Cocci (spherical)

Streptococci (in chains).

Neisseria spp.

Staphylococci (in clusters) Bacilli (rod shaped)

Corynebacteria

Haemophilus spp.

Clostridia

Bordetella spp.

Bacilli

Klebsiella, Proteus and Shigella spp., E. coli

Listeria spp.

Vibrio, Salmonella spp.

Spiral

Treponema, Borrelia, Leptospira (spirochaetes)spp. Helicobacter, Campylobacter spp.

Pleomorphic

Part 1: Disease, health and medicine

Branching

Rickettsia spp. (intracellular obligates) Actinomyces, Nocardia spp.

medically important organisms and, in some cases, this confusingly means that the name of the organism has changed, often with the generation of a new genus to comply with taxonomic rules. A good example is in the genus Pseudomonas where the organism Pseudomonas maltophilia (a cause of bacteraemia in immunocompromised patients) became Xanthomas maltophilia, and is currently known as Stenotrophomonas maltophilia and where the organism Pseudomonas cepacia (a cause of respiratory tract infection in patients with cystic fibrosis) is now Burkholderia cepacia. Some organisms have now been found to be made up of a complex of several different species. The genus Streptococcus is a good example of this: Streptococcus bovis has now been divided into two species – Streptococcus gallolyticus and Streptococcus pasteurianus. Is this important? In some cases it has little practical relevance, but in the case of the former Streptococcus bovis it does. It was known in the past that there was an association with colonic carcinoma in patients who

had a bacteraemia with this organism. It has now been shown that this association is really with Streptococcus gallolyticus not with Streptococcus pasteurianus. More profoundly, some microbes have been found to have been completely misclassified. Pneumocystis carinii, which can cause respiratory infection in immunocompromised hosts, is no longer classified as a protozoan and is now known to be a fungus and has been renamed Pneumocystis jirovecii. Molecular techniques such as 16-S ribosomal DNA polymerase chain reaction (PCR) have allowed previously unculturable organisms to be identified, e.g. the bacterium Tropheryma whipplei, the cause of Whipple’s disease. Although most infectious disease is due to bacteria and viruses, there are other categories that are just as important, and that have evolved their own mechanisms for bypassing host defences and causing disease. Briefly, these are the fungi, protozoa, parasites, helminths and prion proteins.

Biological agents as causes of diseases

19

Figure 1.14 A radiograph in a young patient of 32 with AIDS showing bilateral perihilar infiltrates due to a pneumocystis infection (pneumocystis pneumonia). This fungus causes infection in immunosuppressed patients.

Nucleic acid

Symmetry

Envelope

Strand

Family

Example

rNA

Icosahedral

No

SS1

Picorna

Polio, Coxsackie

DS

reo

rotavirus

SS

Toga

rubella (rubivirus)

Flavi

Dengue fever

SS1

Corona

Colds

SS2

orthomyxo

Influenza A, B and C

Paramyxo

Mumps, measles

rhabdo

rabies

Yes

Helical

DNA

Yes

Complex

Complex

SS1

retro

HIV, HTLV

Icosahedral

No

SS linear

Parvo

red cell destruction

DS circular

Papova

Papillomavirus

DS linear

Adeno

Colds

DS linear

Herpes

Herpes simplex

Yes

Varicella-zoster Cytomegalovirus Epstein–Barr virus

Complex SS, single strand; DS, double strand.

Complex

DS circular

Hepadna

Hepatitis B

DS linear

Pox

Smallpox

Chapter 1: What causes disease?

Table 1.4 Outline classification of viruses (see Figs 2.30 and 2.31)

20

What causes disease?

ProToZoA

Protozoa are free-living, single-celled eukaryotes with nuclei, endoplasmic reticulum, mitochondria and organelles. They ingest nutrients through a cytosome and can reproduce sexually and asexually. Most are able to form cysts when in hostile environments (Table 1.5).

water, penetrate the skin or be transmitted by insects (Table 1.6). FuNGI

Helminths or worms can usually be seen by the naked eye. They can enter from contaminated food or

Fungi are eukaryotic cells requiring an aerobic environment. They are more likely to cause significant diseases if a patient is immunocompromised (so-called opportunistic infection) or the normal flora of the mouth, gut or vagina are altered by antibiotics. They are generally classified as superficial, cutaneous, subcutaneous and systemic (Table 1.7).

Figure 1.15 Leishmania sp., a parasitic flagellate protozoan (arrows).

Figure 1.16 ovum from a schistosome, one of the trematodes.

HELMINTHS

Table 1.5 Examples of protozoa that cause human disease Organism

Disease

Part 1: Disease, health and medicine

a,b

Toxoplasma gondii

Cerebral, ocular and lymphoid damage

Plasmodium (falciparum, vivax, ovale, malariae, knowlesi)

Malaria

Leishmania spp. (various)

Cutaneous and visceral leishmaniasis (Fig. 1.15)

Trypanosoma spp. (various)

Sleeping sickness and Chagas’ disease

Entamoeba histolytica

Diarrhoea

Giardia lamblia

Diarrhoea a

Cryptosporidia

Diarrhoea

a

Isospora spp.

Acanthamoeba spp.

Diarrhoea a

Keratitis

Naegleri fowleri Trichomonas spp. a

Vaginal discharge

People with defective immune systems are more liable to have significant problems with these organisms. Problems often relate to reactivation because the immune defences are reduced rather than there being a primary infection.

b

Biological agents as causes of diseases

Helminth

Disease

Trematodes (flukes) Schistosoma spp. (various) Schistosomiasis (liver, (skin penetration) lung, gut and bladder damage) (Fig. 1.16) Cestodes (tapeworms) Echinococcus

Hydatid disease

Taenia solium, Taenia saginata and Diphyllobothrium latum

Tapeworm infestation from pig, cow and fish

Nematodes (roundworms) Necator, Trichuris spp.

Hookworm, whipworm (gut infestation)

Wuchereria, Onchocerca spp. (insect bites)

Elephantiasis, river blindness, filariasis

Table 1.7 Examples of fungi that cause human disease Fungus

Disease

Superficial Malassezia globosa

Tinea versicolor

Cutaneous Microsporum, Trichophyton spp. (dermatophytes)

ringworm, athlete’s foot, etc.

PrIoNS AND TrANSMISSIBLE SPoNGIForM ENCEPHALoPATHIES

This is a short section but we should flag up our ignorance concerning certain diseases and some ‘infectious’ particles. There is great interest in transmissible spongiform encephalopathies which can produce progressive and fatal brain damage in humans (kuru), sheep (scrapie) and cows (bovine spongiform encephalopathy [BSE] or ‘mad cow’ disease). These diseases are experimentally and naturally transmissible with no viruses or bacteria detectable. Prion proteins have been proposed as the cause. They are naturally occurring proteins in most mammals. Their role is not entirely clear, but it seems that they are important in the differentiation of neurons. Prion proteins may be induced to change shape, either spontaneously (as in sporadic Creutzfeldt–Jakob disease [CJD]) or if a mutant or foreign prion protein enters the cell (e.g. variant CJD, thought to be the human equivalent of BSE, caught from infected cattle) (Fig. 1.18). The particle is protein without any evidence of nucleic acid. Its neurotoxic effects are thought to be due to abnormal folding, brought about by a mutation of just a few amino acids, which inactivates the normal relationship between the cytoskeletal components and the proteasome. The misfolding seems to be catching and, once it has begun, the normal protein also becomes misfolded. The effects are to cause degeneration of the brain, which develops a sponge-like texture (‘spongiform encephalopathy’). The disease is characterized by problems with coordination, rapidly progressive dementia and death.

Systemic Histoplasma capsulatum

Pneumonia or disseminated disease

Aspergillus spp. (various)

Pneumonia or disseminated disease (Fig. 1.17)

Candida, Cryptococcus spp.

Disseminated disease in immunosuppressed individuals

Pneumocystis jirovecii

Interstitial pneumonia

Figure 1.17 Aspergillus spp.

Chapter 1: What causes disease?

Table 1.6 Examples of helminths that cause human disease

21

22

What causes disease?

Table 1.8 Examples of inflammatory conditions

Figure 1.18 Diffusion-weighted MrI of the brain showing ‘bright’ areas representing cytotoxic oedema in the cortex of a 60-year-old patient with rapidly progressive dementia. This is a variant of prion disease called Creutzfeldt–Jakob disease.

Part 1: Disease, health and medicine

INFLAMMATorY CoNDITIoNS These cover a broad range of causes and mechanisms. The mechanisms are based on the different types of hypersensitivity reactions (see pages 80–86). The causative allergens can be environmental, genetic or a mixture (Table 1.8). CoELIAC DISEASE: GENETIC PrEDISPoSITIoN AND HYPErSENSITIVITY

Coeliac disease is an example of a genetic predisposition to disease. In such patients, inherited characteristics that are dictated by the major histocompatibility complex (MHC) class I or II molecules on the surfaces of all nucleated cells (type I) or antigen-presenting cells (APCs, type II) mean that a person is at increased risk of developing a particular disease. Coeliac patients almost always have either MHC molecules of HLA-DQ2 or -DQ8 type displayed by their APCs.

Disease

Target antigen

Asthma

Pollen, dust mite

I

Myasthenia gravis

Acetylcholine receptor

II

Pernicious anaemia

Intrinsic factor of gastric parietal cells

II

Systemic lupus Nuclear antigens erythematosus

III

reactive arthritis

III

Bacterial antigens, e.g. Yersinia spp.

Hypersensitivity type

Type 1 diabetes Pancreatic islet mellitus  cells

IV

rheumatoid arthritis

IV

? collagen

If gluten molecules enter the lamina propria of the small intestine in an undigested state they may trigger an immune response. Why this should happen has not yet been fully elucidated, but gliadin, the antigenic component of gluten, is said to be particularly resistant to digestive enzymes. The key to the genetic element is that people with MHC-DQ2 or DQ8 MHC class II molecules on the surfaces of their APCs can easily bind and display deamidated gliadin peptides, altered by the tissue enzyme tissue transglutaminase (tTG). The immune response causes enterocyte destruction, local inflammation in the lamina propria and formation of specific coeliac antibodies. Affected people develop coeliac disease when sufficient enterocytes disappear. This causes microscopically visible flattening of the intestinal villi and reduces the available absorptive surface. Patients with coeliac disease have malabsorption of all types of food and often complain of diarrhoea, extreme fatigue or abdominal pain. However, presentation is very variable, and often it is the incidental finding of anaemia or investigation for skin rashes or irritable bowel syndrome (IBS)-like symptoms that leads to diagnosis. Osteoporosis may develop, and occasionally osteomalacia, because vitamin D is poorly absorbed. A particular kind of blistering, itchy skin

Inflammatory conditions

rash, dermatitis herpetiformis, is virtually always associated with coeliac disease. Treatment is by withdrawal of the stimulus, gluten, and this is usually sufficient to reverse the process.

23

Patients with coeliac disease often have other ‘autoimmune’ diseases, such as diabetes mellitus (type 1) and thyroid diseases (Fig. 1.19, and see the clinical scenario below).

Normal duodenum Normal slender villi with few intraepithelial lymphocytes

Crypt to villus ratio 1:3

People with type II MHC antigens DQ2 or DQ8 are at risk of developing coeliac disease if wheat protein enters the lamina propria, possibly following an infection. Some enteroviruses are thought to mimic wheat antigens. An anti-inflammatory enzyme, tissue transglutaminase (tTG) deamidates gliadin, changing its three dimensional shape. The altered wheat protein 'fits' a groove in the DQ2 or DQ8 MHC antigens. Intestinal epithelial cells present the wheat antigen associated with MHC II antigens to T cells. The T cells delete the epithelial cells bearing the foreign antigen, causing villous atrophy.

Gluten-induced enteropathy (coeliac disease) Lymphocytes

Near total villous atrophy Crypt elongation, inflammation in lamina propria (LP) and increased lymphocytes in the surface epithelium

LP

Figure 1.19 Coeliac disease occurs in genetically predisposed individuals who are exposed to gluten molecules that enter the lamina propria in an undigested state. A reaction to gluten in these patients causes enterocyte destruction and local inflammation in the lamina propria. removal of gluten from the diet is curative and prevents long-term complications of the disease. APC, antigen-presenting cell; MMP, matrix metalloproteinases; tTG, tissue transglutaminase enzyme.

Chapter 1: What causes disease?

Removal of wheat from the diet restores normality in 3-6 months. Relapse occurs within days if wheat is eaten again.

24

What causes disease?

Case study: coeliac disease Clinical

Pathology

A slightly built 25-year-old woman consulted her family doctor. She complained of fatigue and breathlessness on exertion, worse over the last 6 months. On examination, she was of medium height (1.65 m), low weight (50 kg) (see BMI chart in Fig. 1.20) and had pale mucous membranes. Glancing through her medical notes, the GP observed that she had been diagnosed with IBS a few months ago, having complained of abdominal pain and bloating. He also noticed that she had some bruising.

Pallor and symptoms suggest anaemia. Iron deficiency due to menorrhagia is the most common cause of anaemia in a young woman, but bruising would be unusual. Of patients with coeliac disease, 40% have IBS. This is a common diagnosis; however, 35–40% of patients with IBS will have coeliac disease if investigated.

Investigations were as follows: l l l l l l l l

Hb: 9.6 (normal 11–13.5) g/dL MCV: 86 (normal 78–95) fL WCC: 3.8 × 109/L (normal 3–5.5 × 109/L) Platelets: 330 × 1012/L (normal 150–400) × 1012/L Red cell folate: 152 (normal 150–750) μg/L Vitamin B12: 68 (normal 150–1000) ng/L Iron: 13 (normal 14–30) μmol/L International normalised ratio (INR): 2.5 (normal 0.8–1.1)

Provisional clinical diagnosis: malabsorption, probably due to coeliac disease (gluten-induced enteropathy)

Microcytic anaemia is the hallmark of iron deficiency and macrocytic anaemia typically indicates folic acid and/or vitamin B12 deficiency. This patient has a low serum iron and low normal red cell folate, and her anaemia is normocytic, which means that the average red cell size is normal. Prolonged prothrombin time (measured as the INR) is likely to be due to decreased vitamin K causing a deficiency of clotting factors (see Table 1.9).

The incidence of coeliac disease in the UK is approximately 1:100 people. Many patients have subclinical disease.

Management and progress

Part 1: Disease, health and medicine

She was referred for investigation at the gastroenterology clinic in her local hospital. The results of the investigations were as follows:

l

l l

Upper gastrointestinal (GI) endoscopy and duodenal biopsy: macroscopically normal, but biopsy showed subtotal villous atrophy and increased intraepithelial lymphocytes. Serology for anti-tTG IgA antibodies was positive. Bone density scan showed early osteoporosis.

Subtotal villous atrophy with crypt hyperplasia and increased intraepithelial lymphocytes is the typical microscopical appearance in coeliac disease, due to increased destruction of enterocytes by a T-cell-mediated reaction. IgA anti-tTG will diagnose 95% of patients with untreated coeliac disease. IgA anti-endomysial antibodies are even more specific (99%) but slightly less sensitive and the test is more expensive. (Of patients with coeliac disease 2–4% are IgA deficient, compared with 0.2–0.5% of the general population.) IgG antibodies to tTG or endomysial antibodies are slightly less sensitive but are used if the patient is IgA deficient. Testing for deamidated gliadin is fractionally less sensitive. Her diagnostic endoscopic biopsy in coeliac disease showed subtotal villous atrophy, increased lymphocytes among the surface enterocytes, lamina propria inflammation and crypt hyperplasia. The mild osteoporosis noted on the bone density scan was due to vitamin D malabsorption, with resultant poor calcium absorption.

25

The suspected diagnosis of coeliac disease (gluteninduced enteropathy) was confirmed serologically and by endoscopic duodenal biopsy, and she was advised to exclude wheat, barley and rye from her diet (a gluten-free diet). oats contain avenin, to which some people with coeliac disease are sensitive. She felt symptomatically improved after 1 month.

A gluten-free diet is difficult to sustain in western countries: wheat is present in bread, cakes, biscuits and sauces. Any trace is sufficient to spark a recrudescence of disease. Social occasions are fraught with difficulty.

After 6 months she returns to the clinic. She is bored with the gluten-free diet. She feels well and is putting on weight. She requests a return to a normal diet.

Short-term problems due to coeliac disease include vitamin deficiencies, growth retardation (in children), steatorrhoea, malnutrition and anaemia, and osteoporosis. These are relatively quickly improved on a gluten-free diet. She must not give up the gluten-free diet, however, or the problem will recur.

repeat duodenal biopsy after 6 months showed greatly improved microscopical appearances, which were almost normal.

The duodenal biopsy may take up to 1 year to return to normal (but just days to show subtotal villous atrophy again after a gluten challenge!).

She is warned to remain on a strict gluten-free diet, to prevent long-term sequelae.

Long-term problems include an increased incidence of malignant gastrointestinal tract tumours, e.g. patients with coeliac disease are at 50–100 times the normal risk of developing malignant lymphoma and there is a moderately increased risk of small and large bowel adenocarcinoma, and squamous carcinoma of the oesophagus. It appears that the increased risk can be averted by adherence to a gluten-free diet.

other members of her family are screened for occult coeliac disease. Should the general population be screened for coeliac disease?

The prevalence of coeliac disease in relatives is as follows:

18.5

m ft in

25

27

1.88 6'2"

1.62 5'4"

we igh t

t igh Ov erw e

Som

ee

xce

ss

hy wei g alt He

Height

ht

1.78 5'10"

1.68 5'6"

First-degree relatives: 10%

l

HLA-identical siblings: 30%

l

Dizygotic twins: 25%

l

Monozygotic twins: 70–100%.

There are strong associations between HLA-DQ2 (95% of patients) and HLA-DQ8 and coeliac disease. All patients with coeliac disease inherit a particular HLA type, which renders them more likely to develop the disease – if negative it is not coeliac disease. However, HLA-DQ2 is common, present in 40% of the world’s population, of whom only 3–5% will have coeliac disease, so serology for coeliac antibodies and endoscopic biopsy are recommended for diagnosis.

1.83 6'0"

1.72 5'8"

l

1.57 5'2" 1.52 5'0" kg lb

45.5 100

54.5 120

63.5 72.5 140 160 Weight

81.5 180

91.5 200

100.5 220

Figure 1.20 Body mass index (BMI) is calculated as follows: weight (kg)/height (m)2. Values from 18.5 to 25 are healthy. our coeliac patient’s BMI is 18.3, so she is fractionally underweight.

Chapter 1: What causes disease?

Inflammatory conditions

26

What causes disease?

Table 1.9 Clinical consequences of malnutrition Nutrient deficiency Clinical effect Calories

Fat loss Muscle wasting organ atrophy Growth failure in children

Protein (kwashiorkor)

As above but without fat loss and with oedema and fatty liver

Fat-soluble vitamins Vitamin A

Epithelial changes affecting eyes, skin and viscera

Vitamin D

rickets and osteomalacia

Vitamin E

Neuromuscular degeneration

Vitamin K

Haemorrhagic disease of the newborn Effect on anticoagulants

Water-soluble vitamins Vitamin C

Scurvy (bleeding, poor wound healing, bone lesions)

Vitamin B1 (thiamine)

Beri-beri (neural, cardiac and cerebral problems)

Part 1: Disease, health and medicine

Vitamin B12 Pernicious anaemia (cyanocobalamin) Niacin

Pellagra (dermatitis, dementia, diarrhoea)

Folate

Megaloblastic anaemia

Vitamins B1 and B2 (riboflavin)

ocular lesions, glossitis, stomatitis

Vitamin B6 (pyridoxine)

Infant convulsions, anaemia, dermatitis, glossitis

Minerals Iron

Microcytic, hypochromic anaemia

Copper

Nerve and muscle dysfunction

Iodine

Goitre

Zinc

Growth retardation and infertility

Selenium

Myopathy and cardiomyopathy

NuTrITIoNAL DISEASES For centuries, the major problems in nutrition were centred around deficiencies. Too little protein resulted

in kwashiorkor, whereas too few calories in the first year of life produced marasmus. Other diets lacked one or more vitamins or minerals because the variety of foodstuffs was not available. These problems, sadly, still exist in some developing countries but, in the developed world, malnutrition is more likely to be a consequence of bowel problems, other illnesses or psychosocial issues, although nutritional imbalance has become a major public health problem for otherwise healthy people. The key point is that nutritional imbalance for normal adults is a choice, and does not have a biological cause. The most common imbalance is ingesting too much. The consequence of excess calorie intake is obesity, with its long-term effects on cardiovascular disease, diabetes, osteoarthritis and some cancers. The other most commonly over-ingested substance is alcohol. Chronic alcoholism is now known to produce most of its damage through direct toxic effects rather than secondary to nutritional deficiencies and results in central nervous system (CNS) atrophy, cardiomyopathy, peptic ulcers, pancreatitis, liver damage, varices, testicular atrophy and upper GI tumours. We are now becoming much more aware of the effects of other imbalances in our diet. Too much salt worsens hypertension, too much fat increases the amount of atheroma and the risk of heart attack and strokes, and certain foods (e.g. smoked foods) may increase the occurrence of GI tumours. Metabolic syndrome is a common condition in developed countries and is generally considered to be at least partly due to a mismatch between our nutrition and our biochemistry. It is diagnosed when there is abdominal (central) obesity and two abnormalities of raised blood pressure, raised fasting plasma glucose, raised serum triacylglycerols or reduced highdensity lipoprotein (HDL) levels (criteria International Diabetic Federation 2006 http://www.idf.org). It is considered further in the section on cardiovascular disease because it is a major risk factor. The pathophysiology of metabolic syndrome is complex, but it can be induced by eating too much sucrose and fructose (sucrose is the disaccharide consisting of glucose and fructose). It is thought that the fructose is metabolised in the liver to short-chain fatty acids; this overloads the liver, leading to elevated blood triacylglycerol levels which induces ectopic fat (i.e. fat around the viscera and in organs not designed for fat storage). Fructose is predominantly metabolised in the liver, in contrast to glucose, which is used by muscle.

Genetic causes of disease

Overprovision of dietary calories compared with energy used in physical activity could also conceivably create an excess of mitochondrial oxidation products and result in insulin resistance, which is a key feature of metabolic syndrome. Other risk factors for metabolic syndrome are increasing age, lack of physical activity and stress. The amount and type of food that we eat is influenced by social and psychological factors beyond the scope of this book. There are also genetic factors, including leptin coded for by the LEP gene. This is secreted by adipocytes and acts by reducing food intake and increasing energy expenditure. It is part of a complex system including ghrelin, peptide YY and insulin which acts on the brain to influence appetite. Through feedback loops, this aims to balance intake with output and is an example of homeostasis. The food that we eat and the level of exercise are lifestyle choices for most people but some are unfortunate and have specific problems with ingesting, absorbing, metabolising or controlling excretion of essential nutrients (Table 1.10).

27

These conditions mostly affect general nutrition and this is what you will most commonly see, especially in those with chronic illness. Specific deficiencies are less common, with iron, folate, vitamin D and vitamin B12 probably being most important. If you are working in areas where nutrition is poor then make sure that you are aware of the clinical effects of all of them (Table 1.9).

GENETIC CAuSES oF DISEASE Genes are so fundamental to the functioning of every cell that it would be hard to think of a disease in which alterations in gene activity did not occur. However, that would not mean that the disease was caused by an abnormal gene; so in this section we are interested in diseases that result from specific gene abnormalities and what might cause those abnormalities. First let us look at some common ones (Table 1.11).

Table 1.10 Conditions affecting general nutrition Mechanism

Examples

reduced ingestion

Psychiatric illness Anorexia, e.g. linked to malignancy or chronic illness Food allergy GI disorders

reduced absorption Gut hypermotility Inflammatory bowel damage Pancreatic or biliary disease Achlorhydria Abnormal metabolism

Malignancy Hypothyroidism Liver disease

Increased excretion

Diarrhoea

Increased demand

Fever Pregnancy and lactation Hyperthyroidism

Table 1.11 lists the incidence per 1000 live births of the most common genetic disorders. It is helpful to subdivide them into abnormalities of chromosomal structure or number. The single-gene disorders are due to abnormalities of structure, which will be inherited in a mendelian fashion. Abnormalities of chromosomal number are not normally inherited. There are several points to highlight. The first is that the incidence relates to live births, which means that genetic abnormalities causing intrauterine death will be under-reported. This principally influences the figures for chromosomal abnormalities, because their incidence in spontaneous abortions and stillbirths is 50% whereas the incidence in live births is 6.5/1000. In spontaneous abortions with chromosomal abnormalities, around 50% will have a trisomy, 18% will be Turner’s syndrome (XO) and 17% will be triploid. The most common condition is X-linked red–green colour blindness, which, fortunately, is only a very minor handicap (and is not an excuse for avoiding histology sessions!). Klinefelter’s syndrome is due to an extra X chromosome in males (47,XXY). Affected individuals are generally of normal intelligence and are tall, with hypogonadism and infertility. XYY syndrome also produces tall males. They may have behavioural problems, especially impulsive behaviour.

Chapter 1: What causes disease?

INCIDENCE

28

What causes disease?

Table 1.11 Common genetic disorders Condition

Estimated frequency/1000 live births

red–green colour blindness

80

X

Total autosomal dominant disease

10

AD

Dominant otosclerosis

3

AD a

Klinefelter’s syndrome (XXY)

2

N

Familial hypercholesterolaemia

2

AD

Sickle cell disease

2

Ar

Total autosomal recessive disease

2

Ar

Trisomy 21 (Down’s syndrome)

1.5 a

N

XYY

1.5

N

Adult polycystic kidney disease

1

AD b

Triple X syndrome

0.6

Cystic fibrosis

0.5

Fragile X-linked learning disability

N Ar

a

X

a

0.5

Non-specific X-linked learning disability

0.5

X

recessive learning disability

0.5

Ar

Neurofibromatosis

0.4

Ar

Turner’s syndrome (Xo) Duchenne muscular dystrophy

Part 1: Disease, health and medicine

Abnormality

a

b

N

a

X

a

0.4

0.3

Haemophilia A

0.2

X

Trisomy 18 (Edwards’ syndrome)

0.12

N

Polyposis coli

0.1

AD

Trisomy 13 (Patau’s syndrome)

0.07

N

AD, autosomal dominant; Ar, autosomal recessive; N, disorder of chromosomal number; X, sex-linked disorder. a Number/1000 male births. b Number/1000 female births.

In familial hypercholesterolaemia, patients have increased plasma low-density lipoprotein (LDL) levels and a predisposition to developing atheroma at an early age, which gives them an eightfold increased risk of ischaemic heart disease. The primary defect is a deficiency of cellular LDL receptors, so that the liver uptake is reduced and plasma levels are two to three times normal. Around 30 different mutations of the LDLreceptor gene have been identified. About 1 in 500 people are affected and they are heterozygotes who have half the normal number of LDL receptors. One in a million people is a homozygote and he or she usually dies from cardiovascular disease in childhood.

The incidence of sickle cell disease varies significantly in different populations and is greatest in groups that have originated in malaria-infested areas. In US African–Americans, the incidence of disease is 1 in 500 and the carrier heterozygous state is 1 in 12. For details on sickle cell disease see page 279. Adult polycystic kidney disease is due to a defect on the short arm of chromosome 16 which is inherited in an autosomal dominant fashion. Both kidneys are enlarged, with numerous fluid-filled cysts and may weigh a kilogram or more (normal is 150 g). The patients develop symptoms of renal damage and hypertension in their third or fourth decade (Fig. 1.21).

Genetic causes of disease

Figure 1.21 Adult autosomal dominant polycystic kidney disease. Gradually cystic dilatations of renal tubules and collecting ducts accumulate and distend the kidney, causing pressure atrophy of the renal tissue.

Triple X syndrome produces tall girls who may have below average intelligence; in addition, although gonadal function is usually normal, there may be premature ovarian failure. Fragile X syndrome was first described in 1969 and is now recognised as the second most common cause of severe learning disability after

29

Down’s syndrome. Affected males have a reduced IQ, macro-orchidism, and a prominent forehead and jaw. Heterozygote females can show mild learning disability but counselling is difficult because not all female carriers show the chromosomal abnormality on testing. Turner’s syndrome (monosomy X, i.e. 45,X) is a common cause of fetal hydrops and spontaneous abortion. About 95% of affected pregnancies will abort. Those surviving to delivery will be less severely affected and generally show short stature, webbing of the neck, normal intelligence, infertility, aortic coarctation and altered carrying angle of the arm (cubitus valgus). These disorders all relate to genes in the germ cell lines that give rise to all the cells in the body. Genes can also mutate during life and this is most obvious in various cancers. The mechanisms operating at the gene and cell level in cancers are covered on page 324 of Chapter 13. Although we have listed the common genetic disorders and their inheritance pattern, this does not answer the question: ‘How are they caused?’ This is really two questions: 1 How does the genetic abnormality produce disease? 2 How does the genetic abnormality arise?

Basic biology

Just to remind ourselves of the basic biology: there are 23 pairs of chromosomes with one of these pairs being the sex-determining chromosomes, X and Y. Each chromosome is formed of two chromatids, joined by a centromere and with telomeres at each end. The appearance of these chromosomes, stained with Giemsa during mitosis, is the basis of classic cytogenetics and karyotyping, and the simplest way to detect changes in chromosome number or structure (see below). Each chromatid is cleverly packaged DNA. If the human genome were constructed as a single straight strand, it would be 6 feet long, yet it fits into a tiny nucleus! DNA is composed of purine (adenine and guanine) and pyrimidine (cytosine and thymine) bases, arranged as pairs in a helical ladder. This DNA helix forms double loops around nucleosomes (globular aggregates of histone proteins) to produce a beaded string structure. This beaded string is coiled into a solenoid, composed of five or six nucleosomes per turn. This is chromatin in its relaxed state. During mitosis and meiosis, there is further condensation involving non-histone proteins. For those who like numbers, the DNA in each human cell contains around 7000 megabase-pairs (Mbp) (1 Mbp = 1 million base-pairs). only 3% of the genome codes for genes (the exons). There are around 25 000 nuclear genes and a small but important set of 37 genes in the mitochondria that are inherited exclusively from the mother. There is much interest in the 97% non-coding DNA and its functions, some of which are related to gene regulation. You will appreciate that breaks in chromosomes, deletion of segments and point mutations in DNA will have different impacts depending on whether they affect exons or non-coding sections. (Continued)

Chapter 1: What causes disease?

Chromosomes, DNA and genes

30

What causes disease?

(Continued) Haploid cells have only one set of chromosomes, i.e. ‘n’ as occurs in gametes. Diploid cells have a normal number of chromosomes, i.e. 46= ‘2n’ as in all somatic cells. Polyploid cells have extra sets of chromosomes, i.e. ‘xn’. Aneuploid cells have an abnormal number of chromosomes but not an exact multiple of the haploid ‘n’ state. Mosaicism is the presence of two or more cell lines that are both karyotypically and genotypically distinct but are derived from the same zygote. These generally arise from post-zygotic, mitotic non-dysjunction. Sex chromosome mosaicism is more common than autosomal mosaicism. Short region of DNA double helix

2 nm

‘Beads on a string’ form of chromatin

11 nm

30-nm chromatin fibre of packed nucleosomes

30 nm

Part 1: Disease, health and medicine

Section of chromosome in an extended form

300 nm

Condensed section of chromosome

700 nm

Entire mitotic chromosome

1,400 nm

Centromere

Genetic causes of disease

31

HoW DoES THE GENETIC ABNorMALITY ProDuCE DISEASE?

HoW DoES THE GENETIC ABNorMALITY ArISE?

Genes code for proteins and so the disease pathophysiology will depend on what the normal protein does and how the changed protein functions. Going back to our list of common genetic disorders, we can now list the abnormal proteins and their function (Table 1.12). Sometimes molecular biology can seem daunting with a vast amount of detail that can be difficult to relate to clinical practice. Often a way to simplify this is to decide into which category the abnormality falls and then to understand the typical mechanisms operating for that category. The majority of genes code for proteins and proteins can be:

We need to consider abnormalities of chromosome number separately from abnormalities in chromosome structure or genes, because different mechanisms operate.

l l l

structural receptors or ion channels enzymes growth regulators.

In Chapter 2, we look at the mechanisms for how genes operate in abnormalities affecting the embryo, our biochemistry and the immune system. We choose examples in the embryo that affect cell differentiation which is structural; in our biochemistry the examples are linked to enzyme abnormalities and in the immune system the example is the generation of diverse surface receptor molecules to detect antigens. The topic of abnormalities in genes affecting growth regulation is covered in Chapter 11.

This most commonly occurs because of problems at the anaphase stage of either meiosis 1 or meiosis 2 during production of the gametes. This leads to unequal sharing of the chromosome material so that, after fertilisation, one daughter cell will have an extra chromosome (trisomy) whereas the other is missing a chromosome (monosomy) (Fig. 1.22). This is called non-dysjunction and can affect a pair of homologous chromosomes in meiosis 1, or sister chromatids in meiosis 2. It can also occur through delayed movement (anaphase lag) of chromosomes, so that one is left on the wrong side of the dividing wall. The cause is unknown but the incidence increases with maternal age, as discussed when considering Down’s syndrome (page 38 or case study). If non-dysjunction occurs in mitosis of normal tissue, it can result in mosaicism. Polyploidy means that the cell contains at least one complete extra set of chromosomes. Most commonly, this is one extra set, i.e. 69 chromosomes, or triploidy. Affected fetuses usually die in utero or abort in early pregnancy. It can result from fertilisation by two sperm (dispermy) or from fertilisation in which either the sperm or the ovum is diploid because of an abnormality in their maturation divisions.

Table 1.12 Genetic abnormalities and their dysfunctional proteins Disease

Abnormal protein

Function

Familial hypercholesterolaemia

Low-density lipoprotein receptor

receptor transport

Neurofibromatosis type 1

Neurofibromin 1

Growth regulation

Adult polycystic kidney disease

Polycystin 1

Cell–cell and cell–matrix interactions

Cystic fibrosis

CF transmembrane regulator

Ion channel

Sickle cell disease and thalassaemia

Haemoglobin

oxygen transport

Haemophilia A

Factor VIII

Coagulation

Muscular dystrophy

Dystrophin

Structural support: cell membrane

Fragile X syndrome

FMrP

rNA translation

Chapter 1: What causes disease?

l

Abnormal chromosome number

32

What causes disease?

1° oocyte or 1° spermatocyte Diploid (2N) with double chromatids (4C) (one pair of chromosomes shown)

Crossover occurs between maternally and paternally derived chromatids in prophase 1 In females; halted at this point until ovulation ie duplicated and crossed over but not separated

Anaphase I

Chromosomal non-disjunction results in 2N and 0N in secondary oocytes

Normal

2° oocytes and 2° spermatocytes contain 23 chromosomes, each with 2 chromatids

N, 2C

2N, 4C

0N, 0C

In females; halted at this point until fertilisation occurs when large haploid (N) ovum pronucleus is produced to fuse with sperm pronucleus and second polar body (N) is shed Anaphase II Normal

Chromatid non-disjunction produces abnormal ovum pronuclei

N, 2C

Ovum pronuclei

Part 1: Disease, health and medicine

N, C sperm form

0N, 0C

N, C

N, 2C

N, C

NC Trisomy and monosomy will result if affected gametes are fertilized Fertilized egg N + 2N, C + 2C

N, C N + N, C + C Normal

Figure 1.22 Abnormal chromosome numbers may arise through problems in Anaphase I and Anaphase 2 of meiosis. resting cells (and fertilised eggs) have 23 pairs (2N) of single stranded (2C) chromosomes. Dividing cells (and 1º oocytes/ spermatocytes) have double strands (4C) known as sister chromatids. Highlighted in red are the abnormal secondary oocytes produced by chromosomal non-disjunction in Anaphase 1, abnormal ovum pronuclei produced by chromatid non-disjunction at Anaphase 2 and abnormal fertilized eggs. The left hand side shows normal meiosis for comparison.

Genetic causes of disease

33

p arm Centromere

Metacentric, (chs 1,3,16,19,20,X)

q arm Submetacentric, (chs 2,4,5,6,7,8,9, 10,11,12,17,18)

Deletion

Acrocentric, (chs 13,14,15,21,22,Y) Translocation

Break point

Reciprocal Two chromatids exchange DNA segments Inversion

Pericentric

Paracentric

Non-reciprocal One chromatids donates DNA to another

Ring formation

New chromatid formed of q arms Key: dissolution of redundant chromosome

Figure 1.23 Structural chromosomal abnormalities.

Chapter 1: What causes disease?

Robertsonian translocation Generally involves two acrocentric chromosomes

34

What causes disease?

Abnormal chromosome structure

Abnormalities in chromosome structure occur when chromosomes are inaccurately repaired after breaks have occurred. Chromosomal breakage can happen randomly at any gene locus, but there are some areas that are particularly liable to breakage. The rate of breakage is markedly increased by ionising radiation and certain chemicals, and some rare inherited conditions. Structural abnormalities, such as translocations, deletions, duplications and inversions (Fig. 1.23), occur when two break points allow transfer, loss or rearrangement of chromosomal material. Abnormalities in chromosome number and structure have been identified for many years using classic cytogenetic methods and various karyotypes are associated with specific diseases (Table 1.13). Gene level changes

Once we are thinking about single genes and disorders that may be produced by changes in those genes, it is worth reminding ourselves that we do not all have the same genes. At any locus, there may be multiple possible alleles that vary in their frequency with ethnic origin, depending on their survival advantage. This is normal polymorphism. Over 30% of genes coding for proteins are polymorphic. When patients react

differently to diseases, it is now possible to look at their DNA sequence and compare it with the known genes listed in worldwide databases. If there are differences, then it may be relevant because of the known function of that gene, e.g. resistance to HIV occurs in people with a naturally occurring different gene for the T-cell receptor CCR5, due to a 32-bp deletion (Δ32) causing a frameshift mutation. Homozygotes can become HIV positive but do not develop AIDS because the new protein disrupts intracellular signalling. The allele is most common in Finns (16%) and virtually absent in Africa. Many agents are capable of causing mutations, and mutations also occur spontaneously (Fig. 1.24). Physical causes include ultraviolet (UV) light, electromagnetic radiation and atomic radiation. UV light normally links adjacent pyrimidine bases and causes only somatic cell mutation not germline mutation. It is a major cause of skin cancer, especially in red-haired individuals whose lack of pigment allows release of damaging free radicals when exposed to sunlight. Electromagnetic radiation acts by knocking electrons out of their orbits and causes single- and double-strand breaks in DNA and base-pair destruction. This can result in major deletions, translocations and aneuploidy. They have most effect on dividing cells such as sperm and in early embryonic development. Chemical and viral mutagens often result in

Part 1: Disease, health and medicine

Table 1.13 Examples of karyotypes 46XY

Normal male

46XX

Normal female

47XXY

Male with Klinefelter’s syndrome

If there is a change in chromosomal number, then the affected chromosome is indicated with a + or -, e.g. 47XX+21

Female with Down’s syndrome

If there is a structural rearrangement, the karyotype indicates the precise site affected and the nature of the abnormality, e.g. 46XXdel7(p13-ptr)

Deletion of the short arm of chromosome 7 at band 13 to the end of the chromosome

46XYt(11;14)(p15.4;q22.3)

A translocation between chromosome 11 and 14 with the break points being band 15.4 on the short arm of chromosome 11 and band 22.3 on the long arm of chromosome 14

Mosaicism indicates that two different cell lines have derived from one fertilised egg and the karyotype specifies both cell lines 46XX/47XX+ 21

Down’s syndrome mosaic

46XX/45X

Turner’s syndrome mosaic

Genetic causes of disease

35

MUTATION New sequence in viable cell Single-strand break DNA damaged by: Irradiation Drugs UV light Heat (Spontaneous mutation)

Multiple breaks, and random joining of strands

Point mutation

Double-strand break (s)

Cross-linkage Changes Loss of base Alteration of base Bullk lesion Alkylation

Repair, using opposite strand as template

Irreparable damage, transcription impossible

RESTITUTION

DELETION

cancer and the molecular biology of their mechanisms can be found on pages 323 and 335. Mutations at the gene level can either involve millions of bases or be a point mutation. The factors that affect chromosome structure can also occur at a single gene level, i.e. segment deletions, duplications and translocations. Point mutations are usually spontaneous and of unknown cause, but are probably mostly the result of copying errors. Bases may be substituted, inserted, deleted and replicated (amplification). Substitution of one base within a codon may lead to a different amino acid being inserted into the protein and major pathological effects, e.g. sickle cell disease. However, this is not inevitable because there are only 20 amino acids but 64 possible codons (4 × 4 × 4), which is the basis of the degeneracy of the genetic code, e.g. an mRNA sequence of GAA or GAG will code for alanine, so some point mutations can alter the codon but have no effect on the amino acid sequence. These are called silent mutations and approximately 25% of point mutations have no effect. As well as coding for amino acids, codons also act as start and stop instructions. UAA, UAG or UGA is used as a stop codon by mRNA. If a point mutation produces a stop codon, the amino acid chain will

terminate too early and is the effect of about 5% of point mutations. If one or two (but not three) bases are inserted or deleted, this will produce a frameshift mutation with much greater effect, because the code is based on triplets of bases (codons) and so changes dramatically if the groups of three are shifted along to produce a nonsense message. Amplification of sequences of three bases (triplet repeats) is called ‘dynamic mutation’ and is recognised as the cause in about 20 diseases, 3 examples of which are Huntington’s disease, fragile X disease and myotonic dystrophy type 1. In each of these diseases, there is ‘anticipation’ which means that the disease often becomes more severe with each generation or occurs earlier. If the genes are investigated, it is apparent that the number of triplet repeats correlates with the severity. The number needed for severe disease is different in the different conditions, with Huntington’s disease requiring more than 40 CAG repeats, whereas fragile X disease requires several hundred repeats and is more severe in males. The mechanism is also different with the abnormal gene for Huntington’s disease coding for a protein toxic to neurons, whereas the fragile X mechanism blocks transcription by methylating the promoter.

Chapter 1: What causes disease?

Figure 1.24 Causes and types of DNA mutation and their consequences.

36

What causes disease?

Basic biology Gene disorders are most commonly produced by mutations in the DNA sequence. These can occur in the germline or the somatic cells. Mutations (Fig. 1.25) take various forms: l

Substitution of single nucleotide bases, producing missense mutations, e.g. sickle cell anaemia

l

Insertion or deletion of fewer than three bases, causing a frameshift mutation

l

Creation of a stop code by a point mutation

l

Amplification of a sequence of three bases

l

Copy number variations (CNVs).

Normal sequence coding for 3 amino acids (A, B, C) A

B

C

Substitution of a single nucleotide resulting in a different amino acid (or could be silent or produce a stop code) i.e. D amino acid instead of B A

D

C

Insertion causing a frameshift resulting in amino acids E and F

Part 1: Disease, health and medicine

A

E

F

Amplification of a sequence of three bases (triplet repeat expansion) A

B

B

B

C

Figure 1.25 Mutations.

Epigenetic change: this is modulation of genes without alteration of the DNA sequence. Imprinting: this is an epigenetic process during gametogenesis that switches off an allele so that there is monoallelic gene expression. Single nucleotide polymorphism (SNP) is a variation in single nucleotides anywhere in the DNA. They can be useful for DNA fingerprinting and other investigations.

Genetic causes of disease

CNVs are probably responsible for most of the diversity in normal humans and at least half are involved in gene-coding sequences. As they are so common, it is thought that some may convey an evolutionary advantage. They have also been identified in a variety of disorders, such as epidermal growth factor receptor (EGFR) copy number in non-small cell cancer and high CCL3L1 copy number in people with lower susceptibility to HIV infection. Single nucleotide polymorphisms

Over 6 million SNPs have been identified in humans but only 1% are in coding regions and so they may have less biological effect, although they have been useful in studying inheritance if located near (and inherited with) a known disease-associated gene. This method is known as linkage disequilibrium. Epigenetic change

Modulation of genes without alteration of the DNA sequence (epigenetic change) is important for homeostasis and development, and allows regulation of gene and protein expression through alteration of gene promoters and so on, e.g. by methylation of cytosine residues. It is important in cancer progression and treatment. If it occurs as a part of normal development, it is known as imprinting (see later). Imprinting

Imprinting silences genes and occurs to some genes during gametogenesis, so that either only the father’s or only the mother’s allele will be expressed. The mechanisms involved in imprinting differ for different genes, but they are being intensely investigated because imprinting disorders occur in the assisted conception techniques of in vitro fertilisation and intracytoplasmic sperm injection. The most common disorders of imprinting are Prader–Willi, Angelman’s and Beckwith–Wiedemann syndromes. Alterations in non-coding RNAs

These are exciting areas of new research because non-coding RNAs are involved in regulation of genes and, hence, cell pathways. There are estimated to be around 1000 genes in humans for micro-RNAs (miRNAs) which are crucial for post-transcriptional

silencing of genes. There are even more long non-coding RNAs (lncRNAs) that may be important in diseases such as atherosclerosis and cancer. MuLTIFACTorIAL DISorDErS

Every new patient is asked about his or her ‘family history’, the idea being that if the parents and siblings have a particular disease then the patient is at increased risk. Unfortunately, for most diseases it is not known how great that increased risk may be because the inheritance does not follow simple mendelian principles but is rather multifactorial. It is likely that there will be a variety of genes involved that interact with a number of environmental factors. Research into multifactorial disorders adopts a similar approach to single-gene problems. First, it is necessary to identify the diseases with a significant genetic component by comparing the incidence in family groups with that in the general population. This genetic contribution is termed heritability (Table 1.14) and some examples are listed below. The next step is to look for genetic, biochemical and immunological features that affected individuals have in common. It has been well established that certain HLA types are associated with particular diseases and this may be helpful in counselling affected families, e.g. in a family with ankylosing spondylitis, a firstdegree relative has a 9% risk of developing the disease if HLA-B27 positive but less than a 1% risk if HLA-B27 negative. Table 1.14 Heritability or the genetic contribution to the aetiology of the disorder Disease

Estimate of heritability (%)

Schizophrenia

85

Asthma

80

Cleft lip and palate

76

Coronary artery disease

65

Hypertension

62

Neural tube defect

60

Peptic ulcer

35

Chapter 1: What causes disease?

Copy number variations

37

38

What causes disease?

The ultimate goal is to identify the gene or genes and the environmental factor(s), so that those at particularly high genetic risk could attempt to avoid the relevant environmental hazard. At a simple level, this would mean giving vitamin supplements to pregnant women at risk of producing babies with neural tube defects, or advising individuals with potential ‘arteriopathy’ to modify their diet and not smoke. Type 1 diabetes is a disease with a wide geographical variation, in which a genetic predisposition and

environmental factors seem to interact to produce the clinical manifestations of pancreatic islet cell damage. It is associated with HLA and two haplotypes have a particular association with the disease – DR4-DQ8 and DR3-DQ2 – which are present in 90% of children with type 1 diabetes. It is clear, however, that the environment plays a significant role, with viruses being the major culprit. Enteroviruses, rotaviruses and, in particular, rubella appear to be the principal candidates.

Part 1: Disease, health and medicine

Case study: an abnormal baby A 41-year-old mother gives birth to a live male infant. She has had two previous normal pregnancies with the same father. Shortly after delivery, the baby seems to be relatively floppy, although he is moving all four limbs and has been crying and feeding successfully. Examination reveals that the baby’s head is particularly round, the palpebral fissures are upward sloping and the medial epicanthic folds are prominent. The tongue is enlarged and heavily fissured. The baby’s hands are broad and possess only a single palmar crease, whereas the little fingers are short and curved inwards. These features persist over the next few months and there is a delay in the child reaching the early motor milestones. Later, the child does not learn to walk until the age of 3 and his speech is delayed until 54 months, although by this time the diagnosis has already been made. Question 1: What is the diagnosis? Answer: Down’s syndrome – the physical features are characteristic. Question 2: What is Down’s syndrome? Answer: Down’s syndrome is a chromosomal disorder in which the affected individual has trisomy 21. It should be noted that not all of chromosome 21 needs to be present in excess to produce Down’s syndrome and that trisomy of a certain portion of the long arm alone will cause the disease. Question 3: other than confirming the diagnosis in the child, is there any other role for

Answer:

postnatal genetic investigation in this family? Yes. occasionally (around 3% of cases), Down’s syndrome results from a balanced (robertsonian) translocation in one parent. In this situation, the parent has an abnormal karyotype in which the crucial part of one of the chromosome 21s is translocated on to another chromosome. The parent is phenotypically normal because, although the translocated portion of the chromosome is not in the right place, it is still present and can function normally. However, this means that the germ cells also carry the balance translocation and, when they undergo meiosis, a gamete results in which a normal chromosome 21 is accompanied by the augmented other chromosome, so that gamete has a double dose of the long arm of chromosome 21. (Chromosome 14 seems to be susceptible to acquiring the extra part of chromosome 21.) If a parent has a balanced translocation, they are at particular risk of having another child with Down’s syndrome because the underlying cause persists. This is in contrast with the majority of cases in which the mutation is sporadic.

read more in Pathology in Clinical Practice Case 26

Genetic causes of disease

39

Radiology

Imaging, in particular ultrasound, is extensively used in developed countries to identify prenatal disorders. The nuchal translucency scan (NTS) (Fig. 1.26) uses ultrasound to non-invasively measure the amount of fluid collecting subcutaneously in the nape of the fetal neck at 11–13 weeks’ gestation. The measurement is combined with blood tests to give a risk estimation for the presence of aneuploidy (an abnormal number of chromosomes). If the pregnancy is high risk, the mother can elect to have a more invasive test such as chorionic villous sampling or amniocentesis to detect a chromosomal disorder in the fetus. one of the most common disorders that NTS may help to identify is Down’s syndrome (trisomy 21), which may otherwise have a normal morphological fetal ultrasound.

Figure 1.26 (a) A normal nuchal translucency measurement is seen; (b) an abnormally thick nuchal translucency.

At 18–22 weeks the morphology scan is performed. By this gestational age the fetus is virtually fully developed and large enough for a detailed ultrasound examination of the brain, facial bones, spine, heart, abdomen and limbs to be performed. An abnormal morphology ultrasound can trigger further invasive investigations such as amniocentesis, usually at a tertiary referral centre. Some disorders may be surgically treatable at birth and prenatal identification of these disorders, such as sacrococcygeal teratoma (Fig. 1.27), allows the obstetrician and surgeon to closely monitor the pregnancy at a specialist centre, and plan an elective delivery and subsequent surgery. There are other disorders that may be identified which are untreatable genetic disorders. This allows parents to make informed choices whether to continue or terminate the pregnancy. Some centres will perform fetal MrI to help identify disorders. Imaging in the form of MrI or CT is also used in fetal or neonatal demise (see Fig. 1.10) to identify genetic skeletal and visceral anomalies that may explain the death of the child. This also has implications for genetic counselling of the involved families, particularly with regard to future pregnancies.

Figure 1.27 Three-dimensional fetal ultrasound showing a mass arising from the lower back and buttocks of an 18-week-old fetus. This is a sacrococcygeal teratoma, which can be benign or malignant, and is treated with surgical excision at birth.

Chapter 1: What causes disease?

The role of imaging in identifying disease in the unborn baby

CHAPTER 2

WHAT ARE THE COMMON MECHANISMS OF DISEASE? Introduction Genetic effects on the embryo Genetic effects on biochemistry Case study: haemochromatosis Genetic effects on the immune system Case study: shortness of breath Molecular genetics of cancer Cell damage and cell death Clinical scenario: stroke

40 43 45 47 48 49 49 49 50

In considering the Origin of Species, it is quite conceivable that a naturalist, reflecting on the mutual affinities of organic beings, on their embryological relations, their geographical distribution, geological succession, and other such facts, might come to the conclusion that each species had not been independently created, but had descended, like varieties from other species. Charles Darwin (1809–1882)

Part 1: Disease, health and medicine

INTRODUCTION

40

This is the moment to move from the whole patient and delve down into the tissues, cells and molecules. We have to assume that you are familiar with the basics of genetics, molecular biology and cell biology, and so we won’t cover these in detail but will remind you of some key concepts, terminology or interesting examples in ‘basic biology boxes’. To begin gently, we will consider Mendel and his pea experiments. Mendel was born in 1822 and, at the age of 21 years, joined the Augustinian order. He studied at the University of Vienna for 10 years and then entered a monastery at Brunn. He started experimenting with peas in 1856, looking at the probability of inheriting certain characteristics. He published his results in 1865

Clinical relevance of cell changes Case study: fatty liver Necrosis Apoptosis and autophagy Degenerative diseases and ageing Case study: osteoporosis Mechanisms in infectious diseases Hypersensitivity reactions

55 57 58 62 69 70 71 80

but they didn’t come to prominence until 1900 when other botanists were doing similar work. Mendel looked at seven distinct characteristics and only bred plants that differed in one characteristic. For the sake of discussion, let us consider violet and white flowers. He crossed plants with violet flowers with those bearing white flowers to produce the next generation, called the F1 generation. He found that the F1 generation plants all had the same colour flowers – let us say that they were all violet. The F1 plants were then self-pollinated (inbred) to produce the next generation, called F2. Interestingly, there were three plants with violet flowers for every one plant with white flowers. He took this one step further and self-pollinated the white plants, which gave rise to an F3 generation of plants that all had white flowers. Self-pollination of the violet plants produced an intriguing result: some plants produced only violet plants whereas others produced a mixture of white and violet plants in the ratio 1:3 (Fig. 2.1). As Mendel correctly deduced, although the violet plants in the F2 generation all looked the same, they had different inheritance factors. He postulated that each plant must possess two factors that determine a given characteristic, such as colour of the flower. If two plants are crossed, each will contribute one factor to the next generation and it is purely random as to which

Introduction

White flower

41

Violet flower

Both genes code Both genes code for violet for white

Both genes code for white

×

F1

F1 generation of four off spring, all heterozygotes F1× F1

F2

F2 generation (product of two F1 generation parents) of four offspring: expert 1 homozygote (white) + 1 homozygote (violet) + 2 heterozgotes (violoet) Each plant self-pollinated (F2 × F2)

×

×

×

×

Can only produce homozygote (white) Of four offspring, 1 homozygote (white), 1 homozygote (violoet) and 2 heterozygotes (violet)

Figure 2.1 Mendelian inheritance.

Can only produce homozygote (violet)

Chapter 2: What are the common mechanisms of disease?

×

Part 1: Disease, health and medicine

42

What are the common mechanisms of disease?

factor is passed on. This is the law of segregation, also known as Mendel’s first law. We now know that these ‘factors’ are genes on chromosomes, which are paired, and the two genes on the two chromosomes are alleles of each other. In Mendel’s experiment, violet is the dominant allele and white is the recessive allele. The F1 generation has one white plant that is homozygous for the white allele, one violet plant that is homozygous for the violet allele and two violet plants that are heterozygous, i.e. they have one white and one violet allele but the violet one dominates. In simple examples such as these, one allele completely dominates, i.e. if the plant has at least one violet allele then all the flowers will be violet. Sometimes the situation is more complicated because there will be variable penetrance, i.e. the ‘dominant’ allele dominates only in a percentage of cases. The physical basis for Mendel’s ‘factors’ started to be discovered in the 1880s when Walter Fleming, working at the University of Kiel, stained nuclei and observed the changes in chromatin and chromosomes during cell division, which he called mitosis. He realised that cell nuclei came from the division of precursor nuclei with distribution of their chromosomes, but knew nothing of Mendel’s work and did not make the link to inheritance. Friedrich Weismann was another German biologist and he recognised the significance of meiosis for reproduction and inheritance in 1890. He also led on the idea that the characteristics bred true and were not altered during life, in contrast to Lamarck who believed that inherited characteristics could be acquired. To ‘prove’ the point, Weismann removed the tales of 68 mice over five generations producing 901 offspring who all had normal tails! Weismann was also unaware of Mendel’s work but was strongly influenced by Darwin. There was a huge amount of interest in inheritance in the early part of the twentieth century as Mendel’s work was rediscovered and several people reached similar conclusions. Chromosomes were recognised as the physical basis for inheritance and these were studied as the science of cytogenetics. As more data were accumulated, it was recognised that Mendel’s law of segregation wasn’t true for all characteristics – many

characteristics were inherited together. What we now call genes were referred to as ‘units’ and it was postulated that these units were positioned on chromosomes in a linear fashion, were always in the same position (locus), and the likelihood of inheriting characteristics together depended on first whether they were on the same chromosome and second how far apart they were on a chromosome. The American Alfred Sturtevant worked on drosophila flies, between 1915 and 1928, to divine the principles of genetic mapping by realising the significance of unequal crossing-over and genes that formed ‘linkage groups’ and were inherited together. This was the phase of classic genetics based on cytogenetic experiments. Avery, MacLeod and McCarthy, in 1944, demonstrated that DNA was the biochemical basis of genetics; this marked the beginning of the molecular genetics era (however, their ideas were not well received at the time and had relatively little impact and no Nobel Prize). Watson and Crick, in 1953, used crystallographic images of DNA provided by Wilkins and Franklin to discern the structure of DNA, and most crucially its helical nature. In 2007, James Watson became the second person to publish his fully sequenced genome online. I am putting my genome sequence online to encourage an era of personalised medicine in which information contained in our genome can be used to identify and prevent disease and to create individualised medical therapies. James Watson (2007)

Between the discovery of DNA and the widespread availability of whole genome sequencing, there are two crucial technological advances. The first came from Fred Sanger’s laboratory in 1977 when they used the ‘dideoxy’ chain termination method (the Sanger method) to rapidly and accurately sequence long sections of DNA (in contrast to the earlier use of DNA polymerase). This was ultimately used to sequence the whole human genome in 2003. The second is ongoing and is collectively called ‘next-generation sequencing’. These techniques are dependent on sophisticated automation and huge data processing capability as well as

Genetic effects on the embryo

Frederick Sanger (1918–2013) (Fig. 2.2) Sanger was a British biochemist who won two Nobel Prizes. His father was a general practitioner and a Quaker. Sanger was a pacifist and conscientious objector during the Second World War and started his PhD in 1940, investigating whether an edible protein could be obtained from grass. His first Nobel Prize, in 1958, was for determining the complete amino acid sequence of the two chains of bovine insulin, at a time when protein structure was not understood and thought to be amorphous. He correctly concluded that the sequences were precise and different, and that every protein might have a unique amino acid sequence. This was crucial to Crick’s later ideas on how DNA codes for proteins. The second Nobel Prize was in 1984 for the Sanger method for sequencing DNA molecules.

Figure 2.2 Frederick Sanger.

the fundamental biochemical methods. At present, ‘massive parallel signature sequencing’, ‘parallel pyrosequencing’ and ‘Illumina (Solexa) sequencing’ are among the leading types. So what does all this mean for patients and those of us interested in the mechanisms of disease? Let’s look at four settings influenced by genes and their products: 1 Genetic effects on the embryo 2 Genetic effects on biochemistry

3 Genetic effects on the immune system 4 Molecular genetics of cancer. These provide examples related to different actions of proteins: l l l l

Structural Enzymes Receptors or ion channels Growth regulators.

GENETIC EFFECTS ON THE EMBRYO The obvious place to start is in the embryo with body patterning, which is the term used to describe the regular features of our anatomy. If fertilisation is day 0 and implantation occurs at day 6, then the embryo is forming at 14 days and the major organs are formed by 42 days. The key embryological cell layers are ectoderm, endoderm and mesoderm, and the cells need to know what their position is in the embryo and how they should differentiate. This is achieved by creating gradients across and along the body to distinguish anterior from posterior, right from left, and dorsal from ventral. You wouldn’t be surprised to learn that this is mediated through differential gene expression, but in a particularly interesting way because of the need to set up linear gradients so that the genes are arranged in a co-linear way. The anteroposterior axis is dependent on the four HOX gene clusters and these gene clusters have up to 13 genes arranged in line (co-linearity), with the genes at the 3-end being activated earlier than those further down the line. Not surprisingly, this creates a linear gradient of gene products, but what are the gene products? They are proteins containing a domain that can bind specific DNA sequences in the nucleus to regulate other genes crucial for forming specific body structures. Dorsoventral differentiation and right–left differentiation are more complex but one important clinical example of an abnormality in dorsoventral differentiation involves abnormalities of the SHH (sonic hedgehog) gene. This affects Shh protein expressed in the notochord and neural tube folding, potentially resulting in

Chapter 2: What are the common mechanisms of disease?

History box

43

44

What are the common mechanisms of disease?

Structure of the four paralogous HOX clusters Order of expression is from the right (3’) to left (5’) 5’ HOXA Chr.7p14 A13 A11 A10 A9 A7 A6 A5 HOXB Chr.17q21 B13 B9 B8 B7 B6 B5 HOXC Chr.12q13 C13 C12 C11 C10 C9 C8 C6 C5 HOXD Chr.7q31 D13 D12 D11 D10 D9 D8

3’ A4

A3

A2

A1

B4

B3

B2

B1

C4 D4

D3

D1

Anterior expression limits of the HOX genes in the early CNS Rhombomeres Hindbrain r9 r8 r7

r6 r5 r4

r3 r2 r1

Midbrain

Forebrain Overlapping expression patterns of the four HOX gene clusters 1 2 3 4 5 6 7 8 9 10 11 12 HOX gene expression

Figure 2.3 Linearity of HOX genes results in an antero-posterior linear gradient of gene products important for early body patterning. From Medical Genetics at a Glance. 3rd Edn, D J Pritchard, B R Korf.

Part 1: Disease, health and medicine

Basic biology l

l

l

Homeobox: DNA sequence involved in morphogenesis Homeodomain: protein domain coded by homeobox DNA, which itself can bind and regulate nuclear DNA, generally by acting on promoter regions. Human genes are written in italic capitals (e.g. SHH) and non-human are in italic lower case (e.g. Shh).

non-division of the forebrain (holoprosencephaly) and cyclopia (single central eye). Shh protein is also one of the first signals in right–left differentiation as the heart

tube loops. Other genes include Lefty1 (LEFTA), Lefty2 (LEFTB) and Nodal (NODAL). Nodal and Lefty1 are activated by asymmetrical flow of fluid and this depends on having functioning cilia to waft the fluid. Kartagener’s syndrome is the combination of dysfunctional cilia, situs inversus (mirror imaging of heart, lungs, liver, spleen, stomach and small bowel), chronic sinusitis and bronchiectasis. It is caused by a variety of cilia-related genes but especially those coding for the dynein arms on the microtubules. In children and adults, the abnormal cilia fail to move bronchial mucus normally or create flow along the fallopian tubes and so they have increased chest infections and reduced fertility. They also have random situs (i.e. the right–left arrangements of the heart, lungs and abdominal organs can be the wrong way round), and this is all explained by the underlying problem with cilia.

Genetic effects on biochemistry

l

l

l

l

A primary failure in development (malformation), e.g. cleft lip and palate Damage to a normally formed organ (disruption), e.g. loss of fingers or arm by an amniotic band, damage due to infections such as cytomegalovirus Mechanical distortion (deformation), e.g. dislocation of hips because of a lack of amniotic fluid (oligohydramnios) Abnormal tissue organisation (dysplasia), e.g. kidney cysts which are often monogenic in cause.

GENETIC EFFECTS ON BIOCHEMISTRY

Chromosomal Limb Genital Urinary Abdominal wall defects Digestive system Oro-facial clefts Respiratory Heart defects Nervous system 0

20

30

40

50

60

70

Figure 2.4 Estimated prevalence per 10,000 births of congenital anomalies in England and Wales 2010. Drawn from data in Springett A, Morris JK. Congenital Anomaly Statistics 2010: England and Wales. London: British Isles Network of Congenital Anomaly Registers, 2012.

being ingested, so DNA testing is likely to be used increasingly. After birth, around 40 disorders can be screened for using tandem mass spectrometry on heelprick blood samples rather than the time-honoured approach of examining the urine of newborn babies (e.g. maple syrup urine disease, named because of its sweet smell). CELL AND TISSUE DEPOSITS

In several of these disorders it is possible to actually see the accumulated substance with simple stains down a light microscope. Cells can accumulate substances that may cause damage or be harmless. In addition to inborn errors, the main mechanisms are as follows: l l

‘Inborn errors of metabolism’ have been recognised for many decades, and over time the biochemical and genetic details have been elucidated. The most important are listed in Table 2.1. There are some general principles worth appreciating. Most of these conditions involve loss of a specific enzyme, leading to accumulation of the enzyme substrate and lack of its normal product. Diagnosing these conditions can involve measuring the accumulated substrate, assaying the enzyme activity or looking specifically for DNA mutations. Prenatal diagnosis is often important at stages of development when the enzyme would not be functioning, the relevant tissue is not accessible or the dietary substrate is not yet

10

l

Altered metabolism, e.g. fatty change in hepatocytes Inability to digest ingested substances, e.g. carbon, melanin and haemosiderin Misfolding of proteins, e.g. Creutzfeldt–Jakob disease (prion proteins), Alzheimer’s disease (A-peptides).

Material can also accumulate in tissues rather than cells and the most important of these is cholesterol in the intima of arteries in atheroma. This occurs through a combination of increased intake and reduced removal. For details see page 240 Fig. 8.5. Proteopathies

Some genetic abnormalities can predispose individuals to conditions related to misfolding of proteins. Often, however, the misfolding seems to be caused by an

Chapter 2: What are the common mechanisms of disease?

Mutations of LEFTA, LEFTB and NODAL can also cause situs inversus through a combination of actions, because Nodal and Lefty2 proteins affect genes specifically on the left side of the embryo and Lefty1 protein prevents signal transmission across the midline. Right– left symmetry is quite commonly abnormal in monozygotic twins, not because of a primary abnormality of genes, but because of diffusion of lateralising gene products from the left side so affecting the twin on the right. The gradients extend right to the fingers and toes and HOX gene mutations can cause problems such as syndactyly (fusion of digits). Before we leave the subject of embryogenesis, it is worth emphasising that the majority of congenital abnormalities are of unknown cause, 20% are genetic (chromosomal or monogenic), 20% are multifactorial and 10% are environmental, especially infection or alcohol related (Fig. 2.4). They can cause the following:

45

46

What are the common mechanisms of disease?

Table 2.1 Main inherited defects of metabolism Type of disorder

Disorder

Incidence

Genetics

Defective biomolecules

Amino acids

Phenylketonuria

1/10 000

AR

Phenylalanine hydroxylase

Cystinuria

1/7000

AR

SLC3A1/SLC7A9

MODY

1/400

AD (>8 genes) Various affecting beta cell function

Diabetes type 1

1/400

Polygenic >40 genes

Various affecting autoimmunity

Diabetes type 2

1/10

Polygenic around 30 genes

Various affecting insulin resistance

Lactose intolerance

1/10 (white people)

AD

Lactase

Metal transport

Haemochromatosis

1/300 (white people)

AR – four genes

HFE, transferring receptor 2, ferroportin

Lipid metabolism

Familial hypercholesterolaemia

1/500

AD

Low-density lipoprotein receptor

MCAD deficiency

1/20 000

AR

Medium chain acyl-CoA dehydrogenase

Porphyrias

For example, acute intermittent

All rare

Mostly AD

Defective enzymes

Purine metabolism

Severe combined immunodeficiency

1/70 000

AR

Adenosine deaminase affecting production of T/B cells

Sphingolipidoses

Gaucher’s disease

1/900 AR (Ashkenazim)

Part 1: Disease, health and medicine

Carbohydrates

-Glucosidase

Mucopolysaccharidoses Hurler’s disease (MPS1) 1/10 000

AR

-l-Iduronidase

Peroxisomal

Adrenoleukodystrophy

1/20 000

XR

Very-long-chain fatty acid synthase

Hepatic glycogen storage

GSD1: von Gierke’s disease

Rare

AR

Glucose 6-phosphatase

Muscular glycogen storage

GSD2: Pompe’s disease Rare

AR

Lysosomal -1,4-glucosidase

GSD, glycogen storage disease; MCAD, medium-chain acyl-CoA dehydrogenase; MODY, maturity-onset diabetes of young people; MPS, mucopolysaccharidosis.

initiator (e.g. prion disorders), or because there are large amounts of identical proteins (e.g. light chain amyloid in myeloma or hormone-related amyloid).

The proteins change their three-dimensional shape by increasing the amount of -sheet secondary structures. This can cause disease through loss of the protein’s

Genetic effects on biochemistry

Table 2.2 Some important proteopathies Condition

Protein

Alzheimer’s disease

Amyloid , Tau

Prion diseases

Prion proteins

Primary systemic amyloidosis

Monoclonal Ig light chains

Secondary systemic amyloidosis

Amyloid A (acute phase reactant)

47

Table 2.3 Risk factors for proteopathies l l l l

l

Unstable primary amino acid sequence Introduction of an initiator Changes to temperature or pH Increased protein concentration through increased production or decreased clearance Post-translational modification, e.g. hyperphosphorylation

Dialysis amyloidosis 2-microglobulin, CFTR protein

normal function (e.g. cystic fibrosis transmembrane conductance regulator protein [CFTR]) or interference with the tissue through accumulation (e.g. cardiac amyloid). The accumulation can be intracellular or extracellular. Historically, the material was called amyloid because it could be seen down a microscope and stained in a similar way to starch (Latin for starch is amylum). More

recently, there has been particular interest in proteopathies affecting the brain and causing dementia. It is appreciated that the classic amyloid plaques visible on Congo red light microscopical staining may be relatively benign, whereas invisible, non-fibrillary, misfolded oligomers are more potent causes of neuronal damage. A particularly important protein in neurodegenerative conditions is the tau protein which is the basis for the neurofibrillary tangles seen in Alzheimer’s Disease and other conditions. The tau protein is associated with microtubules and aggregates after being hyperphosphorylated.

Case study: haemochromatosis A 59-year-old man presents with a 1-month history of swelling of his ankles, urinary frequency and polydipsia. Further questioning elicits a history of reduced libido and shortness of breath on moderate exertion. On examination, the patient seems to have a greyish change to his skin, although he does not appear anaemic or cyanosed. There is mild pitting oedema of the ankles, an irregular pulse and bilateral basal crackles in the lungs. The testes show a mild degree of atrophy. Blood tests showed normal full blood count (FBC), normal urea and electrolytes (U&Es), abnormal liver function tests (LFTs) and a raised fasting glucose level. Question: Answer:

What endocrine abnormalities does this patient have? The elevated fasting blood glucose indicates diabetes mellitus. This would also explain the polyuria and polydipsia

Question: Answer:

caused by an osmotic diuresis induced by glycosuria. There is also likely to be an element of hypogonadism. There has been a change in the patient’s libido and there seems to be some degree of testicular atrophy. Further investigation would reveal that the patient has a low blood follicle-stimulating hormone (FSH), luteinising hormone (LH) and testosterone. What is the diagnosis? The patient has haemochromatosis. He has abnormal LFTs, cardiac disease, diabetes mellitus, pituitary disease (hypogonadism secondary to decreased levels of FSH and LH from the pituitary) and characteristic skin pigmentation changes. (Continued)

Chapter 2: What are the common mechanisms of disease?

Cystic fibrosis

48

What are the common mechanisms of disease?

(Continued)

Question: Answer:

Question: Answer:

What is haemochromatosis? Haemochromatosis is an autosomal recessive disease in which there is an abnormal increase in iron absorption from the gut. How could the diagnosis be confirmed? Genetic studies can be performed and are the most useful, but measurement of iron transport and storage levels in the blood is also helpful and

often quicker. Liver biopsy can reveal the presence of excess iron, as well as assessing other histology. The gene for haemochromatosis is located on chromosome 6p21.3, although alternative loci have been described for variants of the disease. There is linkage with HLA-A3 and -B14. Read more in Pathology in Clinical Practice Case 46

Basic biology Point mutation mechanism in somatic hypermutation The latest experimental evidence suggests that these mutations are deliberately introduced by using the AID (activation-induced [cytidine] deaminase) enzyme and ‘error-prone DNA polymerase’. The AID enzyme deaminates one or more cytosine bases in the DNA, so creating uracil bases, which are not normally found in DNA, and are detected by DNA mismatch-repair enzymes. The ‘error-prone DNA polymerase’ introduces the mutation as it is ‘repairing’ the DNA strand. This is why the mutations are predominantly single-base mutations rather than insertions or deletions.

Part 1: Disease, health and medicine

GENETIC EFFECTS ON THE IMMUNE SYSTEM The immune system and its role in defending the body against pathogens is covered in section X, its defence against cancer in section Y and its harmful effects in autoimmune disease in section Z. Here we will consider how the genes in B lymphocytes can produce billions of different antibody specificities (generation of diversity) and the genetic component of allergic reactions (atopy). GENERATION OF DIVERSITY

See Figure 6.17 on page 196 for gene rearrangements in heavy chain production. An immunoglobulin (Ig) molecule is composed of two identical heavy chains (H) and two identical light chains (L), each of which has constant (C), joining (J) and variable (V) regions. The heavy chains also have diversity (D) regions. The genes for light chain production are on chromosome 2 ( chains) and chromosome 22 ( chains), and there are around 40 V genes, 5 J genes and 1 C gene. For heavy chains, there are

9 C genes, 27 D genes, 44 V genes and 6 J genes on chromosome 14. This is the situation in the germline but, during B-cell maturation, there is rearrangement of the genes to bring together the required number of V, D, J and C genes to provide a specific code for that B cell. This code specifies the shape of the B-cell surface receptor and the antigen-specific antibody that it produces. You could multiply the number of V and J genes for light chains by the number of V, D and J genes for heavy chains to have an idea of the number of different B-cell specificities produced. However, this is only part of the story because there is somatic hypermutation, which is a programmed process of mutation that occurs in individual B cells after they have been stimulated by antigen to divide. The B-cell-receptor locus undergoes single-base substitutions at ‘hotspots’ called ‘hypervariable regions’ and the mutation rate is up to 1000 000 times higher than in normal dividing cells. Why? It is thought this allows the B cell with the closest fit to the new antigen to refine its progeny to be even better at responding to that antigen. So the germline cell has a variety of genes that are rearranged to produce a diverse population of B cells; these then hypermutate when stimulated to proliferate

Cell damage and cell death

Allergic reactions

Atopy is the triad of asthma, atopic eczema and allergic rhinitis, and it is both common and multifactorial. Some of the basic mechanisms are covered on page 80 (hypersensitivity reactions) and page 22 (gluten sensitivity in Chapter 1). As our knowledge of the genome increases, it is becoming clearer how many genes may be implicated, although it has long been appreciated that a family history of atopy is relevant. In particular there is a dominant autosomal allele carried by 25% of northern Europeans that can result in overproduction of IgE. The incidence is highest in offspring of affected mothers and it is thought that the gene may be imprinted (see page 37).

Read more about allergic reactions in Pathology in Clinical Practice Case 6

MOLECULAR GENETICS OF CANCER This is covered in depth in Chapter 13. It is worth mentioning here that the decades of work involved in looking at genetic changes in tumours is beginning to allow a personalised approach to therapy, increasing the likelihood of response and reducing the number of patients exposed to unpleasant side effects. In the UK, there is a trial for non-small-cell lung cancer involving 14 drugs, with patients having their tumour genetics assessed before being enrolled in the trial using the most appropriate drug. Around 21 genetic abnormalities have been identified in non-small-cell tumours but each occurs only in a small percentage of tumours, so it is important to know which patient’s tumour has which genetic abnormality. CELL DAMAGE AND CELL DEATH Let us move from the genome to the cell but using a common clinical scenario, a patient who presents with sudden onset of neurological symptoms.

Case study: shortness of breath A 23-year-old woman goes to see her general practitioner because she has had several episodes of shortness of breath in the past 2 months. The episodes last for up to 30 minutes and are sometimes associated with wheezing. She thinks that there is a connection with being exposed to dust because she had her worst episode when she was helping to clean a friend’s loft and another when she was changing the bag on her vacuum cleaner. She also thinks that cats trigger her shortness of breath and has noticed that she sometimes has a dry cough, particularly at night. She has not had a productive cough, haemoptysis or chest pain. She is otherwise fit and well, takes regular exercise and her exercise tolerance remains unchanged. Question: What is the likely diagnosis in this patient’s case? Answer: Asthma – the presentation in a young person of episodic dyspnoea that is

Question: Answer:

associated with wheezing and a dry cough and related to precipitating factors is very suggestive. Extrinsic allergic alveolitis might also be considered, but the precipitating factors are somewhat diverse (assorted dusts and cats) and the cough is not associated with the exposure. The nocturnal exacerbation of the cough is also suggestive of asthma. What is asthma? Asthma is a chronic condition in which there is bronchial hypersensitivity and hyperreactivity, leading to reversible episodes of bronchospasm that produce dyspnoea and/or wheezing.

Read more in Pathology in Clinical Practice Case 6

Chapter 2: What are the common mechanisms of disease?

by antigens. Of course, it is always possible that a new mutation could react with your own cells to cause an autoimmune (atopic) reaction or produce other cellular changes, leading to B-cell lymphoma.

49

50

What are the common mechanisms of disease?

Part 1: Disease, health and medicine

CLINICAL SCENARIO: STROKE

A 76-year-old woman is brought into accident and emergency having had a sudden onset of weakness of her right arm and leg that occurred 90 minutes earlier and is associated with difficulty in speaking. The weakness was without warning or precipitating event. The patient’s previous medical history is unremarkable. On examination, the cardiovascular, respiratory and abdominal systems are unremarkable. Examination of the limbs reveals normal power in the left arm and lower limb but grade 0 movement in the right arm and lower limb. There is hyperreflexia of the biceps, brachioradialis and triceps, knee and ankle reflexes on the right, and the right plantar response is upgoing. Tone is increased. There are no abnormal movements, fasciculations or wasting. Coordination is not assessable in the right limbs due to the loss of power but is normal in the left arm and lower limb. Widespread sensory loss for all modalities is present in the right arm and lower limb. Examination of the cranial nerves reveals a right homonymous hemianopia and weakness of the upper part of the right side of the face. The patient understands verbal instructions but has great difficulty in speaking, using only short, incomplete sentences. She seems to be frustrated by her inability to speak. She is able to sip a glass of water without problems. She is treated with appropriate thrombolytic treatment but the symptoms persist. She has had a stroke and some cells in the brain had been deprived of oxygen (hypoxia) because of an interruption to the blood supply (ischaemia). The medical term for a stroke is a cerebrovascular accident (CVA) because the cause is most commonly an obstruction (occlusion) or rupture of a vessel supplying the brain. The radiological images would show a sequence of changes from the initial insult through to limited repair. If the patient died, then there would be specific changes seen down the microscope that reflect cell damage and cell response. Read more in Pathology in Clinical Practice Case 12

Microscopical appearances

The first phase is generalised oedema as the damaged cell membranes and altered permeability of the blood brain barrier affect both intracellular and extracellular fluid distribution. This produces acute cellular oedema or ‘cloudy swelling’. At the lightmicroscopy level, this appears as enlargement of the cell and a pale granular look to the cytoplasm. Vesicles may also appear due to the distension of the endoplasmic reticulum. This picture of cellular oedema is also referred to as hydropic or vacuolar degeneration. Within 12 hours of irreversible injury, the microscopical changes in neurons involve shrinkage of the cell body with eosinophilia (red staining on the haematoxylin and eosin stain) of the cytoplasm, pyknosis and disintegration of the nuclei (karyorrhexis), disappearance of the nucleolus and finally complete dissolution of the nuclei (karyolysis), see fig 2.11 page 58. If the axon is damaged, there is a microscopically visible spheroid, caused by swelling and disruption of axonal transport systems. The brain has nerve cells (neurons) and supporting cells (glial cells). The neurons are most susceptible to ischaemia. The glial cells may be so badly damaged that they also die, or they may survive and work alongside other glial cells invading the damaged area, removing dead cells and attempting limited repair. This is called gliosis. The astrocytes increase in number (hyperplasia) and size (hypertrophy), and have some similarities to fibroblasts in other tissues but don’t produce significant collagen. The microglia are phagocytic and cluster around damaged cell bodies and axons or areas of haemorrhage. Our unfortunate patient has suffered an infarction, which is the term used when there is a localised area of irreversible ischaemic tissue damage, most commonly caused by a reduced blood supply. The brain does not have any glycogen or fat storage and so is dependent on a steady supply of glucose and oxygen via the blood. In the adult, the brain weighs 2% of the body weight but consumes 20% of the body’s oxygen, and so is the most vulnerable organ to hypoxia. Neurons (nerve cells) are also so specialised, with their complex axonal and dendritic structures and myelin coatings, that they are unable to divide to repair damage. This is why strokes are

Cell damage and cell death

51

Radiology of a stroke Series of images showing the evolution of a stroke are shown here in Fig. 2.5.

d

b

c

Figure 2.5 (a) The acute CT scan when the patient is brought into accident and emergency and is confused and has an impaired level of consciousness. The image shows that the patient is moving (hence the slight blurring). In the left hemisphere one notices a loss of the grey–white differentiation of the cerebral cortex and the subcortical white matter (abnormal side – blue arrow, normal side – red arrow). This indicates oedema as the blood–brain barrier breaks down and the cell membranes let fluid in. (b) MRI within the first week in hospital: this is a diffusion-weighted image (DWI) that shows the distribution of trapped water in the brain – the water filling the dead cells – cytotoxic oedema (blue arrow). The infarct is in the territory of the left middle cerebral artery. (c) The CT scan 3 months down the line. The infarct is on the way to becoming cystic and therefore blacker (blue arrow). There is also a loss of volume and the lateral ventricles dilate, filling the space previously taken up by brain tissue. (d) Another patient 5 years after his original stroke, which shows a large cystic cavity replacing much of the left hemisphere (blue arrows) after a large middle cerebral artery infarct.

so devastating. If symptoms persist beyond the first few hours, it indicates irreversible cell damage with no prospect of structural repair, but sometimes there is some compensatory functional recovery through neuronal plasticity and opening up of alternative neuronal pathways. So what is happening at a cellular level?

BIOCHEMICAL CHANGES IN THE CELLS

There are two important questions to consider: 1 What are the biochemical changes that occur in an injured cell? 2 What distinguishes reversible from irreversible injury?

Chapter 2: What are the common mechanisms of disease?

a

52

What are the common mechanisms of disease?

There are four sites within the cell that are of paramount importance in cell damage and death: l l l l

Mitochondria Plasma membrane Lysosomes Cytoskeleton. The key mechanisms are:

l l l l l

depletion of ATP increased intracellular calcium increase in free radicals disruption of membranes DNA and protein damage.

Part 1: Disease, health and medicine

Depletion of ATP

The first effect of ischaemia is to reduce the production of adenosine triphosphate (ATP) by the mitochondrial oxidative phosphorylation system (Fig. 2.6). If the production of energy slows down or stops, then the cells cannot function. In tissues that have glycogen, the ischaemic cells will switch over to anaerobic metabolism to produce ATP, but this has the unwanted effect of producing lactic acid so that the pH drops in the cell, potentially creating more damage by affecting cellular enzymes and clumping the nuclear chromatin, so-called pyknosis. Ischaemia also has profound effects on the plasma membranes and the ionic channels situated within the membranes. You will recall that these are vital in maintaining the normal ionic gradients across the cell membranes, with sodium and calcium at low concentrations inside the cells and potassium lower in the extracellular space. These concentrations are maintained by pumps that are energy dependent, so it is not difficult to see that the loss of oxidative phosphorylation and any direct damage to the membranes will disrupt the function of these pumps. So what is the effect? First, the failure of the pumps will result in leakage of sodium into the cells and potassium out of the cells. Sodium has a larger hydration shell than potassium, so more water moves in association with sodium ions than exits with the potassium. Additional water enters because the acidosis and raised intracellular concentrations of high-molecular-mass phosphates will increase the osmotic pressure inside the cell. The result is acute swelling of the cell due to cellular oedema. The endoplasmic

reticulum (ER) also swells, the ribosomes detach from the ER, the mitochondria become swollen and blebs begin to appear on the cell surface. This last phenomenon is intriguing because the changes in cell shape and surface blebbing imply alterations in the cytoskeleton of the cell. The changes in the microfilaments of the cytoskeleton are believed to be due to the increased concentration of calcium, which also results from the failure of the membrane pumps. Calcium is a very important ion in cell death and we shall see why in a minute. You might find it difficult to believe but all the changes described so far are reversible! If the oxygen supply is restored, the cells still have the capacity to return to the normal state and the neuron will transmit again. So what are the changes that finally tip the cell beyond the point of no return? The morphological hallmarks are severe disruption of the membranes affecting, in particular, the mitochondria, plasma membranes and lysosomes. Calcium is thought to play a central role in this final progression to irreversible cell death. Increased intracellular calcium

In the normal cell, the calcium concentration is tightly controlled by the calcium pump in the cell membrane. Ischaemia disrupts oxidative phosphorylation, so affecting the energy-dependent calcium pump, leading to a rapid influx of calcium and saturation of the calcium-regulating proteins. Damage to cell organelles also leads to release of intracellular calcium. The high levels of calcium are toxic to the cell, leading to changes in the cytoskeleton, cell surface blebbing, and damage to the mitochondria, lysosomal membranes and cell membranes. The release of enzymes from the ruptured lysosomes also contributes to the final destruction of the cellular components. Increase in free radicals

In addition, there is another important pathway common in many types of cell damage that generates reactive oxygen species (ROS). This is called oxidative stress and involves free radicals derived from oxygen. As you will recall, free radicals have a single unpaired electron in their outer orbit, which makes them unstable and strongly reactive to cellular lipids, proteins and nucleic acids. ROS are produced normally in mitochondrial respiration but don’t cause damage because they are removed through the action of superoxide dismutases,

Cell damage and cell death

53

Necrosis

Insult (e.g. ischaemia)





Anaerobic metabolism

Failure of Na+/K+ pump

↑Lactic acid and inorganic phosphate

↑Na+ (with H2O) Na+ in cell

ATP)



Na+ H2O

Swelling of cell and organelles

Reversible



Decreased energy production ( Oxidative phosphorylation in mitochondria

Failure Ca2+ pump K+ out

↑Intracellular Ca2+

Rupture of organelles: releases Ca2+

Phospholipase activation

Protease activation

Endonuclease activation

Membrane disruption

Proteolysis of cytoskeleton

Hydrolysis of DNA

Ca2+

Lysosmal contents cause autolysis

Plasma membrane plebs form: loss of structures e.g. microvilli

Pyknosis, karyorrhexis and karyolysis

Cellular disintegration: inflammatory response to cell debris

Figure 2.6 Necrosis is the culmination of a series of events, the first phases of which are reversible. The failure of the calcium pump marks the onset of irreversible change. The cell breaks down and its contents are exposed to the extracellular space, with leakage of lysosomal membranes and other cellular constituents. The cells appear swollen and degenerate under the microscope.

Chapter 2: What are the common mechanisms of disease?

Irreversible

54

What are the common mechanisms of disease?

glutathione peroxidases, catalases and antioxidants, such as vitamins A, C and E. ROS are also essential, in low concentration, for many cell signalling pathways. They are produced in significant amounts by neutrophils and macrophages for killing harmful microbes, but then they are safely packaged in lysosomes that will fuse with phagosomes. (Phagosomes are membrane-bound ingested microbes or other pathogenic substances.) Free radical production is pathologically increased by ionising radiation, chemicals (e.g. carbon tetrachloride and paracetamol) and oxygen toxicity, and in cellular ageing and ischaemia–reperfusion damage. ROS damage the DNA by producing single-strand breaks that contribute to cell death, ageing and cancer. They cause cross-linking or fragmentation of proteins so that the proteins cannot function normally. They attack polyunsaturated lipids in membranes, which not only damages the membrane but also generates more peroxides, so amplifying the process.

Part 1: Disease, health and medicine

Disruption of membranes

Membranes can be damaged by physical and chemical agents, microbial toxins and components of the complement cascade (see page 137), as well as ischaemia and ROS. These generally act through some combination of increased degradation of membrane phospholipids, probably by calcium-induced activation of endogenous phospholipases, or decreased phospholipid synthesis because of a reduction in energy-dependent cell activities. The impact of phospholipid breakdown products can be devastating on the cell because they can insert into the membrane lipid bilayer, altering permeability and electrophysiological behaviour (obviously important in our patient with ischaemic nerve cells). The cytoskeleton connects the plasma membrane to the cell interior and is important in signalling, motility and internal structure. If the skeleton is damaged by proteases then the membrane function deteriorates.

Table 2.4 Features of reversible and irreversible cell damage Reversible

Irreversible

Cell swelling

Release of lysosomal enzymes

Mitochondrial swelling

Protein digestion

Endoplasmic reticulum swelling

Loss of basophilia

Detachment of ribosomes

Membrane disruption

‘Myelin’ figures

Leakage of cell enzymes and proteins

Loss of microvilli

Nuclear changes: pyknosis, karyorrhexis, karyolysis

Surface blebs Clumping of nuclear chromatin Lipid deposition

DNA and protein damage

The final item on our list of mechanisms is DNA and protein damage. These commonly cause a specific type of cell death called apoptosis, which we consider on page 62.

Figure 2.7 Reversible and irreversible changes.

CLINICAL RELEVANCE OF CELL CHANGES So much for science! Do these biochemical and microscopical changes help us to understand any of the clinical manifestations of our patient with cerebral ischaemia? The nerve cell’s ability to conduct electrical signals is dependent on the integrity of its plasma membrane and the pumps that maintain the ionic gradients. For the transmission of signals from one nerve to another, it requires the production and controlled release of transmitter substances, followed by their reuptake or degradation. For the movement of substances from their site of synthesis in the cell body to the ends of the axon, it needs an intact cytoskeleton, especially the microtubules. You can easily appreciate that the changes that occur in an ischaemic cell totally disrupt these functions and nerve transmission ceases. The clinical impact depends on which area of the brain is affected because many nerve pathways are quite discrete and the blood supply to the brain has distinct territories, so the effects can be quite localised but still very disabling. Books on neuroanatomy and clinical neurology will provide the detail but, in the specific case of our patient, the CVA has occurred in the left cerebral hemisphere and the combination of neurological defects indicates damage to brain regions that are all located in the territory of the left middle cerebral artery. The left cerebral hemisphere deals with motor and sensory function for the right side of the body and also from the right visual field. The localisation of speech function is slightly more complex: >99% of right-handed people and around 50% of left-handed people have their speech centres in the left cerebral hemisphere. Specialised cells are affected differently by ischaemia so it is worth looking at the clinical effects of damage on liver cells and cardiac muscle cells. The hepatocyte is principally a chemical factory synthesising and degrading molecules through many complex biochemical pathways. This requires a range of enzymes and any severe damage to the cell that disrupts cell membranes will allow leakage of those enzymes. This provides the basis of some of our most commonly used laboratory blood tests for screening and assessing a patient. So-called LFTs look at the blood levels of key enzymes that are predominantly or exclusively found in hepatocytes. Their level in the blood will rise if there is acute or ongoing liver cell damage. The same is true of

55

muscle cells that leak enzymes (such as creatine kinase [CK], aspartate aminotransferase [AST] and lactate dehydrogenase [LDH]) and troponin molecules, which can be used to help confirm a clinical diagnosis of myocardial infarction. The brain and the heart are the two major organs affected by acute or chronic ischaemia. Just as in the brain, the heart is dependent on electrical conduction to keep it beating in a coordinated way. If the membrane potential is altered, it can result in cardiac arrhythmias, either because the pacemaker is affected or the conduction pathways are damaged, or because myocytes are not responding normally. This happens extremely quickly and can result in a sudden cardiac arrest and death. At that point, the cytoskeleton with all the mitochondria and myofilaments is still intact and so the damage is reversible. Later the proteins degrade, the membranes rupture and inflammatory cells, especially macrophages, infiltrate the damaged area to remove the debris and lay down a scar (see healing and repair, page 147). That section of the heart can no longer contract and so the ventricular pump is weaker and the patient may be breathless, have peripheral oedema or limited exercise tolerance – the features of chronic heart failure. We have talked about reversible and irreversible cell injury; however, there is a situation where a cell that has suffered only minor damage can suffer a further fatal insult; this is ischaemia–reperfusion injury and it is clinically important in cerebral and cardiac ischaemia. An increase in damage caused by ROS is thought to be a major mechanism and this could result from the following: l

l

l

Increased production of ROS by damaged mitochondria Increased action of oxidases in endothelial cells and infiltrating leukocytes Reduced cellular antioxidant mechanisms.

In addition, reperfusion will bring inflammatory cells and mediators, such as components of the complement system, which can cause damage. Finally we must consider the clinical effects of alcohol on liver cells because it is such a common cause of disease. The earliest and most reversible damage is fatty change. This refers to an excess of intracellular lipid, which appears as vacuoles of varying size

Chapter 2: What are the common mechanisms of disease?

Clinical relevance of cell changes

56

What are the common mechanisms of disease?

within the cytoplasm. Similar to cellular oedema, it is entirely reversible and is a non-specific reaction to a variety of insults. Sometimes it is present adjacent to tissues that are more severely damaged or show frank evidence of necrosis. Fatty change can occur in any organ but is most frequent in the liver, which is not surprising because the liver is the major site of lipid metabolism. Figure 2.8 illustrates the main causes and effects of fatty change in the liver. Put simply, adipose tissue releases fat as free fatty acids which enter the hepatocytes, where they are converted to triacylglycerols and, to a smaller extent, cholesterol. Triacylglycerols are complexed with apoproteins to form lipoproteins, which are then secreted into the blood. Changes will lead to lipid accumulation within the hepatocytes. The liver appears enlarged and pale (Fig. 2.9) and fat globules are seen microscopically (Fig. 2.10). Alcohol is a hepatotoxin that has wide-ranging effects on fatty acid metabolism. It increases peripheral tissue release of fatty acids so that more are delivered to the

Part 1: Disease, health and medicine

Liver is overwhelmed with free fatty acids, as in alcohol excess, diabetes mellitus, obesity or genetic hyperlipidaemia

liver and, within the liver, it is implicated in increasing fatty acid synthesis, decreasing the utilisation of triacylglycerols, decreasing fatty acid oxidation and blocking lipoprotein excretion. Thus, it interferes with a variety of biochemical pathways. Gross examination of the organs affected by fatty change will show that they are enlarged and yellow, and tend to be greasy to the touch. Microscopically, the characteristic finding is of vacuoles within the cytoplasm. These may begin as small vacuoles but, if the fatty accumulation continues, the vacuoles will coalesce to form larger vacuoles or ‘fatty cysts’. This type of change is entirely reversible if the insult is withdrawn. A binge in the medical school bar on a Friday night may produce fatty change but this will disappear if one is able to abstain for a few days afterwards! Chronic abuse of alcohol may produce sufficient fatty change to interfere with the normal function of the hepatocytes and, in the long term, excessive alcohol consumption may lead to cell death, scarring and cirrhosis, which is not reversible.

Metabolism of fatty acids and triglycerides is inhibited by competition with drugs such as alcohol, or interfered with, e.g. hepatitis C virus infection (some strains)

Excretion is prevented, e.g. due to a lack of protein (chronic alcoholism, protein calorie malnutrition) or paralysis of the secretory apparatus, e.g. drugs of various types

Fatty liver Fat accumulation in hepatocytes may disrupt the normal blood flow in the hepatic sinusoids

Oxidation of excessive fat stores in hepatocytes may generate free-radical formation and cause local liver damage, stimulating fibrosis Fat accumulation may cause hepatocytes to become resistant to insulin

Fat may activate angiotensin and lead to hypertension

Figure 2.8 Fatty liver: main causes of fat accumulation and their deleterious effects.

Clinical relevance of cell changes

57

Key facts Causes of fatty change in the liver Diabetes, metabolic syndrome and obesity Alcohol Protein malnutrition Acute fatty liver of pregnancy Congestive cardiac failure Ischaemia/anaemia

Figure 2.10 Photomicrograph of liver showing fatty change (vacuoles arrowed).

Carbon tetrachloride

The other two major causes of fatty change in the liver are malnutrition and diabetes. Malnutrition in particular affects two steps in fat metabolism. It increases the release of fatty acids from peripheral tissue and protein deficiency reduces the cell’s ability to combine triacylglycerols with apoprotein. Disordered carbohydrate metabolism in uncontrolled diabetes leads to excessive peripheral release of fatty acids. Metabolic syndrome has many similarities to type 2 diabetes and can be considered a pre-diabetic state. It has similar disturbances of carbohydrate metabolism, insulin resistance and raised levels of triacylglycerols, so it is also a cause of fatty change in the liver.

Case study: fatty liver Malcolm, a 48-year-old man, was hit by a car as he weaved his way home by bicycle (he lost his licence for drink driving a year ago). His injuries were mild, but he had hit his head so he was kept in for observation. After a day, the nurses noted that he was plucking agitatedly at his sheets and cringing from ‘vultures’ circling him. He was also sweating, with tachycardia and rapid breathing. The doctor diagnosed delirium tremens caused by acute alcohol withdrawal. Malcolm recovered over the next 2 days and a psychiatric referral was made, but Malcolm denied an alcohol problem and discharged himself.

One year later, Malcolm was rushed into hospital in a weak, jaundiced and semi-comatose state. His breath was foul smelling (‘foetor hepaticus’). His sclerae were deep yellow and his liver was enlarged and tender. Investigations showed that he was in fulminant liver failure. Question: Answer:

Discuss the consequences of a fatty liver. A fatty liver can impair blood flow in the hepatic sinusoids due to mechanical obstruction. There can be oxidation of fat causing free radical formation, which stimulates inflammation and fibrosis. Hepatocyte insulin resistance (Continued)

Chapter 2: What are the common mechanisms of disease?

Figure 2.9 Normal and fatty liver (pale).

Drugs: steroids, methotrexate, intravenous tetracyclines

58

What are the common mechanisms of disease?

(Continued)

Question: Answer:

of toxic damage), often with fibrosis. Patients are at risk of acute liver failure, advanced fibroses or cirrhosis. Acute liver failure has massive hepatocyte necrosis due to a toxic insult such as alcohol, paracetamol overdose or acute viral hepatitis. The mortality rate in acute liver failure due to steatohepatitis is around 40%.

leads to type 2 diabetes mellitus. Fat can also activate angiotensin, leading to hypertension. Compare fatty liver disease, steatohepatitis and acute liver failure. Fatty liver shows fat with minimal inflammation and no fibrosis. This is reversible but about 50% of patients later develop fibrosis. Steatohepatitis has fatty change, inflammation and ballooning of hepatocytes (a feature

NECROSIS There comes a stage at which reversible damage becomes irreversible and cell damage becomes cell death (Fig. 2.11 and see Fig. 2.6). The final events follow one of two distinct processes: ‘necrosis’ or ‘apoptosis’ (see page 62). Necrosis is cell death due to lethal injury. Unlike apoptosis, cell death is not an energy-dependent active process but is a consequence of sudden changes in the microenvironment abolishing cell function. The changes

Read more in Pathology in Clinical Practice Case 19

seen in the tissues are a consequence of denaturation of proteins and release of digestive enzymes that destroy the tissue. The principal types are: l l l

coagulative colliquative or liquefactive (Table 2.5) caseous.

COAGULATIVE NECROSIS

If you consider Fig. 2.12, (a) is of a normal kidney with normal glomeruli and tubules, whereas (b) is from a kidney that has suffered an ischaemic insult and is showing coagulative necrosis. Spot the difference?

Part 1: Disease, health and medicine

Normal

Table 2.5 Key facts about coagulative and liquefactive necrosis Pyknosis

Coagulative

Liquefactive

Mechanism Karyolysis

Severe ischaemia destroying proteolytic enzymes

Strong proteolytic enzyme action destroying tissue

Appearance Karyorrhexis

Figure 2.11 Cell death involves nuclear changes (left) and loss of dendritic and axonal structures in nerve cells (right).

Initial preservation of cell outlines and tissue architecture

Loss of cell outline Tissue becomes cystic or fluid

Occurrence Kidney, heart

Brain, bacterial infections

Necrosis

a

c

components; the resulting debris will be removed by phagocytic macrophages. So the appearance of an area of coagulative necrosis will change with time and the final result will be an amorphous fibrous scar. COLLIQUATIVE OR LIQUEFACTIVE NECROSIS

The hallmark of this type of necrosis is the release of powerful hydrolytic enzymes that degrade cellular components and extracellular material to produce a

b

Figure 2.12 (a) Normal kidney; (b) coagulative necrosis; and (c) photomicrograph of lymph node with tuberculous granuloma and caseous necrosis (arrowed).

Chapter 2: What are the common mechanisms of disease?

The second picture (Fig. 2.12b) is essentially the ghost outline of the first! The difference between the two is that the damaged kidney shows loss of nuclei from the cells and the cytoplasm stains a slightly darker pink. This pattern of necrosis is the most common type and occurs in many solid organs such as the heart and kidney (Fig. 2.13a). The tissue, of course, doesn’t remain in that state forever. Polymorphs start to move in within 24 hours of infarction, and release enzymes that digest the cellular

59

What are the common mechanisms of disease?

60

proteinaceous soup. Characteristically, it occurs in the brain where it produces a cystic cavity containing fluid and necrotic debris (Fig. 2.13b). Liquefaction may also be encountered in tissues when there is a superadded bacterial infection. Then, enzymes are released from both the bacteria and the inflammatory cells that have been recruited to fight the infection. a

b

CASEOUS NECROSIS

Caseous necrosis typically occurs in tuberculosis and is so called because of a resemblance to soft crumbly cheese! The necrotic area is not quite liquid but nor is the outline of the tissue retained as in coagulative necrosis. On microscopical sections stained with haematoxylin and eosin (H&E) (see Fig. 2.12c), the necrotic area appears homogeneously pink (eosinophilic) with a surrounding inflammatory response involving multinucleate giant cells, macrophages and lymphocytes (see granulomatous inflammation, page 165). It is believed that lipopolysaccharides in the capsules of the mycobacteria may be responsible for this peculiar reaction but the mechanism is unclear. OTHER TYPES OF NECROSIS

Although these are the main types of necrosis, for completeness, we should briefly mention four others. These are fat necrosis, gangrene, fibrinoid necrosis and autolysis.

Part 1: Disease, health and medicine

Fat necrosis

c Figure 2.13 (a) Kidney, showing wedge-shaped cortical infarct; (b) brain with cerebral infarction; and (c) coronal CT after intravenous contrast showing massive splenomegaly with a nonenhancing infarct at the tip of the spleen (red arrows).

This type of necrosis is peculiar to fatty tissue and is most commonly encountered in the breast following trauma and within the peritoneal fat due to pancreatitis. Within the breast, trauma may lead to the rupture of adipocytes and the release of fatty acids. This will elicit an inflammatory response and the area will become firm due to scarring, forming a palpable lump. Clinically, the lump may be mistaken for a carcinoma and excision and microscopical examination may be required to determine the diagnosis. In pancreatitis, damage to the pancreatic acini results in the release of proteolytic and lipolytic enzymes, which denature fat cells in the peritoneum and lead to an inflammatory reaction. Calcium is also deposited in the tissues in combination with fatty acids to form calcium soaps. This is a form of dystrophic calcification

Necrosis

Gangrene

This does not represent a distinctive type of necrosis but is a term used in clinical practice to describe black, dead tissue. It is most commonly seen in the lower limb in patients with severe atherosclerosis, which often causes irreversible ischaemic damage to the most peripheral tissues in the body. If the pattern of necrosis is mainly of the coagulative type, it is referred to as dry gangrene, whereas the presence of infection with Gram-negative bacteria converts it into a liquefactive type of necrosis, when it is called wet gangrene. It will be apparent from the preceding discussion that the type of necrosis encountered depends on a number of different factors, including the type of tissue involved and the nature of the offending agent. Gas gangrene is the disastrous complication that follows infection of tissue by the Gram-positive organism Clostridium perfringens, found in soil. The bacterium releases a toxin and also produces gas, which can be felt as crepitation when the affected area is pressed. Fibrinoid necrosis or fibrinoid change refers to the microscopical appearance seen when an area loses its normal structure and resembles fibrin. It does not have any distinctive gross appearance. Autolytic change is completely different from the others because it refers to cell death occurring after the person has died. Obviously, the heart stops pumping and all the tissues become irreversibly ischaemic. Enzymes leaking from the cells digest adjacent structures, but there is no inflammatory response because the inflammatory system is dead!

such as heart valves, it may affect function (Fig. 2.14). Dystrophic calcification may be useful, as in the microcalcification observed on mammograms; this can alert the radiologist to an early breast cancer (Figs 2.15 and 2.16). Dystrophic should be distinguished from metastatic calcification. This is linked to abnormally high serum calcium levels, due perhaps to hyperparathyroidism (increased parathyroid hormone secretion mobilises calcium from the bones), or excess vitamin D ingestion (increased calcium absorption from the gut). Metastatic carcinoma within bones may liberate

Figure 2.14 Dystrophic calcification in damaged aortic valve, producing stenosis.

Calcification in necrotic tissue

Necrotic tissue may become calcified. When a cell undergoes necrosis, large amounts of calcium enter the cell and this combines with phosphates within the mitochondria to produce hydroxyapatite crystals. Extracellular calcification can also occur, the crystals forming in membrane-bound vesicles derived from degenerating cells. This is the initiation phase. There is then propagation of crystal formation, depending on the local concentration of calcium and phosphate and the amount of collagen present (this enhances calcification). Usually such dystrophic calcification is not a problem to the body, but if it affects an important site,

Figure 2.15 Radiograph of a resected breast specimen showing a circumscribed abnormality, which is radio-opaque because of dystrophic calcification.

Chapter 2: What are the common mechanisms of disease?

and we consider calcification again in the section on tissue response to necrosis.

61

62

What are the common mechanisms of disease?

Figure 2.16 Dystrophic calcification in a necrotic tumour.

calcium and result in metastatic calcification, the term ‘metastatic’ referring to the widespread and scattered nature of the lesions encountered rather than inferring a similar mechanism. APOPTOSIS AND AUTOPHAGY

Part 1: Disease, health and medicine

Programmed cell death is a planned and coordinated mechanism to achieve the death of individual cells and is a process requiring energy input from the targeted cell. The packaged, membrane-bound bundles that are produced are quickly tidied away by nearby cells. On the other hand, necrosis is the result of accidental damage of various types, invariably involves groups of cells, and usually generates an inflammatory reaction, tissue damage and often scarring (Fig. 2.17).

The word ‘apoptosis’ is derived from Greek and was originally used to describe the falling of individual leaves from a tree. In pathology (Fig. 2.18), it is a specific type of cell death that involves single cells or small groups of cells in a tissue where the other cells are functioning normally. Programmed cell death has an important role in all animals for controlling cell numbers, facilitating tissue modelling and removing damaged cells. It results in the death of the cell. Autophagy, literally self-eating, need not kill the cell but is an essential part of normal cell homeostasis that allows recycling of cell constituents and a source of energy when the cell is starving. Sometimes it can produce cell death and the main differences between autophagic and apoptotic cell death is that apoptosis requires the involvement of a phagocyte to tidy up the cell packages resulting from apoptosis: the ‘come-and-eat-me’ component. Autophagy is an adaptation for survival when there are limited nutrients, a means of removing damaged organelles or misfolded proteins, and an important route for destroying intracellular pathogens, such as Mycobacterium tuberculosis. Failure of autophagy is likely to lead to accumulation of damaged cells and the clinical effects of ageing. Autophagic processes are often identified in areas of apoptotic cell death, and it is not clear whether there is a form of autophagy that deliberately results in cell death or whether the autophagic processes are attempting to repair damaged cells and avoid death. Table 2.6 compares apoptosis with necrosis and autophagy.

Why me?

Figure 2.17 Necrosis is a messy business, whereas programmed cell death (apoptosis) is an ordered event.

Apoptosis and autophagy

63

S H RE D D E R

Apoptosis

Necrosis

Autophagy

CAN CELL DEATH BE USEFUL?

The importance of programmed cell death is evident from the earliest stages of the embryo through to the involutional changes of the menopause (Fig. 2.19). It is also a part of daily life because it is estimated that an adult loses more than 50 billion cells each day through apoptosis. Let us consider the production of a limb with its five digits. To achieve this, tissue growth has to occur by cell division but it is also necessary to produce interdigital cell death. It is either that or ending up as a duck! This type of cell death is genetically controlled. Similarly, there are the stages of metamorphosis that take place to turn a tadpole into a frog. Metamorphosis requires not only mitotic activity and tissue growth, but also a large amount of programmed cell death. When a tadpole turns into a frog, the most obvious change is that limbs are formed and the tail is resorbed. During the process of resorption, there is an increase in thyroxine, which appears to lead to the activation of collagenases and, hence, destruction of the tail. Here we have an example of how programmed cell death may depend on the production of a hormone with activation of protein enzyme systems to assist the process.

The development of the nervous system is also dependent on programmed cell death. There is an excess of neurons and only those that produce the correct synaptic connections with their target cells survive. The rest, up to 50% of the neurons, die as a result of apoptosis. In gene knock-out mice unable to undergo apoptosis, the cells can be lost via autophagic cell death. The endometrium is a hormone-dependent tissue that undergoes cyclical changes during the reproductive period as well as involutional changes after the menopause (Fig. 2.20). The oestrogens secreted by the ovary in the early part of the menstrual cycle induce endometrial proliferation and, if pregnancy does not occur, there is programmed cell destruction that results in menstrual shedding. If pregnancy occurs, then there is hyperplasia of the breast in preparation for lactation, which will be followed by physiological atrophy involving apoptosis after weaning. This atrophy not only is due to cell loss but also results from a reduction in cell size and loss of extracellular material. After the menopause, the withdrawal of the hormonal influence results in involution of the uterus and ovaries. Apoptosis plays an important role in the immune system. It is necessary for the selection of specific subpopulations of both T and B lymphocytes, and is

Chapter 2: What are the common mechanisms of disease?

Figure 2.18 The term ‘apoptosis’ derives from the Greek for the falling of a single leaf from a tree. Extrapolating the analogy, in this diagram necrosis involves the death of many leaves at once, whereas in the process of autophagy there is death of leaves but their constituents are recycled.

64

What are the common mechanisms of disease?

Table 2.6 Comparison of necrosis, apoptosis and autophagy Necrosis

Apoptosis

Autophagy

Results in cell death once past the reversible point

Results in cell death from point of initiation

Mostly assists cell survival but sometimes results in cell death

Caused by energy deprivation

Requires energy for process

Requires energy but recycles cell constituents within individual cells so improving energy levels

Caused by injurious agent or event

Response to oxidative stress, hormones, growth factors and cytokines

Occurs normally in exercise, calorie restriction and defence against intracellular pathogens

Haphazard destruction of cell with release of contents including enzymes from ruptured lysosomes

Orderly packaging of organelles and nuclear fragments in membrane-bound vesicles

Cell integrity generally maintained

Cellular debris stimulates inflammatory response

New molecules expressed on membranes stimulate phagocytosis without inflammatory response

Cell integrity generally maintained

Normal process for cell turnover

Normal muscle homeostasis in exercise

Depletion of CD4 cells in HIV

Response to calorie restriction

Virally induced tissue damage

Repair of internal cell structures by degradation of damaged organelles and proteins

Clinical relevance Detrimental effect due to loss of cell function exacerbated by inflammatory response and fibrosis, e.g. myocardial and cerebral infarction

Cancer pathobiology and treatment

Elimination of intracellular Mycobacterium tuberculosis

Part 1: Disease, health and medicine

Cancer pathobiology and treatment

Figure 2.19 Programmed cell death (apoptosis) at the embryonic stage allows limbs to develop appropriately.

Figure 2.20 Programmed cell death in the endometrial glands (arrows show apoptotic debris).

Apoptosis and autophagy

Apoptosis, causes and mechanisms

You’ll appreciate from the section above that apoptosis is crucial in normal physiological processes and will occur in normal healthy cells, for example: l l

l

l l

Normal cell turnover Normal tissue remodelling in embryogenesis or repair Normal hormone-dependent involution of tissues, e.g. endometrium Removal of self-reactive lymphocytes Removal of immune cells as the inflammatory response ends.

It is equally important in pathological circumstances where damage limitation is needed via the removal of affected cells without inducing inflammation, e.g.: l

l

l

Virally infected cells (either stimulated by the virus, e.g. HIV or the T-cell response, e.g. viral hepatitis) Radiation, cytotoxic drugs or oxidative stress inducing the DNA damage Misfolded proteins inducing ER stress.

The final common pathway for apoptosis involves enzymes called caspases, which are activated through an intrinsic (mitochondrial) or extrinsic (death receptor) pathway or through increases in cell calcium. The mitochondrial pathway operates by altering the balance of pro-apoptotic and anti-apoptotic members of the BCL-2 family, which affects the permeability of the mitochondrion and the release of factors that control the caspase cascade. This occurs particularly when cells are deprived of growth factors or have damaged DNA or accumulated proteins. The death receptor pathway operates through members of the TNF (tumour necrosis factor) receptor family and the Fas (CD95) receptor. This is the most important mechanism for removing self-reactive lymphocytes and T-cell-mediated, virally infected cell death (Fig. 2.21). It is increasingly recognised that accumulating misfolded proteins is an important mechanism in many diseases and ageing. It is called ‘ER stress’ because the endoplasmic reticulum is the normal site where

newly synthesised proteins undergo folding with the assistance of chaperone molecules. The ER becomes overwhelmed in conditions where misfolding of proteins increases and this results in either apoptosis or cell adaptation (the ‘unfolded protein response’) so that protein production is reduced and chaperone synthesis increased. Conditions that increase misfolded proteins include: l l l l

Genetic mutation in proteins or chaperones Viral infections Chemical insults Metabolic alteration that depletes energy stores.

You can actually see the changes of apoptosis down a light or electron microscope and, in contrast to necrotic cells which swell, apoptotic cells shrink. They also lose their contact with neighbouring cells early on. After 1–2 hours the nuclear chromatin condenses on the nuclear membrane and then the membrane ‘packages’ these small aggregates of nuclear material to give membrane-bound nuclear fragments. The cytoplasm shrinks and the cell’s organelles also become parcelled into membrane-bound vesicles. These are called apoptotic bodies and they contain morphologically intact mitochondria, lysosomes, ribosomes, etc. Finally, these apoptotic bodies are phagocytosed by neighbouring cells or by macrophages and the contents degraded in secondary lysosomes. They are marked ‘for disposal’ by specific markers, such as phosphatidylserine or thrombospondin. Thus no messy acute inflammatory process is initiated. However, this process requires the expenditure of energy, as with any good garbage disposal system! Defects in this apoptotic corpse clearance might be associated with the development of autoimmune and inflammatory conditions. The uptake of apoptotic fragments stimulates release of anti-inflammatory mediators and can inhibit secretion of pro-inflammatory mediators (i.e. the opposite to the effect of uptake of necrotic cells). Autophagy

Autophagy is mostly concerned with cellular housekeeping. It takes damaged or redundant proteins, organelles, parts of organelles or areas of the nucleus, and digests them into their essential constituents through the action of lysosomal enzymes. Thus, proteins

Chapter 2: What are the common mechanisms of disease?

also important in the destruction of target cells by cytotoxic T cells.

65

66

What are the common mechanisms of disease?

Mitochondrial pathway Loss or withdrawal of growth factors or hormones

Death receptor pathway

DNA damage e.g. radiation

Misfolded proteins

Mitochondrial pathway sensors ANTI Bcl-2 Bcl-xL

Apoptotic balance H’ H’

PRO Bax Bak CytoC H’

H’

H’ H’

H’

H’ H’

H’

Dimerise and insert into membrane creating pores Tc and NK cells

H’

H’ H’ H’ H’

H’

Fas L expression

Cytochrome c

95

CD

Caspase - Cascade

Fas ligand binding TNF ligand binding

Endonuclease activation, DNA cleavage and condensation on the nuclear membrane: then fragmentation

Nucleus DNA fragments

Part 1: Disease, health and medicine

Orderly catabolism and condensation of cytoskeleton Dead

Macrophage

Dead

Disposal Phagocytosis by macrophages and adjacent epithelial cells

Apoptotic bodies form and bud off from cell membrane. A marker moves from inside to the outside of the membrane to signify that the apoptotic body is to be phagocytosed

Figure 2.21 Programmed cell death (apoptosis) is an ordered and energy-requiring process that can be stimulated by diverse agents. Once initiated, the process is irreversible. There is no associated inflammation. It can be thought of in three stages: initiation, execution and disposal. Activation of the caspase cascade results in the breakdown of nuclear and cytoplasmic components which are packaged in membranes for disposal. Caspases are activated by the mitochondrial and death receptor pathways.

become amino acids and nucleic acids become nucleotides. All this occurs within a membrane-bound vesicle where the lysosomal enzymes mix with the substrate. How this vacuole forms is what distinguishes the three types of autophagy (Table 2.7). In macroautophagy, the substrates are first packaged in an autophagosome, which has an outer membrane that is not derived from a lysosome. This then joins with a lysosome and the dividing membranes break down, allowing the enzymes to reach the substrate. In microautophagy, the lysosome itself engulfs the substrate. In chaperone-mediated autophagy, receptors on the surface of the lysosome selectively bind specific substances and allow them to translocate into the lysosomal lumen. Autophagy (Atg) genes identified in yeast are conserved and function in many animals, including mammals. Autophagy appears to be able both to protect a cell and to destroy it (Fig. 2.22). When nutrients are scarce, digestion of intracellular macromolecules can provide the energy to maintain minimal cell functioning. Pathogens and toxins may be segregated and degraded. Abnormal proteins or damaged organelles can be eliminated. All of these can play an important homeostatic role in the early stages of a disease process Why then is it also a mechanism for killing the cell? The theory is that it protects the cell by removing damaged cell components for as long as it can, but if it loses that battle it is best to remove the whole cell, i.e. to order cell death. Apoptosis, autophagy and disease

It follows from the previous discussion that a lack of balance in initiating or suppressing programmed cell death can lead to problems.

67

Apoptosis is an important host defence mechanism against viral infection. When viruses infect cells, they attempt to take over the cell’s replication machinery in order to proliferate and spread. Viral antigens are expressed on the host cell membrane and (in concert with CD4+ lymphocytes) CD8+ lymphocytes bind to this and secrete perforin, lysing the affected cell. Alternatively, natural killer (NK) cells recognise the viral antigen and initiate cell lysis. Viruses can sometimes get around this, with antiapoptotic mechanisms, which come into play when breaks occur in DNA strands as the viral genome incorporates itself into the cell’s DNA. Many viruses code for proteins that block apoptosis. Examples include the inactivation of p53 by human papillomavirus type 16 (HPV16) and Epstein– Barr virus (EBV), which produce molecules that either simulate bcl2 or block molecules related to the TNF/ CD40 pathway. Too much apoptosis may also be seen if the effector mechanisms malfunction, as is seen in AIDS, when infection of T cells by HIV leads to deletion of the CD4+ population of T cells, wreaking havoc in the immune system due to the loss of its most crucial regulatory cell. HIV expresses gp120, which activates the fas ligand on CD4+ cells and leads to apoptosis. CD4+ cells are essential for the generation of memory to intercurrent and opportunistic infections. It has become clear that apoptosis (or the lack of it) has a role in the development of tumours. Inactivation of the cell cycle regulatory genes (which act as ‘quality control officers’ on the integrity of the DNA) will permit mutations to be passed to daughter cells by allowing cell replication to take place; usually such cells would be commanded to undergo apoptosis.

Table 2.7 Comparison of the types of autophagy Characteristic

Macroautophagy

Microautophagy

Occurs in

Stress

Normal physiology Stress

Sequestering membrane

Non-lysosomal

Lysosomal



Receptor mediated

No

No

Yes

Engulfs organelles

Yes

Yes

No

Digests soluble cytosolic proteins

Yes

Yes

Only KFERQ-tagged

KFERQ, the one-letter code for the amino acid sequence Lys-Phe-Glu-Arg-Gln.

Chaperone-mediated autophagy

Chapter 2: What are the common mechanisms of disease?

Apoptosis and autophagy

68

What are the common mechanisms of disease?

Starvation Infection

Autophagic mechanisms activated

Stimulation of receptor, e.g. TNF receptor lead to activation of TRAIL

Autophagosome forms by surrounding cellular protein or other material in a double lipid membrane

Early stages: conservation of materials by recycling of worn out or surplus intracellular components

Later stages: critical loss of cellular elements, cell undergoes central vacuolation

TNF receptor Lack of growth factor

Autophagolysosome digests the contents into protein and other elements

Lyosome fuses with autophagosome Recycling of protein and other elements

Autophagic cell death

Indigestible remnants stored in membrane-bound vesicles

Cell survival through lean period

Part 1: Disease, health and medicine

Figure 2.22 Autophagy: its role in cellular conservation and in cell death. In autophagic cell death, which is often a response to chronic adverse environmental factors such as starvation, the process of lysosomal degradation of cellular components within membrane-bound vesicles has already occurred before cell breakdown. The cells often appear vacuolated under the microscope as they undergo this process. Table 2.8 Possible effects of autophagy in various conditions Disease

Change in autophagy

Cancer – early stage

Inactivation favours tumour growth

Cancer – late stage

Activation allows survival of cells in centre of tumour

Vacuolar myopathies

Inactivation leads to accumulation of vacuoles that weaken muscles

Neurodegeneration – early

Activation helps remove protein aggregates (helpful)

Neurodegeneration – late

Damaged neurons undergo autophagic cell death

Axonal injury

Inactivation prevents removal of damaged organelles and transmitter vacuoles so that neurotransmitters are released in cells and induce apoptosis

Infectious disease

Inhibition facilitates viral infection of cells and allows survival of bacteria

Degenerative diseases and ageing

Physiological changes of ageing

DEGENERATIVE DISEASES AND AGEING

Cardiovascular system ↓ Vessel elasticity ↓ Number of heart muscle fibres ↑ Size of muscle fibres ↓ Stroke volume Respiratory system ↓ Chest wall compliance ↓ Alveolar ventilation ↓ Lung volume Gastrointestinal system ↓ Bowel motility ↓ Enzyme, acid and intrinsic factor production ↓ Hepatic function Urinary system ↓ GFR ↓ Concentrating ability Nervous system Degeneration and atrophy of 25–45% of neurons ↓ Neurotransmitters and conduction rate Musculoskeletal system ↓ Muscle mass ↑ Bone demineralisation ↑ Joint degeneration Immune system ↓ Inflammatory response Skin ↓ Subcutaneous fat and elastin ↓ Sweat glands ↓ Temperature regulation through arterioles

and mitochondrial functioning declines between the age of 40 and 100 years. Replicative senescence is the term describing a cell’s inability to divide. Cells do appear to have a finite number of divisions and this may be linked to telomere shortening. Telomeres are at the linear ends of chromosomes and are important for ensuring their complete replication. With each replication, the telomere becomes shorter until the chromosome ends are damaged and this stops

Chapter 2: What are the common mechanisms of disease?

This is a pathology book about the mechanisms of disease and this is a chapter on cell damage and cell death – so you won’t be surprised that we will concentrate on the tissue, cellular and molecular changes in ageing. However, that is the reductionist approach, and thinking more holistically about patients and how and why they age would lead us to psychological and sociological theories. Just as we debated ‘what is a disease?’, we can similarly ask ‘what is ageing?’. Is it just the passage of time (chronological ageing), changes that will happen to us all if we live long enough (universal ageing) (see box) or specific agelinked diseases that affect only some people (probabilistic ageing)? Many societies treat people differently as they become older and retire, and these changes in expectations can affect health, especially mental health. Some people benefit from keeping active whereas others may be content to disengage; personality, past events and current circumstances will all have an influence. To return to the cell, you could deduce that the factors which cause damage to DNA, cell membranes and cell organelles are likely to play a part in accelerating ageing and factors important in repair, such as autophagy and poly(ADP-ribose) polymerase, slow down cellular ageing. Starting with DNA, it is important to distinguish between DNA damage and DNA mutations. Damage occurs normally every day as single- and double-strand breaks occur and are detected and repaired by enzymes. In the mouse, this is estimated to be thousands of lesions per hour in each cell! Mutations are changes in base sequences in DNA and these are not detected or repaired. You won’t be surprised to learn that there are a variety of human disorders called ‘DNA repair deficiency disorders’ and these cause accelerated ageing and increased risk of cancer. Generally, if the tissue is composed of frequently dividing cells (e.g. gut), then there is an increased risk of cancer, whereas cells which don’t divide (e.g. heart and brain) have increased ageing. Hereditary non-polyposis colorectal cancer is a common tumour and sufferers have mutations in one of the mismatch repair genes (see page 346). An alternative mechanism is that transcription declines with age and that affects function. It is known that transcription of certain genes in brain tissue responsible for synaptic plasticity, vesicular transport

69

70

What are the common mechanisms of disease?

Key facts Structural and biochemical changes in cellular ageing l l

Reduced oxidative phosphorylation in mitochondria Reduced synthesis of nucleic acids, transcription factors, proteins and cell receptors

l

Decreased uptake of nutrients

l

Reduced repair mechanisms

l

Irregular, abnormally lobed nuclei

l

Pleomorphic mitochondria

l

Decreased endoplasmic reticulum

l

Distorted Golgi apparatus

l

Accumulation of lipofuscin

l

Accumulation of abnormally folded proteins

Figure 2.23 Ageing is normal!

Case study: osteoporosis

Part 1: Disease, health and medicine

A 70-year-old woman, Mrs Moore, comes to accident and emergency with severe back pain after stretching to open her bedroom window. On examination she is tender over her thoracolumbar junction. She has a lateral radiograph of her thoracolumbar spine. which shows osteoporosis. Question: What is osteoporosis? Answer: Osteoporosis occurs when bone becomes demineralised faster than the body can replace the minerals. Bones become fragile and weak and may break with normal stresses, such as climbing stairs and opening windows. Sites most likely to have osteoporotic fractures include the distal radius, pelvis, femoral neck and vertebral column. The condition is very common in elderly people, affecting almost 80% of postmenopausal women. Osteoporotic patients gradually lose height over time as they develop crush fractures in several vertebrae. The back also becomes more curved (kyphotic) as the vertebrae become wedged. Question: What are the risks for osteoporosis?

Answer:

Menopause – lack of oestrogen leads to rapid demineralisation of bone Decreased calcium and vitamin intake Inactive lifestyle Alcohol Smoking

Question:

What do the three pictures in Fig. 2.24 show? (a) An isotope bone scan showing activity in the L1 vertebral fracture indicating that there is an osteoblastic response at the fracture site, so it is a relatively recent injury. (b) Sagittal short TI inversion recovery (STIR) MRI showing bone marrow oedema in the L1 crush fracture. The bright signal reflects increased water content at the fracture site and tells the radiologist that this is an acute and symptomatic injury that may benefit from vertebroplasty. (c) Typical appearance of a pathological wedge fracture of a vertebra. In this example the spine contains metastatic tumour.

Answer:

Mechanisms in infectious diseases

(b)

Figure 2.24

the cell cycle. In cells, such as stem cells or germ cells, that need to continue to divide, there is an enzyme called telomerase that can lengthen the chromosomes. This might also be important in some cancers. Ageing is not just a random process but involves specific genes, receptors and signals. One of the mechanisms involves insulin growth factor 1 (IGF-1) pathways and another involves mTOR, which you will recall affects autophagy. Several of these seem to be affected by calorie restriction diets and this has been shown to delay ageing in the nematode C. elegans, yeast, flies and mice. However, the effect decreases in higher order animals. Around 800 genes are currently being investigated for their role in ageing and are collated on the GenAge Database.

MECHANISMS IN INFECTIOUS DISEASES Two common mechanisms of bacterial disease are toxin production (endotoxin and exotoxins) and direct cell damage. BACTERIAL ENDOTOXIN

Endotoxin is not secreted by living bacteria but is a cell wall component that is shed when the bacterium dies. The component is called lipid A, which is part of the lipopolysaccharide (LPS) in the outer cell wall of Gram-negative bacteria. It causes fever, and

(c)

Read more in Pathology in Clinical Practice Case 29

macrophage and B-cell activation by inducing host cytokines. Only Gram-negative bacteria have endotoxin, with the one exception being the Gram-positive Listeria monocytogenes. Endotoxin is a potent stimulator of a wide range of immune responses. To the immune system, the recognition of LPS spells danger and warrants an immediate and dramatic response, which is often detrimental to the host itself. Clinically, this manifests as fever and vascular collapse or shock. Macrophages are stimulated by LPS to produce TNF and interleukin 1 (IL-1) which have many effects, including acting directly on the hypothalamus to produce fever (see page 139). LPS also stimulates, directly or indirectly, the complement and clotting pathways and platelets to produce DIC (disseminated intravascular coagulation, see page 236), thrombosis and shock (see page 259, Fig. 9.1). Shock results from increased vascular permeability produced by mediators from mast cells and platelets, combined with the TNF and LPS affecting endothelial cells. LPS also stimulates the liver to produce acute-phase proteins (see page 139) and hypoglycaemia. In Gram-positive organisms, lipotechoic acid within the bacterial wall causes problems similar to LPS in Gram-negative bacteria (Fig. 2.25). BACTERIAL EXOTOXINS

Exotoxins are proteins released by living bacteria and there are a wide variety with different actions. Neurotoxins act on nerves or endplates to produce

Chapter 2: What are the common mechanisms of disease?

(a)

71

72

What are the common mechanisms of disease?

Gs protein (inactive form) Basolateral membrane

γ

Gs protein (active form) α

β α

α

GDP

Normal state Activation of Gs proteins in the basolateral intestinal cell membrane by the addition of GTP leads to the production of cAMP via adenyl cyclase. Together with calcium, cAMPcontrols intracellular kinases, which regulate sodium, chloride and water levels in the cell. Any excess passes into the intestinal lumen

Other membrane interactions

Adenyl cyclase

GTP GTP

GDP

+ CAM

GTPase

Regulation of Na+,CI- and H2O

Ca2+

cAMP

+

A and G kinases

Other kinases

+/-

+/-

+/-

+/-

(Villus cell)

(Crypt cell) Intestinal lumen

GM1 receptors

Na+ CI- H2O B B A B Cholera toxin B B

1. Cholera toxin: one A and 5 B subunits, the latter attaching via GM1 ganglioside receptors on the intestinal cell

CI- and H2O efflux

B B A B B B

Part 1: Disease, health and medicine

Block A

Intestinal lumen (Crypt cell)

(Villus cell) -

A1

A2

+

-

+

A and G kinases

Other kinases

A1

4. The altered subunit cannot be ‘switched off’ by the normal enzymic mechanism and there is an uncontrolled increase in cAMP 5. The net result is an antiabsorptive effect at the villus cell and a secretory effect at the crypt cell level, causing severe diarrhoea

CI- H2O

Na+ CI- H2O

2. The A unit is translocated across the membrane and splits to form an active enzyme (A1 component) 3. The A1 enzyme causes ADP ribosylation of the alpha subunit of the Gs protein complex

CI- H2O

+ cAMP CAM

Ribosylation GDP

ADPr

ADPr

α

α

GDP γ β

α

Adenyl cyclase

Ca2+

Other membrane interactions

Gs protein locked in active form

Figure 2.25 Cholera toxin permanently activates a Gs protein in the intestinal cell wall. The diagram shows an intestinal cell in the normal state (top) and affected by cholera toxin (below); the diagrams are mirror images with the intestinal lumen in the middle. Cells in the intestinal villus: Na+ and Cl– absorption is blocked by cholera toxin. Crypt cells: this is the site most affected by cholera toxin. Here Cl– secretion is stimulated. Water passively follows the active ion transport. Dehydration occurs. Although the main Na+ absorption path (Na+/H+ exchange) is blocked by cholera toxin, Na+ can be co-transported with glucose or amino acids, which is why oral rehydration solutions using glucose are effective in cholera. (Courtesy of Professor Phil Butcher, St George’s, University of London.)

Mechanisms in infectious diseases

paralysis, cytotoxins damage a variety of cells, tissue-invasive toxins are often enzymes capable of digesting host tissues, and pyrogenic toxins stimulate cytokine release and cause rashes, fever and toxic shock syndrome (TSS). A very important group is the enterotoxins which act on the gastrointestinal tract to produce diarrhoea by inhibiting salt absorption, stimulating salt excretion or killing intestinal cells. There are two broad categories: infectious diarrhoea and food poisoning with preformed toxin. In infectious diarrhoea, the bacteria proliferate in the gut, continuously releasing enterotoxin. The symptoms do not occur immediately after ingestion but require a day or two for the bugs to become established in sufficient numbers. In food poisoning, the bacteria grow in the food releasing their exotoxin. This acts very quickly after ingestion to produce diarrhoea, abdominal pain

73

and vomiting, but the symptoms last for only 24 hours because no new toxin is created. Bacterial exotoxins may be classified by their site of action: l

l

l

Extracellular, e.g. epidermolytic toxin Staphylococcus aureus which causes scalded skin syndrome At cell membrane level (not transported into cell, but cause changes in intracellular cGMP), e.g. Escherichia coli heat-stable enterotoxin (ST), causing travellers’ diarrhoea, and Staphylococcus aureus TSST 1 toxin leading to toxic shock syndrome On the cell membrane, causing pore formation or disruption of lipid by enzymatic activity, e.g. phospholipase C activity toxin Clostridium perfringens, spore-forming toxins (thiol-activating haemolysins) such as streptolysin (Streptoccus pyogenes),

John Snow and the Broad Street pump It would be quite possible to go through the entire medical curriculum without ever hearing the name of John Snow (1813–58). He was a man of simple habits and seemed to lack the charisma that is vital in attracting attention on the world stage. Yet his contributions to medicine were certainly ‘world class’. If you think that epidemiological studies might be dull, Snow’s biography is well worth a read. It demonstrates eloquently how they can have a truly dramatic impact on public health. Snow is perhaps best remembered for having anaesthetised Queen Victoria in 1853 and 1857, giving credibility to the use of pain relief during childbirth! But it is his contribution to the understanding of cholera that is relevant here. Snow’s link with cholera evolved over a number of years. His first encounter with the disease was in Newcastle upon Tyne during the epidemic of 1831–2, when he had just started his medical training. It was during the next cholera epidemic of 1848–9 that he made his seminal contribution. By now Snow was in London and here he began to unravel the mode of transmission of the disease. Snow’s work was a masterpiece in epidemiological investigation. He meticulously mapped the houses in which new cases of cholera were being diagnosed and observed a marked difference in the incidence and mortality of cholera in the south of London (8 deaths per 1000 inhabitants) compared with other areas (1–4 deaths

per 1000 inhabitants). This led him to hypothesise that cholera was spread by water. He identified the public pumps from which the families living in the affected and unaffected areas drew their water. He noticed that there were surprising sites of sparing within otherwise heavily affected areas. His suspicion that the infection was in the water supply grew when he found that those living in the spared areas worked at a local brewery and received free beer, which was made with water from a source away from that supplying their homes. Even before the identification of bacteria, he postulated that the transmission was the result of a living organism that had the ability to multiply. Snow is of course remembered for urging that the handle be removed from the pump that supplied contaminated water in Broad Street in London during the epidemic of 1854 (Fig. 2.26). This led to a dramatic decrease in new cases in the area. Snow postulated that social conditions and hygiene were of paramount importance in the spread of infection. Although there was no medical treatment for cholera, it became apparent that the way to stop an epidemic was through good sanitation and good hygiene. It follows from this that our first line of defence against infection is the prevention of the multiplication and spread of organisms. Open sewers, overcrowded living conditions, contaminated drinking water, poor food storage and preparation, inadequate personal hygiene and unprotected sexual (Continued)

Chapter 2: What are the common mechanisms of disease?

History

74

What are the common mechanisms of disease?

Part 1: Disease, health and medicine

(Continued) contact are a recipe for disaster. This is not a text on public health medicine or politics, so we will not dwell on these points. But remember that, each year in Asia, Africa and Latin America, roughly 4–6 million people die from diarrhoea and 1–2 million die from malaria. One-third of the world’s population is

subclinically infected with tuberculosis and 3 million die from TB each year; 12 million people are infected with HIV worldwide. All these problems are more likely to be solved by engineers and politicians than by the latest advances in molecular biology!

Figure 2.26 Deaths from cholera (–) in Broad Street, Golden Square, London, and the neighbourhood, 19 August to 30 September 1854.Water pumps are shown. John Snow realised that deaths were clustered around the Broad Street pump. Families working for the local brewery received free beer in preference to water and did not catch cholera! (Reproduced with permission from Wellcome Library, London.)

Mechanisms in infectious diseases

– ADP ribosylation (cholera, diphtheria, pertussis) – N-glycosidases (shiga toxin) – glucosyl transferases (C. difficile toxin A and B) – Zn2+-requiring endopeptidases (tetanus and botulism toxins). GTP-binding proteins are often the target for ADP ribosylation by type III bacterial exotoxins. These proteins are involved in signal transduction and regulation of cellular function either by cAMP levels or kinase cascades leading to transcription modification, for example: l

l

l

G proteins (stimulatory or inhibitory of adenylate cyclase): cholera, pertussis, E. coli LT toxin Elongation factor 2 (translational control): diphtheria toxin Rho proteins (small G proteins; regulate actin cytoskeleton): inactivated by C. difficile A and B toxins, and can also be activated by deamination by pertussis necrotising toxin.

The structure of these toxins consists of either A:B5 (enzyme active A subunit and 5 × B subunits required for binding) or A:B type with A (enzyme active) and B (binding) domains on a single polypeptide chain. A:B5 types are seen in cholera and pertussis (whooping cough), and E. coli LT1 and LT2. A:B types are encountered in diphtheria, botulism and tetanus. It is fascinating to compare the contrasting actions between two structurally similar neurotoxins produced by members of the same bacterial family, C. tetani and C. botulinum, which cause tetanus and botulism respectively. These diseases are purely due to toxin-mediated action following infection and are quite different in pathology, yet the molecular action of the two toxins is identical. They are both endopeptidases specific for synaptobrevin, a protein found in the

cytoplasm of synaptic vesicles. However, the binding (B) domains of the toxins show different specificities for cell receptors. Tetanus toxin binds to the gangliosides of the neuronal membrane, is internalised and moves by retroaxonal transport from peripheral nerves to the CNS, where it is released from the post-synaptic dendrites and localises in presynaptic nerve terminals. This blocks the release of inhibitory neurotransmitter, -aminobutyric acid to cause unopposed, continuous, excitatory synaptic activity, leading to spastic paralysis. Botulism toxin binds ganglioside receptors of cholinergic synapses and prevents release of acetylcholine at the neuromuscular junctions, causing flaccid paralysis (Fig. 2.27). ANTIMICROBIAL RESISTANCE

In the battle between humans and microbes, antibiotics are produced and bacteria respond by developing resistance. How does that occur and what can be done to minimise it? The rapid rate of reproduction aligned with horizontal gene transfer by conjugation, transduction and transformation means that bacteria can readily acquire new characteristics. It is thus inevitable that, after introduction of any new antibacterial, resistance will develop at some stage. Overusage or inappropriate usage will accelerate this problem. Resistance mechanisms can spread from one bacterium to another (and from different species to different species) and the resistant bacteria can of course spread from human to human, or in some increasingly recognised situations from animals to humans and vice versa. The acquisition of antibacterial resistance may sometimes come at a fitness cost to a bacterium – in other words, resistant bacteria are not always as virulent as the wild-type strains. This is because there is a finite capacity, spatially and energetically, for genetic material within a bacterial cell and acquisition of resistance genes may result in loss of virulence genes. However, sometimes resistance factors and virulence factors may be co-acquired, e.g. on a plasmid, resulting in a virulent and difficult-to-treat organism. Over the years there have been waves of different resistant bacteria and the problem organisms seem to shift from Gram-negative bacteria to Grampositive bacteria and back to Gram-negative bacteria.

Chapter 2: What are the common mechanisms of disease?

l

pneumolysin (Strep. pneumoniae), listeriolysin (Listeria monocytogenes), perfringolysin (C. perfringens; gas gangrene), cerolysin (Bacillus cereus; food poisoning) Type III toxins, which act intracellularly by translocating an enzymatic component across the membrane (A subunit of domain) which modifies an acceptor molecule in the cytoplasm. These can be grouped by type of enzymatic activity:

75

76

What are the common mechanisms of disease?

ACH is not released

Ganglioside receptor blocked ACH ACH ACH

Botulinus toxin

ACH ACH receptors Neuromuscular junction Muscle not stimulated to contract

Part 1: Disease, health and medicine

Figure 2.27 Botulinum toxin binds ganglioside receptors of cholinergic synapses and prevents release of acetylcholine at the neuromuscular junctions, causing flaccid paralysis.

It is not entirely clear why this may be, but changes in the use of different classes of antibacterials must play a big part. The mid-1990s saw the rise of problems with multiply resistant Gram-positive bacteria such as MRSA (meticillin-resistant Staphylococcus aureus) and GRE (glycopeptide-resistant enterococci) and C. difficile. Concerted efforts using well-defined infection prevention and control interventions (screening, isolation, decolonisation, improved environmental decontamination and – perhaps the most important – hand washing by healthcare workers), as well as improved antimicrobial stewardship (use of the right drug at the right time by the right route for the right duration), has helped to reduce the problems with these organisms. But despite this there are still major problems currently and ahead of us. Most worryingly there has been a rise in the incidence of multiply resistant Gram-negative bacteria, especially the Enterobacteriaceae such as Klebsiella sp. which can cause urinary tract, respiratory, abdominal and surgical site infections. Some of these multiply resistant bacteria have new resistance mechanisms such as carbapenemase enzymes, which means that the effectiveness

of carbapenems such as meropenem is curtailed by lysis of the drug. Often these isolates are resistant to many other classes of antibiotics including aminoglycosides and quinolones, as well as the remaining -lactams. With virtually no new antibacterials likely to be appearing soon, it means that older drugs have been rejuvenated because, interestingly, these bacteria often remain sensitive to them, probably because there has been little selective pressure to limited recent usage. A good example is the polymyxin antibiotic colistin. However, once usage of these old drugs picks up it is likely that resistance will start to be seen against these agents as well. DIRECT CELL DAMAGE BY VIRUSES

Microorganisms can also damage cells directly and this is particularly true of viruses. Viruses are obligate intracellular organisms requiring the host cell’s machinery for replication (Figs 2.28–2.30 and see Table 1.4 in Chapter 1). Viruses use three main methods for entering cells:

Mechanisms in infectious diseases

77

Virus

Viral entry by membrane fusion

Viral envelope fuses with membrane

Some viruses can translocate directly across the host cell membrane

Viral entry by receptor–mediated endocytosis

Membranes close over endocytotic vesicle

Nucleocapsid enters cell; capsid is lost

Nucleic acid remains

Envelope (if present) + capsid lost, leaving nucleic acid

Further steps depend on nucleic acid type: ‘positive’ RNA viruses can act directly on the cytoplasmic organelles, DNA viruses move to thenucleus, ‘negative’ RNA viruses, retroviruses and some DNA virusesrequire translation by incorporated viral enzymes

Figure 2.28 After attaching to the host cell via a receptor, the viral particle enters the cell by fusion of the viral and host cell membranes or by receptor-mediated endocytosis. It then uncoats and releases its nucleic acid. Replication occurs either in the cytoplasm alone, as with most RNA viruses and rare DNA viruses (the pox viruses), or involves both cytoplasmic and nuclear steps. DNA viruses can integrate directly with the host DNA but the only RNA viruses that can achieve nuclear integration are the retroviruses.

Chapter 2: What are the common mechanisms of disease?

Virus attaches via host cell receptor

Host cell nucleus contains dsDNA

78

What are the common mechanisms of disease?

Retrovirus

Retrovirus binds to a human surface receptor (e.g. HIV-1virus binds using its gp120 antigen to human CD4 onT Iymphocytes). The viral RNA contains the followinggenes:pol (encodes reverse transcriptase and integrase), gag(encodes viral structural proteins), env (encodes envelope proteins, including gp120), reg (encodes regulatory genes, e.g. tat,rev)

Part 1: Disease, health and medicine

The viral and host cell membranes fuse, the viral capsid disintegrates and viral RNA enters the human cell. Viral reverse transcriptase catalyses the production of double-stranded DNA, which enters the host cell nucleus and integrates into host DNA under the influence of integrase

The host cell is induced to manufacture and a ssemble new virions by long terminal repeat sequences and genestatandrev.The new virions peel off an envelope of host cell membrane as they exit

Figure 2.29 Retroviruses, similar to DNA viruses, can insert their genes into the host cell DNA, but require an additional step. Their RNA is translated into DNA by reverse transcriptase, supplied by the virus.

1 Translocation of the entire virus through the cell membrane 2 Fusion of the viral envelope with the cell membrane 3 Receptor-mediated endocytosis of the virus, followed by fusion with the endosome membrane.

What happens once the virus is inside the cell? First the particle must uncoat and separate its genome from its structural components. It then uses specific enzymes of its own or present in the host cell to synthesise viral genome, enzyme and capsid proteins. These must be

Mechanisms in infectious diseases

B After a transient illness, viral DNA integrates into host nuclear DNA and lies dormant

Virus

Viral DNA

If virus is not cleared by immune mechanisms, replication and inflammation continue, e.g. hepatitis B or C infection may lead to cirrhosis

D Tumour formation, e.g. hepatitis B virus (hepatocellular carcinoma), Epstein-Barr virus (Burkett's lymphoma, nasopharyngeal carcinoma, Hodgkin lymphoma)

C Infected cell manufactures new viral particles, released by the rupture of the host cell

E Reactivation of latent infection in a debilitated or immunosuppressed patient, e.g. herpes simplex (cold sore, genital ulcer) varicella zoster (shingles)

Figure 2.30 Possible outcomes in human cells infected by virus in which viral clearance is not achieved by the host’s immune mechanisms.

assembled and released, either directly (unencapsulated viruses) or by budding through the host cell’s membrane (encapsulated viruses). Viruses can damage the host cell directly in a variety of ways: l

l l

Interference with host cell synthesis of DNA, RNA or proteins (e.g. polio virus modifying ribosomes so that they no longer recognise host mRNA) Lysis of host cells (e.g. polio virus lyses neurons) Inserting proteins into host cell membrane, so provoking an immune attack by host cytotoxic lymphocytes (e.g. hepatitis B and liver cells)

l

l

Inserting proteins into host cell membrane to cause direct damage or promote cell fusion (e.g. herpes, measles, HIV) Transforming host cells into malignant tumours (e.g. EBV, HPV, HTLV-1).

Alternatively, the host cell may suffer secondary damage due to viral infection. This may be due to the following: l

Increased susceptibility to infection due to damaged host defences (e.g. influenza viral damage to respiratory epithelium facilitates bacterial pneumonia with

Chapter 2: What are the common mechanisms of disease?

A Viral particles continually manufactured by host cell. Cell remains intact; virus particles exit, some taking part of the cell membrane as an envelope

79

80

l

What are the common mechanisms of disease?

Staph. aureus, HIV depletes CD4+ T cells, allowing opportunistic infections such as Pneumocystis jirovecii pneumonia) Death or atrophy of cells dependent on virally damaged cell (e.g. muscle cell atrophy after motor neuron damage by polio virus).

Re-exposure produces an immediate reaction to the offending agent! The extrinsic allergen (e.g. grass pollen, house dust mite faeces, seafood) binds to IgE on the surface of mast cells in the mucosa of the bronchial tree, nose, gut or conjunctivae, leading to the release of chemical mediators (Fig. 2.31).

HYPERSENSITIVITY REACTIONS The phenomenon of damage caused by the immune system while trying to combat an insult is referred to as hypersensitivity. As a cause, it is partially ‘intrinsic’ and partially ‘extrinsic’. The precipitating factor may be an external agent but, rather than the immune system coping and restoring the body’s homeostasis, the immune response is altered and becomes the ‘internal’ cause of the disease. Four main types of hypersensitivity reaction were described by Gell and Coombs, although several types may operate together. Types I–III involve antibodies and type IV involves cells. The following are the mechanisms involved: l

l

l

Part 1: Disease, health and medicine

l

Release of allergic mediators in type I (anaphylactic) hypersensitivity Binding of self-reactive antibodies to cells in type II (antibody-dependent cytotoxic) hypersensitivity Damage to blood vessel walls and tissues by circulating immune complexes in type III (immune complex-mediated) hypersensitivity Delayed tissue damage due to interactions between sensitised T lymphocytes and other inflammatory cells in type IV (cell-mediated) hypersensitivity.

First exposure: nasal and bronchial mucosa exposed to pollen, dust etc.

Soluble antigen stimulates production of IgE antibodies

Fc of IgE attaches to receptor on mucosal mast cell

TYPE I: ANAPHYLACTIC HYPERSENSITIVITY

This is the mechanism behind atopic allergies such as asthma, eczema, hay fever and reactions to certain food. Patients who have this problem have been previously exposed to the allergen and have generated IgE antibodies against it. The IgE antibodies attach to mast cells via cell surface receptors. A common feature of many allergens is that they have repetition of the same antigenic determinant on their surface. This means that cross-linking of IgE molecules attached to the same mast cell is likely. It is cross-linkage that activates the mast cell.

Second exposure to pollen; cross-linking of IgE molecules on mast cell stimulates release of primary and secondary inflammatory mediators

Figure 2.31 Type I hypersensitivity.

Hypersensitivity reactions

IgE bound to mast cells is crosslinked by binding multiple copies of antigen on parasite

systemic effects. Effects include constriction of smooth muscle in bronchi and bronchioles, causing wheezing, dilatation and increased permeability of capillaries, and resulting in localised tissue oedema, increased nasal and bronchial secretions, red watery eyes, skin rashes and diarrhoea. If laryngeal oedema or bronchospasm is severe, death may result from respiratory obstruction. What can be done about this problem? Antihistamines antagonise the action of histamine and can relieve many of the symptoms. Steroids, which inhibit the leukotriene pathway, can prevent or alleviate the longer-term symptoms. If an ‘atopic’ patient knows that he is likely to encounter an antigen on a particular occasion (e.g. pollen in a hay fever sufferer) he may be able to take drugs that stabilise the mast cell membrane, reducing or preventing its activation.

AA

PLA2

COX

Lipox

PG

LT

Immediate (onset and duration of effect in minutes)

Delayed (onset and duration of effect in hours)

Release of pre-formed mediators from granules, including histamine and lysosyme

Generation of arachidonic acid metabolites

Figure 2.32 Mast cell activation is typically stimulated by IgE antibody cross-linkage as illustrated here, but this is not always necessary. Mast cells can also be directly stimulated by some substances, e.g. mellitin from bee stings or C3a5a complement fragments. AA, arachidonic acid; COX, cyclooxygenase; Lipox, lipoxygenase; LT, leukotrienes; PLA2, phospholipase A2; PG, prostaglandins.

Chapter 2: What are the common mechanisms of disease?

Some mediators, such as histamine, are already formed within mast cell granules and the cross-linking of IgE molecules attached to the mast cell surface stimulates the release of the granule contents into the tissues (Fig. 2.32). This is why the effect of antigen exposure can be seen within 5 minutes; the granule contents elicit an inflammatory response that lasts up to an hour. In the meantime, the cross-linked IgE molecules stimulate the membrane of the mast cell to generate arachidonic acid and its metabolites within its cell membrane. Release of leukotrienes, prostaglandins and platelet-activating factor by this mechanism sparks a response that starts 8–12 hours later and may last from 24 hours to 36 hours. Generally, the mediators released by these processes act locally but they can also produce life-threatening

81

82

What are the common mechanisms of disease?

A course of injections of a very dilute solution of the offending allergen can eventually lead to IgG instead of IgE being generated; because IgG does not stick to mast cells, the problem is avoided. But care must be taken with this approach not to generate anaphylactic shock, a life-threatening complication. In this condition, the patient must be injected immediately, subcutaneously, with adrenaline, which reverses the actions of the mediators causing bronchospasm and oedema. Patients known to be susceptible to this type of response (often to bee stings or peanut protein) carry an EpiPen with them, which can deliver a single dose of adrenaline. The action of adrenaline lasts only a finite time, and patients should also be given antihistamine or steroids as back-up. Why should some people produce the IgE molecules that mediate this reaction? Genes play a part, and also a particularly strong reaction by TH2 cells (helper T lymphocytes) to the allergen, resulting in release of three crucial cytokines: IL-4 that makes B cells switch to producing and secreting IgE; IL-5 that attracts eosinophils which contribute to the response through the release of their granules; and IL-13 that stimulates epithelial cell mucus production. TYPE II: ANTIBODY-DEPENDENT CYTOTOXIC HYPERSENSITIVITY

Part 1: Disease, health and medicine

In this type of hypersensitivity, antibodies react with antigens fixed to the surfaces of various types of body cell to cause damage. The effects can be mediated in the following ways: l l l

Activation of the complement system Targeting the cell for phagocytosis Affecting the cell’s function.

In the complement-fixing type, antibodies bind to an antigen, fix complement, and so cause destruction of the target cell or trigger inflammatory cells. When circulating cells are coated with antibody, the Fc part of the antibody can bind the cell to phagocytic cells, such as neutrophils and macrophages. Circulating blood cells and blood components are particularly vulnerable and so there is autoimmune haemolytic anaemia (destruction of red blood cells), autoimmune thrombocytopenic purpura (platelet destruction) and drug-related agranulocytosis (granulocyte destruction).

You will recall that IgG crosses the placenta and so blood group incompatibility between the mother and the baby can cause problems. This is illustrated in Fig. 2.33 where rhesus antibodies from a rhesusnegative (Rh-) mother cross the placenta to damage the red cells of a Rh+ baby. For routine blood transfusions, it is essential that any blood which is transfused into a patient is first crossmatched to ensure that the recipient does not possess antibodies to antigens on the donor red blood cells. This is because the donor red cells will become coated with antibody and/or complement which may promote phagocytosis due to opsonisation, via the Fc or C3b, or fix complement to produce cell membrane damage through C8 and C9 membrane attack complex. In the functional type of type II hypersensitivity, antibody binding interferes with cell function; often by interfering with hormone receptors, but also with cell growth and differentiation or cell motility. Included in this group are antibodies that block cell function (as in some forms of Addison’s disease where autoantibodies develop to adrenocortical proteins) or antibodies which stimulate receptor function (as in Graves’ disease, causing thyrotoxicosis). Type II reactions are also involved in some drug reactions (e.g. chlorpromazine-induced haemolytic anaemia and quinidine-induced agranulocytosis) where binding of a drug may alter a normal self-antigen. Natural killer cells, which are not restricted by HLA type, can kill through antibody-dependent cell-mediated cytotoxicity (ADCC) which may be important for killing large parasites or tumour cells. TYPE III: IMMUNE COMPLEX-MEDIATED HYPERSENSITIVITY

Similar to type II, this is an antibody-driven process, but the difference is that antibodies react with free antigen. Whether the complexes are soluble or insoluble depends on the ratio of antibody to antigen. These immune complexes can circulate in the blood, giving rise to ‘serum sickness’ or they can be deposited locally (Arthus reaction). Both the soluble and the insoluble types of complex can activate macrophages, aggregate platelets and initiate the complement cascade. Circulating immune complexes can lodge in the small vessels of many organs to cause a vasculitis (inflammation of the blood vessel

Hypersensitivity reactions

83

First pregnancy: Rhesus D-mother, Rhesus D-fetus

Fetal red cells leak into maternal circulation at parturition (delivery)

Treated

Mother develops anti-Rh D antibodies; baby usually unscathed

Anti-Rh D immunoglobulin (Ig) injected within 48 hours

Anti-Rh D antibodies cross placenta

All Rh D-fetal red cells are bound by the injected Ig and cleared by the liver and spleen

Second pregnancy with Rh D-fetus Anti-Rh D antibodies and memory B cells remain

Second pregnancy proceeds as first

Fetal red blood cells lysed. Fetus dies (‘hydrops fetalis’)

Healthy baby. Mother again injected with anti-Rh D Ig

Figure 2.33 Type II hypersensitivity: rhesus incompatibility: an untreated rhesus-negative mother may develop antibodies if she has a Rh+ baby. In the next pregnancy her antibodies cross the placenta, attach to Rh antigens on the surface of red blood cells and lead to their lysis.

Chapter 2: What are the common mechanisms of disease?

Untreated

84

What are the common mechanisms of disease?

wall), principally affecting the kidney (glomerulonephritis), skin and joints. Nucleoproteins are common sources of endogenous antigens, e.g. in systemic lupus erythematosus (SLE). An Arthus-type reaction is encountered clinically in the lung after an exogenous antigen is inhaled and precipitates locally within the alveolar walls. The inhaled antigens are usually animal or plant proteins and often associated with specific occupations. The resulting damage to lung alveoli, with repeated episodes of inflammation and scarring, leads to a restrictive form of lung disease called extrinsic allergic alveolitis (Fig. 2.34). Of course, antibody binding to antigen is a normal defence mechanism, so why does it also cause disease? Pathogenic immune complexes are more likely to form when there is antigen excess and the complexes are small and harder to remove from the circulation by phagocytosis. It is also affected by the valency of the antigen, avidity of the antibody and charge of the complex. (a)

TYPE IV: CELL-MEDIATED HYPERSENSITIVITY

Unlike the other three forms of hypersensitivity, all of which involve antibody, type IV involves T lymphocytes (Table 2.9). This takes two forms: 1 CD4+ T-cell-related, cytokine-mediated inflammation 2 CD8+ T-cell-related, direct-cell cytotoxicity. The details of how CD4 (helper) cells and CD8 (suppressor/cytotoxic) cells recognise and respond to antigen is covered on page 197. Naive CD4 cells can differentiate into T-helper 1 (Th1) cells and Th17 cells. Th1 cells secrete interferon  (IFN-) which is a strong activator of macrophages. Th17 cells secrete IL-17 and other cytokines that recruit and stimulate neutrophils. Typically the response (delayed-type hypersensitivity) occurs over several (c)

Complement Second exposure: soluble ag swamped by circulating ab, forming ag-ab complexes in capillary wall. FclgG activates complement and neutrophils

Part 1: Disease, health and medicine

First exposure: inhalation of protein e.g. spores in mouldy hay (b)

cap

cap

(d)

Alveolus ab ag

Release of inflammatory mediators causes tissue damage. Increased vascular permeability allows IgG to seep into alveolar wall; ag-ab complexes cause local tissue damage, culiminating in fibrosis and restrictive airways disease

Figure 2.34 Type III hypersensitivity, e.g. farmer’s lung (an example of extrinsic allergic alveolitis).

Hypersensitivity reactions

Table 2.9 T-cell-mediated type IV reactions CD4 T cell

Rheumatoid arthritis Multiple sclerosis Inflammatory bowel disease Contact sensitivity

CD8 T cell

Viral infection Transplant rejection Tumour cell killing

(a)

85

hours and days, recruits and activates other T cells and macrophages, and produces local tissue damage and granuloma formation. It is the basis of the Mantoux test (tuberculin test, Fig. 2.35), used to see whether a person has some T-cell immunity to tuberculosis. It involves injecting a small amount of a (non-infectious) purified protein derivative of M. tuberculosis into the skin and observing whether a localised red induration occurs over the next 48 hours; T cells and macrophages mount a type IV reaction in a sensitised person. This test is not 100% sure, because some patients with overwhelming tuberculous infection are anergic and show no reaction.

(c)

Soution of tuberculoprotein purified protein deri vative (PPD) injected into skin of previously sensitized patient

12–24 hours later: erythema at injection site

(d)

(b)

L PPD

N

Tm

G

APC

M E

PPD antigen presented by antigen presenting cell (APC) to CD4 memory T-cell (Tm). Activated Tm cells undergo clonal expansion and secrete lymphokines and macrophage chemotactic factors

Cellular responce includes recruited lymphocytes (L), and macrophages (M). Tissue damage and necrosis (N) occurs due to release of inflamatory mediators. Macrophages may become epithelioid cells (E) which aggregate to form granulomata (G)

Figure 2.35 Type IV hypersensitivity: Mantoux test reaction.

Chapter 2: What are the common mechanisms of disease?

PPD

86

What are the common mechanisms of disease?

Part 1: Disease, health and medicine

Cytotoxic (CD8 cell-mediated) reactions involve direct killing of target cells through the perforin granzyme system, which are granules contained in cytotoxic cells that are released on activation. Perforin binds to the target cell membrane, allowing the entry of granzymes that activate cellular caspases. This induces apoptotic death of the target cells (see page 62).

In the first part of this book, we considered the interrelationships of disease, health and medicine, and then looked at the different categories of causes of disease and the major mechanisms that operate. We have finished this section with the topics of infection and inflammation, which leads us nicely to the second part of the book, which explores the ways in which we defend against disease and how the body can repair itself.

PArT 2 DEFENCE AGAINST DISEASE

Introduction: The role of epidemiology in disease

89

Chapter 3: The body’s response to infection

95

Chapter 4: The acute inflammatory response

109

Chapter 5: Healing and repair, chronic and granulomatous inflammation

147

Chapter 6: Chronic inflammation and the adaptive immune response

178

87

INTroDuCTIoN In Part 1 we looked at the causes and mechanisms of disease. Part 2 examines how the body deals with disease and the way in which the body responds to noxious agents, be they from outside the body (exogenous), such as infectious organisms, toxins or traumatic injury, or arising within the body (endogenous), as in ischaemic damage, autoinflammatory disorders, metabolic stress or mechanical stress. The body’s response may be appropriate to the degree of tissue damage sustained, in which case restoration of normal function is the expected outcome. Sometimes there is a problem with the normal processes – an inadequate inflammatory response may lead to overwhelming sepsis, or an exaggerated reaction may cause extensive tissue damage, scarring, contractures, critical illness syndromes or death. Even cancers can be linked to inflammation, either directly due to organisms that modify and mutate DNA, or indirectly through mechanisms that provoke the release of reactive oxygen species by inflammatory cells, causing DNA damage. There is so much that could go wrong with the finely balanced mechanisms that regulate the innate and acquired immune responses to infection that it is a wonder that problems are not encountered more frequently. In the coming chapters we discuss the details of the interactions of the inflammatory cells, host cells and the communication role played by mediators and cell receptors and ligand molecules. Inherited defects in neutrophil killing mechanisms, acquired deficiencies of key regulator cells such as CD4 T-helper lymphocytes in HIV/AIDS, or the body turning on itself to produce autoimmune diseases are just some of the potential problems that may be encountered in immune function. First, as an introduction, consider the fascinating story of AIDS. It demonstrates a complex interplay

between an infectious agent and the body’s normal defence mechanisms. Its origins, epidemiology and modes of transmission needed unravelling to reduce its spread. Investigation of its biology has resulted in major improvements for treatment and survival. HIV is a recently discovered infective disease that exemplifies the role of CD4+ cells in the defence against disease. HIV/AIDS, first recognised in the early 1980s, is now the largest infective cause of death in the world. This virus infects and destroys the CD4+ cells that are at the heart of the body’s adaptive immune system. CD4+ helper T lymphocytes (Th cells) recognise microbial antigens presented to them by antigen-presenting cells (APCs). They also stimulate effector cells, the CD8+ T-cytotoxic lymphocytes (Tc cells) and B cells, which produce antibodies. Both the CD8+ Tc cells and the antibodies are specifically targeted to attack the same antigen recognised by the CD4+ Th cell. Not only this, but the generation of memory Tc and B cells is also stimulated by Th cells. By studying how HIV/AIDS evolves, and how the body responds, we can understand much of the way the immune system works. CD4 receptors are found on cells other than Th cells.

Key facts Cells with CD4 receptors are vulnerable to infection by HIV. CD4 is a glycoprotein receptor found on Th cells, dendritic cells, macrophages and monocytes (including microglial cells, the macrophages of the central nervous system). However, the virus must bind a co-receptor, such as CCr5, to gain entry to the cell.

Introduction: The role of epidemiology in disease

THE roLE oF EPIDEMIoLoGY IN DISEASE

89

90

Introduction: The role of epidemiology in disease

Part 2: Defence against disease

WHErE AND WHEN DID AIDS BEGIN?

Almost certainly, HIV originated from a mutated simian virus (SIV) found in certain monkeys and chimpanzees, which at some point made the mutational leap to become infective to humans. When and in what parts of the world this occurred has been debated – retrospective studies show that it may have been present in humans in the 1950s or even earlier. The earliest confirmed HIV infection was in preserved blood from a patient who died in the Congo in 1959. First reports in the early 1980s of clusters of Pneumocystis carinii (now reclassified as Pneumocystis jirovecii) pneumonia (PCP or PJP) in gay men in California and elsewhere in the USA were followed closely by a number of outbreaks of aggressive Kaposi’s sarcoma in several parts of the world, and curious immunodeficiency diseases were described in many parts of Africa, the USA, Haiti and elsewhere. Patients developed chronic diarrhoea, lymphadenopathy, tuberculosis and candidiasis. ‘Slim disease’ was reported in Ugandan men and women, who became mysteriously wasted (now attributed to HIV lipodystrophy), and the terms ‘gay bowel syndrome’ and then GRID (gay-related immunodeficiency disease) were coined in the western world. The disease became a scourge in both sexes in Africa and the East, whereas in western Europe the disease was for many years largely restricted to promiscuous MSM (men who have sex with men). Patients with haemophilia who received pooled blood products were early victims. A combination of promiscuity, unprotected sexual intercourse, intravenous drug use, prostitution and transmission in jails has meant that the disease now constitutes a global pandemic in both sexes. Currently the disease is a major problem in parts of eastern Europe. The term ‘autoimmune deficiency syndrome’ (AIDS) was coined in 1981. Luc Montagnier, in France in 1983, and Robert Gallo, in the USA in 1984, almost simultaneously isolated what we now know as the human immunodeficiency virus (HIV). In 1984, Dalgleish et al. demonstrated that the virus entered cells after attaching to the CD4 receptor. Quite early on in the investigation into these new immunodeficiency diseases, later collectively recognised as AIDS, a link was made between genital herpes simplex virus (HSV type II) and an increased risk of infection with the virus later shown to be HIV. HSVinduced ulceration deprives the mucosa of its protective

Key fact The 2008 Nobel Prize for medicine was awarded jointly to Dr Luc Montagnier and Francoise BarreSinoussi, a mere 25 years after discovering HIV, and to Harald zur Hausen who discovered in 1983–4 that human papillomavirus (HPV) types 16 and 18 caused cervical cancer and in 2006 developed an HPV vaccine.

epithelial barrier, increasing the risk of transmission of HIV. Added to that is the herpes simplex stimulus to CD4+ cell proliferation, which increases HIV viral synthesis by the infected CD4+ cells. The link with homosexual activity reflects the reduced barrier to infection offered by glandular surface epithelium in the

Key fact Initial concerns that, because mosquitoes can spread malaria, HIV might also be transmitted in this way have been allayed. Malarial parasites utilise the mosquito as part of their life cycle. They migrate to the mosquito’s salivary glands and multiply, to be squirted into the next victim, because mosquito saliva contains antithrombotic substances, allowing free blood flow. Any HIV inadvertently sucked out of a positive host dies in the mosquito’s gut. Insufficient contaminated blood can be transferred by mosquito mouthparts to infect by the equivalent of a mozzie needlestick injury!

Key fact Needlestick injury to health workers The risk of transmission of HIV by needlestick injury involving minimal amounts of blood is extremely low – only around 1/300 people exposed to the very highest-risk patients contract HIV if no subsequent protective measures are taken. The wearing of protective gloves at the time of injury drops the risk by 80% and, if the wound site is subjected to 10 minutes of prolonged washing with soap and running water, the risk is virtually nil. Postexposure prophylaxis with antiretroviral drugs, instigated immediately, also brings the risk of infection to virtually zero.

rectum, which is only one-cell thick, compared with the stratified squamous epithelium lining the vagina and ectocervix. HIV can be transmitted sexually, via infected blood (usually from shared needles) and in some body secretions, such as breast milk, but not by saliva, sweat or tears. HIV does not usually cross the placenta. However, the risk of transplacental infection is increased in patients with either recently acquired or very advanced HIV with high blood viral levels, or if there is an intrauterine infection or the mother is malnourished. If the mother has malaria, other sexually transmitted infection, urinary tract infection or respiratory infection, such as tuberculosis (TB), the risk is also increased. Antiretroviral treatment given to the mother greatly reduces this risk, as does the correction of any of the infectious or nutritional problems mentioned above. Infection of an infant more usually occurs at the time of birth or via breast milk (perinatal vertical transmission), if not protected by antiretroviral drug therapy. Around 25% of breast-fed babies are likely to catch HIV in this way, whereas if antiretroviral therapy is instigated the risk of transmission is less than 2%. HoW DoES HIV CAuSE DISEASE?

Enveloped viruses, such as HIV or herpes simplex, can enter cells by binding a receptor on the host cell (Fig. 1). (Non-enveloped viruses are either taken up by receptor-mediated endocytosis or injected into the host cell by phages, see page 101.) In the case of HIV,

91

its gp120 surface antigen binds the CD4 receptor on Th cells and some macrophages, but the virus also needs to bind a co-receptor in order to achieve its goal. This is usually the CCR5 receptor, which is attached to a G protein within the cell. It is a multifunctional receptor that normally recognises chemokines and is central to several inflammatory responses. When HIV binds, it melds and fuses its envelope with the host cell’s phospholipid bilayer membrane and uncoats its viral RNA, which enters the cytosol. The viral proteins and genome, once released, are able to activate the transcription of new viral particles. HIV, a retrovirus, can ‘reverse transcribe’ its RNA into DNA, which enters the host cell nucleus and is then replicated, forming new viral particles, which bud out of the cells. After the initial infection, which may be silent or produce a rash and flu-like symptoms that last approximately 2 weeks, the virus may be unobtrusive and cause few if any problems for many years. During this period, lymph node enlargement may occur at many sites. The virus gradually causes the death of infected T cells, thought to be largely achieved by the T cell’s own pattern recognition receptors (PRRs), which assemble into an inflammasome, activating caspases and causing the death of the host T cell (see page 114). It may take up to 8 years before the number of circulating CD4 T cells in the blood falls below the critical level of 500/μL. At this point, the body finds itself without the key coordinating cells of the adaptive immune response (see page 188) and starts to be prone to unusual infections. CD4+ Th cells are central to adaptive immunity (see page 189). They both receive signals from APCs, which alert the immune system to the presence of invading microbes, and give signals for the activation and proliferation of effector CD8+ Tc cells and B cells – the latter become plasma cells and secrete antibodies. These can cause the destruction of free microbes or infected host cells displaying parts of the microbes on their surfaces. Without CD4+ Th cells, there is no generation of memory T and B cells, and so no memory of past infection. Without CD4+ cells, the body is left to rely on the innate immune system. This is geared to recognise common bacterial and fungal pathogens because they bear ‘PAMPs’ (pathogen-associated molecular patterns, so called because these were first found in pathogenic organisms). PAMPs are actually found on most bacteria and many fungi, not all of which are pathogenic. Certain stress-related chemicals are

Introduction: The role of epidemiology in disease

Introduction: The role of epidemiology in disease

Introduction: The role of epidemiology in disease

92

HIV encounters aCD4+ helper T lymphocyte (Th cell)

CCR5 CD4 receptor (a)

HIV RNA

CD4

CCR5 receptor

CD4

CCR5 receptor

CD4 and CCR5 bind their complementary receptors. This prompts fusion between HIV and Th cell membranes.

HIV RNA is released into Th cell, along with reverse transcriptase (RT) enzyme, which translates RNA into DNA. This moves to the nucleus.

(b)

CD4

CCR5 receptor

New viral proteins are generated by gene transcription in the Th cell nucleus. (c) The Th cell cytoplasmic apparatus assembles new viral particles, which bud out of the Th cell, using cell membrane to make new envelope.

(d)

CCR5 CD4 receptor

Part 2: Defence against disease

New HIV seeks further CD4+ cells to infect.

(e)

1. New HIV virus is produced and infects other cells.

2. HIV viral fragments are presented to cytotoxic T cells and B cells by MHC2 receptors and stimulate an immune response. This is useful for diagnosis (‘HIV+’) but, because of constant viral mutation, is not useful for immunity against HIV.

3. Infection by virus is recognised by Th cell PRR, due to viral PAMP. This initiates cell autodestruction and leads to depletion of CD4+ Th cells. Any virus inside the cell is also destroyed.

Introduction: The role of epidemiology in disease

meningitis (Cryptococcus neoformans). These organisms are widely prevalent in the air, soil and water, and do not generally trouble people with a working immune system. Fascinatingly there are also AIDS-defining malignancies, such as intracerebral lymphoma and Kaposi’s sarcoma (both secondary to infection with HHV8), which demonstrate both the role immunosurveillance plays in guarding against tumour formation and spread, and the fact that some tumours are linked to viruses. NATurAL IMMuNITY To HIV/AIDS

Curiously, some people have natural immunity against HIV, those with Δ32 mutations in their CCR5 receptor genes (Fig. 2). This mutation is seen in around 16% of people in northern Europe, but is rare in sub-Saharan Africa, Asia, and North and South America. A Δ32 mutation is effective only against strains of HIV that use CCR5 as a co-receptor to initiate entry to the CD4 cell. However, most HIV infections do use CCR5. Δ32 prevents the HIV gp120 antigen from binding the mutated CCR5 receptor on the host surface molecule. Berlin has become synonymous with good news for two HIV patients! The first was an anonymous man, infected with HIV in 1995, who in 1998 was successfully treated with a combination of antiretroviral drugs and chemotherapeutic agents, and is still well at the time of writing (2015). Timothy Ray Brown, the ‘second Berlin patient’, was also found to be infected with HIV in 1995. To compound his luck he developed acute myeloid leukaemia in

also PAMPs – see Chapter 4, Table 4.1). The innate immune response is generally effective against all except encapsulated microbes, but of little help against viruses and more complex organisms.

e

Thus it is that HIV infection becomes the acquired immune deficiency syndrome (AIDS), characterised by either a CD4 count > POP, so fluid leaves vessel along whole length with no reabsorption. No protein loss, so this is a transudate

Increased vascular permeability Protein leaks out of vessel, so that there is no osmotic pressure difference between plasma and tissue (POP = 0), therefore fluid leaves vessel. Fluid contains protein, so this is an exudate

Increased hydrostatic pressure & increased vascular permeability POP = 0, HP >>> POP, so large loss of protein-rich fluid to tissues

Part 2: Defence against disease

Figure 4.10 Factors affecting the movement of fluid across vessels.

Ink is injected intravenously

Ink escapes in organs with discontinuous endothelium

...or if the endothelial cells are damaged

Figure 4.11 Vascular endothelium is continuous everywhere except the liver and spleen; here the endothelium is fenestrated. As a result of this, an intravenous injection of Indian ink will lead to the accumulation of ink particles in the liver and spleen. However, if endothelium elsewhere is damaged, e.g. by heat, gaps appear between the endothelial cells and ink particles can leak out of the circulation at these sites.

What is the pathophysiology behind the clinical signs of inflammation?

Now that we understand the normal physiological pressures that govern the way plasma (composed largely of water) moves between the vascular and extravascular compartments – note that we have not considered the intracellular compartment in this discussion – we must start thinking about what can go wrong – in other words the pathology of fluid balance. This is an appropriate time to introduce a number of new words. An exudate is fluid within the extravascular spaces that is rich in protein and hence has a specific gravity >1.020. On the other hand, a transudate has a low protein content and specific gravity 3.5 g/day • Oedema (due to hypoalbuminaemia) – especially marked around eyes • Hyperlipidaemia – the liver synthesizes extra liporoteins as a non-specific response to decreased plasma oncotic pressure (due to proteinuria), especially LDL. Risk associated with nephrotic syndrome: • Hypercoaguability of the blood, with thrombosis of renal or deep veins. • Atherosclerosis (high LDL and cholesterol levels) • Infection (loss of antibodies in the gamma globulin fraction of plasma) Figure 34.3 The salient features of nephritic and nephrotic syndrome.

has crescents, you need to act rapidly and typically aggressively. But that’s what they are, a particular response to injury that indicates a serious threat to renal function.The next thing to discuss is how these pathological processes disrupt renal function and in what ways.’ Dr Southend paused to consider how to move onto this new topic. ‘Okay, how do you think the podocytes feel about all this nonsense breaking out in the glomerulus?’ ‘They’re not going to be too happy,’ replied Sarah, following Dr Southend’s colloquial lead. ‘Exactly. They tend to get cheesed off and can’t do their job properly. In one sort of glomerulonephritis, minimal change disease, there’s virtually nothing to see, hence the name, but the podocytes don’t work properly. If you look with electron microscopy, their foot processes might be a little further apart than they should and it’s said that they lose their negative charge. Now, if the podocytes don’t work properly and their job is to regulate what can and can’t be filtered, what do you think happens?’ ‘The glomerulus leaks.’ ‘Yes. Add in direct damage to capillaries and the mesangium in more aggressive diseases and you can see that things can really get leaky. So what leaks out?’

Nephritic syndrome (‘I’) • Haematuria – can give urine a ‘smoky’ or ‘cola’ colour • Oliguria • Hypertension • Less than 2g proteinurea per day • Slight oedema

133

134

Part 3 Investigations

‘Proteins and blood.’ ‘Which brings us onto the nephrotic and nephritic syndromes.You can probably guess the next question.’ ‘What’s the difference?’ ‘Right.’ ‘Nephrotic syndrome occurs when you lose protein,’ Sarah paused to recall the precise number, ‘At least 3.5 g per day. In nephritic you have haematuria and hypertension but don’t lose much protein.’ ‘Basically. People dispute the precise figure for the amount of protein loss, but most settle around 3.5 g. Others take a more pragmatic approach and say that the protein loss is of a sufficient level to cause oedema. Which disease is nephritic syndrome mainly associated with?’ ‘That’s, erm, post-streptococcal glomerulonephritis,’ stated Sarah. ‘Yes, it’s the weird cross-reactivity thing. If you want to look really smart and give the subtypes of streptococcus, just give the square numbers up to 49, but miss out 9, 16 and 36.’ Sarah nodded, aware that examiners would be impressed by this trivia. ‘Somebody was probably having a laugh with the terminology,’ resumed Dr Southend. ‘Nephrotic and nephritic look like they’re designed to cause confusion, but you can simplify it down to most types of glomerulonephritis giving nephrotic syndrome and in any case, the patient’s presentation defines whether its nephrotic or nephritic, not the type of glomerulonephritis. I suppose if you want to remember it, nephrotic syndrome gives you oedema which starts with an o and makes you swell up like an o and o is in nephrotic but not nephritic. I’ve already given the game away with oedema in nephrotic syndrome but what else happens?’ ‘You can get pleural effusions and ascites,’ replied Sarah, continuing with the theme of a reduced plasma protein pressure. ‘Good.What else? What might the liver do to try to cope with the loss of proteins?

‘I’m not sure.’ ‘Don’t worry. It pumps out lipoproteins to try to hold things together, so the patient gets hyperlipidaemia. There’s also a procoagulant tendency, possibly because you tend to get that in inflammation generally, combined with the liver throwing fibrinogen into the fray to try to keep the oncotic pressure up, as well as a loss of anticoagulant proteins through the damaged glomeruli. Albumin can oppose platelet aggregation and the kidney is losing albumin at a rate of knots.All of this means that the frequency of venous thrombosis, including renal vein thrombosis can really be very significant.’ ‘Can I ask about the terms diffuse and focal and segmental, please?’ ‘Sure. Diffuse means that all glomeruli are affected, focal means that only some are and segmental indicates that within an affected glomerulus, only part of it is involved. Did you have a particular disease in mind?’ ‘Focal segmental glomerulonephritis was one I’d heard about.’ ‘What had you heard?’ ‘That you get nephrotic syndrome and it comes back in transplants.’ ‘That’s the key concept and is probably the place to finish before I overload you. Although the classification is often confusing, the importance of the different types of glomerulonephritis is that they have different prognoses, different rates of progression and different recurrence rates in transplants.The different secondary causes of glomerulonephritis tend to favour certain patterns of glomerulonephritis, although ultimately you need to keep an open mind and investigate widely to make sure that there isn’t an underlying cause that is actually eminently treatable before the kidney is permanently wrecked. I think I’d better leave it there. That’s kind of an overview for you and you can use it to hang the details on.’

PART 4

TREATMENT

Case 35: Novel chemotherapy

136

Case 39: Keeping the pump pumping

151

Case 36: Supporting haematopoiesis

139

Case 40: Radiologist or surgeon?

155

Case 37: A prophylactic resection

143

Case 41: Organ transplantation

159

Case 38: A chronic autoimmune disease 147

CASE 35

CASE 35 NOVEL CHEMOTHERAPY A 65-year-old man presents to his GP with a four-month history of tiredness and a sensation of fullness in the left side of his abdomen. Examination is unremarkable other than splenomegaly which extends towards the right iliac fossa. What are the causes of massive splenomegaly?

Answer 1 This is a popular exam question as there are only a handful of typical causes of massive splenomegaly: ● ● ● ●

Question 2 The GP requests a full blood count (see Investigations box below). Which of the diagnoses is/are most likely?

Question 1



Assorted other causes of lesser degrees of splenomegaly exist, but tend not to produce a massive degree of enlargement.

myelofibrosis chronic myeloid leukaemia leishmaniasis chronic malaria gaucher’s disease.

Investigations Hb WCC Platelets MCV MHC MCHC

10.9 g/dL 65 ⫻ 109/L (neutrophils 59 ⫻ 109/L) 173 ⫻ 109/L 84 fL 30.8 pg 33.1 g/dL

Signal molecule, such as a growth factor

Normal Cell membrane

Transmembrane receptor

Abl tyrosine kinase protein ATP Inactive target effector molecule

Tyrosine kinase site of abl protein

ATP

Binding of the signal molecule to the transmembrane receptor activates the receptor. This in turn activates the abl tyrosine kinase and it phosphorylates a tyrosine amino acid on the target effector protein

Phosphorylation of the target effector protein activates it, P allowing it to relay the message from the signal molecule into the cell. In bone marrow stem cells, this message triggers cell division

CML

bcr-abl chimeric protein

ATP The bcr-abl fusion protein P has an abnormally high level of tyrosine kinase activity. This results in excessive levels of phosphorylated, activated substrate and an inappropriate stimulus to the stem cell to divide

Imatinib binds to the tyrosine kinase site of the bcr-abl fusion protein and blocks it, preventing the bcr-abl fusion protein from activating the effector molecule

Figure 35.1 The action of imatinib, a ‘designer drug’ which was specifically manufactured to block a tyrosine kinase receptor expressed in chronic myeloid leukaemia.The same receptor has since been found to be expressed in other tumours, which are also responsive to this drug (see Table 35.1).

Case 35: Novel chemotherapy Prevailing H1

Answer 2 The markedly elevated white cell count (WCC) makes chronic myeloid leukaemia the most likely diagnosis of the five causes of massive splenomegaly.The early stages of myelofibrosis can also be associated with leukocytosis, although not usually of this degree. Question 3 The patient is referred to a haematologist and a bone marrow trephine is performed.The trephine is hypercellular and shows trilinear haematopoiesis with abundant representatives of all three cell lineages, although myelopoietic elements dominate.The Philadelphia chromosome is detected.What is the diagnosis?

Answer 3 Chronic myeloid leukaemia (CML). Note that the presence or absence of the Philadelphia chromosome is a key diagnostic criterion in the myeloproliferative disorders. Question 4 What is the Philadelphia chromosome?

Answer 4 The Philadelphia chromosome is an abnormal form of chromosome 22 that is the result of a translocation between the long arm of chromosome 9 and the long arm of chromosome 22. This translocation fuses the coding region for the abl gene of chromosome 9 with the bcr region of chromosome 22. Question 5 The abl gene codes a tyrosine kinase.What alterations in function could happen to a tyrosine kinase protein as the result of a mutation (either a translocation or other type of mutation)?

Answer 5 Mutation of a tyrosine kinase could result in a complete loss of function, reduced function or overactivity. Question 6 What is the basic role of tyrosine kinases?

Answer 6 Tyrosine kinases form part of a variety of cell surface receptor systems. Binding of the appropriate ligand to the

receptor activates the tyrosine kinase, resulting in phosphorylation of a target protein which in turn relays a signal to the cell. In some tyrosine kinase receptor systems, this signal is one that contributes to the regulation of cell division. Question 7 The bcr-abl fusion gene encodes a tyrosine kinase that has a much greater activity than normal. How could this explain the myeloproliferative process of CML?

Answer 7 CML is a clonal myeloproliferative bone marrow stem cell disorder in which there is an increase in the number of neutrophils and their precursors in the bone marrow and peripheral blood (the stem cell also generates cells of the erythroid and megakaryocyte series). If a bone marrow stem cell acquires the Philadelphia chromosome, the greatly enhanced tyrosine kinase activity of the bcr-abl fusion protein provides an excessive drive to the cell to divide. However, at this stage, there is no blockage to the maturation pathways of the daughter cells, so CML features an excess of mature forms (compared with acute myeloid leukaemia (AML) and acute lymphoblastic leukaemia (ALL) in which significant maturation is not encountered). Question 8 What possible avenue of treatment does the role of the bcr-abl fusion gene tyrosine kinase suggest?

Answer 8 In view of the central role of the abnormal tyrosine kinase in CML, therapy that targeted this abnormal tyrosine kinase could block the disregulated cell division and control the disease. Until relatively recently, this specific treatment was not available, but there is now available a monoclonal antibody (imatinib, trade name Glivec) which is directed against the abnormal tyrosine kinase and neutralizes its actions. Imatinib is highly effective in returning the blood count to normal and can render the bone marrow negative for the Philadelphia chromosome (the mutated stem cells no longer have a growth advantage over normal stem cells). Some other examples of targeted cancer therapies are listed in Table 35.1. Question 9 Under what circumstances may the effectiveness of imatinib become impaired?

137

138

Part 4 Treatment

Table 35.1 Examples of targeted cancer therapies Agent Monoclonal antibodies Bevacizumab (Avastin) Cetuximab (Erbitux) Trastuzumab (Herceptin)a Rituximab (Mabthera) Small molecule inhibitors Imatinib (Glivec)b

Erlotinib (Tarceva) Bortezomib (Velcade)

Target

Process targeted

Cancer type

Vascular endotheial growth factor Epidermal growth factor receptor HER2 receptor CD20 receptor

Angiogenesis Growth factor signalling Growth factor signalling Induces cell death

Colorectal (metastatic) Colorectal Breast B cell lymphoma

Bcl-Abl fusion protein; Kit

Growth factor signalling

Chronic myeloid leukaemia; gastrointestinal stromal tumours Epidermal growth factor receptor Growth factor signalling Non-small cell lung cancer Proteosome Multiple processes affected Multiple myeloma

a

The monoclonal antibody therapy Herceptin (trastuzumab) targets the cell surface HER2 growth factor receptor (also known as c-ErbB2). HER2 is overexpressed in around a quarter of all breast cancers. Herceptin has been recommended for treating metastatic breast cancer for some time, in combination with chemotherapy. b Glivec (imatinib mesylate) is a tyrosine kinase inhibitor that prevents activation of the Bcr-Abl protein, and, as a result, inhibits cell proliferation and promotes apoptosis. In England and Wales it is now recommended as the first-line treatment for adults with chronic phase BCR-ABL-positive CML and chemoresistant gastrointestinal stromal tumours, where it targets the Kit receptor. Adapted from CRC UK, Genes and Cancer. http://info.cancerresearchuk.org/cancerstats/causes/genes/diagnosisandtreatment/?a⫽5441

Answer 9 CML has the ability to transform to an acute leukaemia, either myeloblastic (70–80 per cent) or lymphoblastic (20–30 per cent) and this is a common cause of death in CML. The additional mutations accumulated by the acute leukaemic cells negate some or all of their dependence on the drive provided by the bcr-abl product. Therefore, while imatinib may still be employed, the appropriate chemotherapy for ALL or AML must be added. Even then, the prognosis of transformed CML remains poor and is typically worse than that for primary ALL or AML. Question 10 Why could a patient with CML require allopurinol?

Figure 35.2 Expansion of the bone marrow cavity by a homogeneous leukaemic infiltrate, seen at higher magnification (inset). In an adult, only the shaft of the femur contains red bone marrow. Here there is featureless fleshy tumour replacing and expanding the marrow cavity, including the femoral head and neck.

Answer 10 Allopurinol is employed in the treatment of gout. Patients with CML have a significant increase in the turnover of nucleic acids and this results in elevated levels of uric acid. In some patients, this will precipitate gout.

CASE 36 SUPPORTING HAEMATOPOIESIS A 23-year-old woman presents to her GP with a fourweek history of increasing lethargy and easy bruising. She has also remarked that her last period, which was two weeks ago, was unusually heavy. On examination, the woman is pale and has scattered bruises.There are no focal abnormalities in the cardiovascular, respiratory, abdominal or neurological systems.The GP requests some initial blood tests.The laboratory at the local hospital telephones these through shortly after they are confirmed, as well as faxing a printed copy of the finalized report.

Investigations Hb WCC Neutrophils Platelets MCV MCH MCHC INR APTTR

5.2 g/dL 1.3 ⫻ 109/L 0.3 ⫻ 109/L 15 ⫻ 109/L 107 fL 30.1 pg 33.5 g/dL 1.1 1.0

Additional details in this case are the neutropenia and a macrocytic anaemia. Question 2 Where are erythrocytes, granulocytes and platelets produced?

Answer 2 In the bone marrow. The bone marrow synthesizes these three cellular elements of the peripheral blood. This is referred to as trilineage haematopoiesis and comprises erythropoiesis (red cells), myelopoiesis (granulocytes) and megakaryopoiesis (platelets via megakaryocytes). Question 3 What conditions could cause the bone marrow to fail?

Answer 3 The causes of bone marrow failure can be divided into the categories of a loss of the normal contents of the bone marrow, replacement of the bone marrow by other tissue or dysfunction of the bone marrow: ●

Question 1 What abnormalities are present?



Answer 1 The patient has pancytopenia.This term indicates the combination of anaemia, leukopenia and thrombocytopenia.

Loss of normal contents – aplastic anaemia Replacement by other tissue – haematological malignancy (leukaemia, myeloma, sometimes lymphoma) – secondary tumour, usually a carcinoma – myelofibrosis (replacement by fibrous tissue)

Megakaryocyte fragments into multiple platelets

Erythropoiesis

Megakaryopoiesis

Myelopoiesis

Neutrophil

Eosinophil

Mast cell

Monocyte/ macrophage

Figure 36.1 Summary of the normal pathways involved in haematopoiesis, which is the generation of blood cells by the bone marrow.The bone marrow stem cell gives rise to specialized myeloid, lymphoid and erythroid precursor cells, which differentiate to form mature cells under the influence of colonystimulating factors and local factors, such as cytokines or erythropoietin.The bone marrow stromal cells can secrete local hormones and therefore are not the simple ‘packing material’ they were once considered to be. Platelets are formed from megakaryocyte cytoplasm and have no nuclear material. Erythrocytes normally lose their nuclear material prior to entering the blood.

140

Part 4 Treatment



– amyloidosis – Gaucher’s disease Ineffective haematopoiesis – myelodysplasia – megaloblastic anaemia – inherited disorders.

Question 6 A diagnosis of aplastic anaemia is made.What is this condition and what are its causes?

Question 4 What other disorders could cause pancytopenia?

Answer 4 Even if the bone marrow is working normally, cytopenias in the peripheral blood can result from either destruction of the blood cells after they are synthesized or their pooling (sequestration) somewhere. ●



Peripheral destruction of blood cells – autoimmune – paroxysmal nocturnal haemoglobinuria Peripheral sequestration – hypersplenism.



● ● ● ● ● ● ● ●

Congenital – various disorders, including Fanconi anaemia Idiopathic Viral Drugs – e.g. gold, chloramphenicol (mainly historical) Chemotherapy Toxic chemicals – e.g. benzene Radiation Associated with haematological malignancy Associated with paroxysmal nocturnal haemoglobinuria.

Question 7

Question 5 The GP contacts the patient and the local haematology team and arranges urgent admission that day for investigation and treatment of the patient’s pancytopenia.Assorted investigations are undertaken, but the key procedure in this patient’s case is a bone marrow trephine and aspirate. Figure 36.2 shows a normal trephine and one that could have come from this patient.What is the striking difference?

Answer 5 The patient’s trephine is effectively devoid of haematopoietic cells and does not show replacement by an abnormal infiltrate.

(a)

Answer 6 Aplastic anaemia is a disease in which there is a chronic pancytopenia due to hypoplasia of the haematopoietic bone marrow.There are assorted causes:

(b)

What complications does this woman face?

Answer 7 The patient has severe anaemia with the consequent problems of impaired oxygen carriage and delivery, cardiovascular strain due to a hyperdynamic circulation and marked vulnerability to any further haemoglobin loss. The patient’s menorrhagia and easy bruising means that her thrombocytopenia is demonstrably of a sufficient degree to compromise blood clotting.This bleeding tendency could exacerbate the anaemia.There is also a danger of a spontaneous intracranial haemorrhage.

Figure 36.2 (a) Normal bone marrow.The bone trabeculae (black arrows) enclose marrow spaces in which haematopoietic tissue (red arrow) and mature adipose tissue (blue arrow) are present. The proportion occupied by the haematopoietic tissue decreases with age. (b) Bone marrow in aplastic anaemia.The bone marrow is formed almost entirely of mature adipose tissue. Haematopoietic elements are inconspicuous.

Case 36: Supporting haematopoiesis Prevailing H1

The leukopenia renders the patient susceptible to infection. The neutropenia in particular makes the patient vulnerable to bacterial infection and fungal infection, especially of the opportunistic type. It should also be remembered that if there is an underlying cause for the aplastic anaemia, this could have other specific complications beyond that of bone marrow failure. Question 8 What issues should treatment address?

Answer 8 There are two main arms to the treatment of aplastic anaemia. The bone marrow failure means that replacement of the missing cellular components of the blood is

Normal red cell

required. In parallel to this is the aim to restore normal haematopoiesis. Question 9 How can this be achieved?

Answer 9a Of the three elements of the pancytopenia, the anaemia is the easiest to address. Blood transfusions can be given as necessary and the donated red cells have a reasonable halflife within the recipient. Problems can arise after multiple transfusions because the recipient can develop numerous antibodies to minor blood group antigens, making crossmatching and locating units of blood harder. Platelet transfusion can be employed to correct the thrombocytopenia, but infused platelets have a

Biconcave disc when viewed from side

Microcytic, hypochromic anaemia, e.g. iron deficiency Macrocytic/ megaloblastic anaemia, e.g. folate/B12 deficiency Fragmented red cells, seen in marro infiltration by tumour Spherocytes, e.g. hereditary spherocytosis

Almost spherical when viewed from side

Reticulocyte – immature red cell which contains nuclear remnants. May enter peripheral blood after large haemorrhage. Howell-Jolly bodies indicate hyposplenism or absent spleen. Basophilic stippling e.g. lead poisoning.

Sickle cell anaemia

Distorted cells sickle if oxygen tension is low

Legend: some examples of normal and abnormal red blood cells

Figure 36.3 Red blood cell appearances in different types of anaemia.

141

142

Part 4 Treatment

disappointing half-life within the patient (perhaps only a few days). Platelet transfusion tends to be reserved for thrombocytopenic patients who have significant bleeding. However, if levels are very low, transfusion may be also given prophylactically. Replacing leukocytes is the most difficult. Leukocyte (buffy coat) infusion is ineffective and seldom employed. Thus, unlike red cells and platelets, direct replacement by transfusion is not an option. Instead, attempts can be made to stimulate the patient’s own myelopoiesis with injections of growth factors such as G-CSF (granulocyte colony-stimulating factor) but this can be of limited efficacy. The risk of opportunistic infection becomes especially marked when the neutrophil count falls below 0.5 ⫻ 109/L. While it may not be possible to replace/regenerate the neutrophils effectively, the dangers of opportunistic infection can be reduced by controlling the patient’s environment.This involves the use of special side rooms and reverse barrier nursing in order to minimize the chances of the patient encountering a pathogen. If a patient with neutropenia does become febrile, or develop an infection, prompt and aggressive antimicrobial treatment is essential because the patient’s depleted immune response means that the infection can become overwhelming and fatal within hours. Answer 9b Supportive therapy cannot be continued indefinitely and it is necessary to endeavour to restore normal haematopoietic bone marrow function. This can be achieved in several ways.

If an underlying cause for the aplastic anaemia is known, this should be addressed, for example, withdrawing a causative drug. In many cases of aplastic anaemia, the destruction of the haematopoietic cells is mediated by the patient’s own immune system, specifically the lymphocyte component. Therefore, despite the existence of the neutropenia, inducing immunosuppression is often beneficial. This is in the form of antilymphocyte globulin (ALG) or antithymocyte globulin (ATG) in order to target the lymphocytes specifically. Androgens (anabolic steroids) have been found to be helpful in some cases. As mentioned above, growth factors such as G-CSF tend to provide supportive treatment rather than restorative. If the above treatments fail and the aplastic anaemia is severe, bone marrow transplantation can be attempted. This requires an HLA (human leukocyte antigen)matched donor. First-degree relatives tend to be the first place to look for a match. Question 10 What is the prognosis?

Answer 10 The course of aplastic anaemia can be variable. Some people make an excellent recovery. However, others have a more complex course and it must be remembered that aplastic anaemia is a serious disease that can readily kill.

CASE 37 A PROPHYLACTIC RESECTION Arthur McTaggart, a 46-year-old businessman, presents to his family doctor complaining of feeling exhausted. He becomes breathless on the slightest exertion and stops twice as he climbs the stairs to his second-floor office. The doctor notices that his mucous membranes are extremely pale and suspects that he is anaemic. On general systems enquiries, Arthur admits that he has begun to pass more wind than usual and that he has recently noted slime when he opens his bowels. He has never been aware of fresh blood per rectum or dark, tarry stools. He has not lost weight.There are no other symptoms of note. On digital rectal examination, the doctor notes that the rectal mucosa feels generally lumpy, but that there is no single discrete mass. A full blood count confirms that Arthur has a microcytic, hypochromic anaemia, most likely to be due to chronic iron loss from bleeding. In view of his gutrelated symptoms the doctor refers him to the rapid access gastrointestinal clinic at the local hospital. Mr Southern, the consultant surgeon, is astonished when on sigmoidoscopy he sees a carpet of polyps of varying size. A colonoscopy is organized for a few days’ time.This confirms that the entire colon is involved by polyposis and that there is a fungating, ulcerated cancer

in the caecum, confirmed by biopsy and histological examination. Question 1 What is the link between colonic polyps and cancer?

Answer 1 The most common colonic polyps are adenomas and hyperplastic polyps. Adenomas are dysplastic, i.e. the epithelial cells show cytological and architectural characteristics similar to those seen in malignant cells, such as a high nuclear:cytoplasmic ratio, disorganized growth and increased numbers of mitoses. Adenocarcinoma often develops within a large polyp, usually those larger than 2 cm diameter, with a villous growth pattern and high-grade dysplasia. Hyperplastic polyps do not show dysplasia and are not regarded as pre-malignant. After staging by CT scan, which shows no metastatic spread, Arthur undergoes a total colectomy.The histology report reads as follows: Arthur McTaggart, age 46 years, Consultant Mr Southern. Operation: Panproctocolectomy for familial polyposis coli with caecal adenocarcinoma.

(c)

(b)

(d)

(e)

(a) Figure 37.1 (a) Total colectomy specimen.The colon is carpeted by adenomatous polyps – tubular, tubulovillous and villous in shape – and there is an ulcerated adenocarcinoma towards the caecum (blue arrow). (b) Inset shows a slice through a polypoid region, which shows (c) the typical microscopical appearance of a tubular adenoma with low grade dysplasia.This area appears blue (because the cells show disordered maturation and have crowded nuclei). (d) Inset shows a slice taken adjacent to the ulcerated cancer in the caecum.The node and fatty tissue (red arrows) have a white appearance due to tumour infiltration. (e) Microscopy of this area shows irregular, atypical glands in the node – this is adenocarcinoma.

144

Part 4 Treatment

The 36 mm diameter ulcerated tumour identified in the caecum is a poorly differentiated adenocarcinoma which has spread to 5/42 pericolic lymph nodes and is present in a medium-sized blood vessel in the serosal fat.The apical lymph node is not involved.The retroperitoneal, peritoneal and surgical longitudinal margins are clear. The tumour has penetrated the serosa and is exposed on the peritoneal surface. TNM stage pT4, N2 MX; Dukes’ stage C1. The remainder of the colon and rectum bear innumerable adenomatous polyps between 3 and 16 mm in diameter.Two of the larger polyps in the rectum show high-grade dysplasia and the remainder show low grade dysplasia. Intervening mucosa shows the presence of unicryptal adenomas, characteristic of familial polyposis coli.

Question 2 Mr Southern explains to Arthur the prognostic implications of these findings – what are they?

Answer 2 The TNM staging system (T ⫽ tumour, N ⫽ nodes, M ⫽ metastasis) is used throughout the body, though each site has a specific set of parameters.T4 is the most advanced degree of spread through the wall, involving breach of the peritoneal covering which means that tumour cells are liable to seed across the peritoneal cavity. More than three lymph nodes are involved, pushing this into the unwelcome N2 stage. No biopsies of putative metastases have been taken, so this is MX. The presence of vascular invasion means that there is a risk that tumour will have spread to the liver via the portal vein, which drains the entire bowel except the anus. This is a Dukes’ stage C2 adenocarcinoma of colon, which is poorly differentiated (grade 3). Between 40–60 per cent of patients with this Dukes’ stage will be alive in five years; his poor TNM parameters place him at the 40 per cent end of the survival statistics. Mr Southern questions Arthur about his family. His mother died of bowel cancer when he was a child. Although he is estranged from his sister and two brothers, an aunt contacted them when she heard of Arthur’s cancer. His sister turns out to have had a malignant polyp removed from her colon last year. Arthur’s blood is subjected to genetic analysis. This shows a germline DNA mutation in the APC gene. Arthur is divorced and has not seen his only child, Jake, since he was 11 years old.

Jake, now aged 24, is contacted and invited for a consultation, at which he is advised to have a colonoscopy. This shows two small adenomatous polyps in his transverse and sigmoid colon. To his amazement, Mr Southern recommends that he undergo a total colectomy. As with his father, an ileal pouch would form a neo-rectum connected to his anus. Question 3 Jake asks Mr Southern why he should have such radical surgery.After all, he only has two small polyps and his father had hundreds.

Answer 3 Once one adenomatous polyp has grown in a patient with familial adenomatous polyposis, the risk of a colorectal cancer supervening in at least one, if not more, of the many polyps which will soon follow is almost 100 per cent.The onset of polyps, usually in the late teenage years, is an indication for a prophylactic panproctocolectomy. Question 4 ‘Why should one mutation have so much influence?’ enquires Jake.

Answer 4 The APC gene on chromosome 5 encodes a large protein with several domains, including that for beta-catenin. Familial polyposis patients inherit a ‘germline mutation’ which means that all cells in the body carry one abnormal copy of the APC gene. However APC gene mutations are some of the most common and earliest found in

(a)

(b)

Figure 37.2 Blood-borne spread from colorectal cancer is usually first seen in the liver. (a) A solitary metastasis such as that shown here may be treatable by wedge resection, giving months to years of symptom-free remission. (b) Multiple liver metastases are nonresectable but may respond to chemotherapy for months/years.

Case 37: A prophylactic resection Prevailing H1

sporadic colorectal adenocarcinomas, which are not inherited, but develop during life (‘somatic mutation’) – in this case only the tumour cells show the mutation and the rest of the cells in the body are normal. One reason for the important role of the APC gene in colon cancer development is that, among other roles, beta-catenin affects the mitotic spindle and can alter the ability of the cell to divide its number of chromosomes equally at cell division.The risk of further mutations is therefore increased, as subsequent progeny inherit excess or deficient chromosomal segments. Cancer is caused by a series of DNA mutations, each conferring on a cell line a growth or survival advantage, or a means of breaking away from the original tissue and

(1) Mutation, e.g. to chromosome 5, produces a hyperproliferative epithelial focus

Epithelium

spreading to distant sites. In a seminal paper in 1980, Vogelstein investigated mutations in a range of tumour suppressor genes and oncogenes and recognized a typical sequence of mutations in the adenoma–carcinoma sequence. An alternative pathway, due to mutation in DNA mismatch repair (MMR) enzyme genes, has been described. The MMR enzymes repair DNA replicative errors prior to the completion of mitosis. They are encoded by tumour suppressor genes so both copies must be inactivated before the effects are felt.The common methods of inactivation are methylation of CpG islands or deletion of large parts of the gene during defective cell division.

(4) Further mutation, e.g. allele loss from chromosome 18, produces a large adenoma

Submucosa m.propria (2) Further DNA alterations, e.g. ↑methylation, leads to the formation of a small adenoma

(3) Further mutation, e.g. to ras gene, produces an intermediate adenoma

(5) Another mutation, e.g. allele loss from chromosome 17, leads to the development of invasive cancer.

Staging of CRC: Dukes' A – tumour confined to bowel wall, 90%⫹5yr survival Dukes' B – tumour beyond bowel wall, lymph nodes not involved, 65%⫹5yr survival Dukes' C – tumour involves lymph nodes. Invasion of muscularized veins outside the bowel wall is associated with a poorer outlook

Figure 37.3 Traditionally accepted sequence of events involved in about 70% of colorectal cancers and in familial adenomatous polyposis (FAP). Staging utilizes the TNM and Dukes’ systems.The TNM staging is described in the text.

145

146

Part 4 Treatment

Question 5 Jake asks if there are any other inherited cancer syndromes.

Answer 5 Inherited cancer syndromes include: ●







Xeroderma pigmentosum: a rare syndrome caused by mutations in nuclear excision and repair genes. Patients with ultraviolet light-induced DNA damage cannot replace the faulty segment and develop multiple skin cancers, often in childhood. Li-Fraumeni syndrome: a predisposition to develop multiple primary cancers, including of the colon. Fifty per cent carry mutated p53, a tumour suppressor gene. BRCA1: this was the first of the familial breast cancer genes to be identified. Patients inherit a predisposition to develop breast, ovarian, prostate and colorectal cancers. It is a tumour suppressor gene, involved in repairing defects in other genes. Hereditary non-polyposis colorectal cancer (HNPCC) and familial adenomatous polyposis (FAP) as above – both rare causes of colorectal cancer (CRC).

Question 6 ‘If colorectal cancer is common, but familial cancer is rare, what else is thought to be important in colorectal cancer development?’ asks Jake.‘And can anything prevent it?’

Answer 6 Environmental and dietary agents including nitrosamines, fibre, stool transit time, smoking (i.e. reactive oxygen species), burned meats and smoked foods have all been implicated in CRC. Chronic inflammatory bowel disease: ulcerative colitis and Crohn’s disease, involving frequent cycles of gut epithelial damage and repair, increase the likelihood of spontaneous mutation. Once cell division has occurred, mutations are incorporated in the genetic blueprint for the new cell and are replicated when it divides. Previous sporadic polyps, especially if multiple, indicate that a patient is likely to have more adenomas and may develop an adenocarcinoma. Lifelong surveillance is recommended. A family history of colonic cancer increases the risk in first-degree relatives. Rectal cancer is less likely to be familial. Both genetic and epigenetic factors are probably involved. Antioxidants such as low-dose aspirin are protective. A high-fibre diet is protective, probably by reducing stool transit time and thus decreasing bowel exposure to carcinogens.









● ●

Jake has the colectomy. He and his father convalesce together and become close again. Sadly, Arthur develops metastatic colorectal cancer six years later, but he dies happy, knowing that Jake is safe.

CASE 38 A CHRONIC AUTOIMMUNE DISEASE A 39-year-old woman presents to her GP with a twomonth history of pain in her hands that is associated with swelling of her fingers and stiffness of her joints. The symptoms are worse in the morning. She has not had any previous serious illnesses. On examination, the patient has symmetrical swelling of her fingers, particularly over the joints. No other joint deformity is present. No focal neurological signs are found. The GP orders various blood tests and some X-rays. Among the blood tests is a request for an autoantibody profile.



Autoantibody profiles are often requested in rheumatological disorders, vasculitis and liver disease. What antibodies are generally included and what is their significance?

Answer 1 The precise composition of an autoantibody profile will vary from hospital to hospital and the clinical case under investigation. The general purpose of the profile is to search for autoantibodies that are present in certain disease states and which help to indicate the diagnosis. Commonly featured antibodies include the following: Antineutrophil cytoplasmic antibodies (ANCA) may be either cANCA or pANCA in type, dependent on whether the pattern of staining on immunofluorescence is coarse, granular and cytoplasmic (cANCA) or perinuclear (pANCA). Both types of ANCA suggest

(a)





Question 1





(b)

● ●



a vasculitis. Wegener’s granulomatosis is strongly associated with cANCA, which have a sensitivity of around 90 per cent and a specificity of approximately 95 per cent. A broader spectrum of conditions manifest pANCA, although they are traditionally grouped with microscopic polyarteritis. Antinuclear antibodies (ANA) are found in systemic lupus erythematosus (SLE), but are not specific. Despite the name, ANA target a variety of both nuclear and cytoplasmic antibodies. Anti-DNA are antibodies to double-stranded DNA and are considered to be specific for SLE. Anti-Sm is said to be specific for SLE. Anti-Ro (SSA) is associated with a variety of conditions, including Sjögren syndrome. Anti-La (SSB) partners anti-Ro. Anti-mitochondrial (AMA) are specific for primary biliary cirrhosis, but their utility is limited to investigating this diagnosis. Anti-liver kidney microsomes (anti-LKM), like AMA, are of use in liver diseases. There are several subtypes.

Question 2 The X-rays shown in Figure 38.1b could have come from this patient.What abnormalities are present?

Answer 2 The X-rays exhibit periarticular soft tissue swellings, periarticular osteopenia, erosion of the ulnar styloid and periarticular erosion.

Figure 38.1 (a) Early changes of rheumatoid arthritis with periarticular soft tissue swelling and periarticular osteopaenia (yellow arrow), erosion of the ulnar styloid (blue arrow) and periarticular erosions (red arrow). b) Late changes of rheumatoid arthritis with multiple carpal bone erosions and fusion (red arrow), a metacarpophalangeal joint prosthesis (blue arrow), and joint deformities.

148

Part 4 Treatment

A few days after having her X-rays, the patient notices a lump that has appeared in the olecranon region of her left elbow. She goes to see her GP who arranges for her to have an excision biopsy performed at the local day surgery unit. The histopathologist’s microscopic description of the lesion is as follows (the conclusion to the report is discussed in the answer to question 6). The specimen exhibits fibrous tissue in which there is a region of necrosis that is surrounded by a rim of fibrohistiocytic tissue in which foreign body type giant cells are abundant.

Question 3 What are the causes of a polyarthropathy?

Answer 3 The causes can be quite diverse, but the following conditions should be considered: ● ● ● ●

● ● ● ●

rheumatoid arthritis systemic lupus erythematosus psoriasis Reiter syndrome (usually an oligoarthropathy of larger joints) vasculitis gout scleroderma osteoarthritis.

Answer 4 This question is quite popular in exams. Nodules on the elbows should prompt consideration of rheumatoid arthritis and gout. Similarly, if asked to examine the hands, always seek to check the elbows as well. Question 5 Is gout likely in this patient?

Answer 5 No. Gout is very unusual in premenopausal women. The presentation is also not typical as the arthritis in gout is usually very painful and episodic. Question 6 What is the diagnosis and what basic pathological process underlies it?

This list is not exclusive. Question 4 What two conditions are characteristically associated with nodules on the elbows?

(a)

(a)

(b)

Figure 38.2 (a) Coronal MRI of hand with contrast showing synovitis (white arrow) and carpal bone erosion (black arrow). (b) Sagittal MRI post intravenous gadolinium showing enhancing rheumatoid pannus around the peg associated with erosion of the odontoid peg (arrow).

(b)

Figure 38.3 Sites of skeletal and non-skeletal involvement by rheumatoid arthritis. (a) Typical symmetrical polyarthritis includes cervical spine, the proximal interphalangeal joints of the hands and feet, knees, elbow, shoulder and temperomandibular joint. (b) About 25% of patients have non-tender red nodules in the skin, especially the elbow, forearm and hands, and elsewhere in the body, including heart valves, lung, spleen and other viscera. Severely affected patients may develop vasculitis which can cause gangrene of distal extremities and skin ulcers.

Case 38: A chronic autoimmune disease Prevailing H1

Answer 6 The patient has rheumatoid arthritis. Rheumatoid arthritis is an autoimmune disorder.The peak age at presentation is around 40–50 years and the disease is more common in women (M:F 1:3).Various patterns of disease exist, but the symmetrical polyarthritis affecting the hands and feet is typical. Rheumatoid arthritis is a destructive arthropathy, as in this patient. Around 20–30 per cent of patients have rheumatoid nodules, as this patient has on her elbow.The histopathologist’s description is consistent with a rheumatoid nodule. The use of the word ‘consistent’ in a report is important to note as it implies that the pathologist’s conclusion or the final diagnosis is dependent on data beyond that available from the specimen alone, for example the clinical context.

extend into the underlying bone, or across the joint to form adhesions, giving further joint destruction. Fibrosis of the granulation tissue can worsen joint adhesions. Question 9 What other features may be encountered in rheumatoid arthritis?

Answer 9 Despite its name, rheumatoid arthritis is a multi-system disorder, mainly due to a combination of a vasculitic process and the development of rheumatoid nodules in various organs. In addition to the arthritis of the limbs, spinal arthritis can develop and atlanto-axial subluxation is a particularly serious complication.

Question 7 Approximately 70 per cent of patients with rheumatoid arthritis have rheumatoid factor in their blood.This can be checked by a blood test.What is rheumatoid factor and what is its significance?

Answer 7 Rheumatoid factor is an autoantibody that is directed against the Fc portion of IgG. It is present in 5 per cent of the healthy population and also in a wide variety of rheumatological disorders, but if found in a patient with rheumatoid arthritis it indicates that the disease is likely to be more aggressive. Question 8 What would a biopsy of the soft tissue of an affected joint show in rheumatoid arthritis?

Answer 8 The synovial cells proliferate in rheumatoid arthritis, causing the synovium to develop villiform folds. There is chronic inflammation that typically includes lymphoid follicles and plasma cells. Fibrinous material is present.There can be an increase in the number of small blood vessels. The above changes are at first reversible, but if not treated they can progress to the generation of granulation tissue on the surface of the articular cartilage which is irreversible. The granulation tissue is referred to as pannus and interferes with the nutrition of the cartilage and causes degradation of the cartilage. The pannus can

Figure 38.4 Whole body nuclear bone scan in a rheumatoid patient with involvement of the sternoclavicular joints, the elbow, the wrist, the metacarpophalangeal (MCP) joints, the shoulder, the knees and the feet (arrows). Note the right shoulder and knee prostheses (arrowheads) and the urinary catheter bag (dotted arrow).

149

150

Part 4 Treatment

The heart may show pericarditis. Conduction disturbances may be caused by rheumatoid nodules. The lung can be affected by fibrosis, pneumonitis, rheumatoid nodules and pleuritis. A neuropathy induced by rheumatoid vasculitis is rare, but the other inflammatory aspects of the disease can cause nerve compression or entrapment, or damage to the spinal cord. Other than Sjögren syndrome, ocular disease is rare but includes episcleritis and scleritis. Haematoreticular disease is usually seen in the form of Felty syndrome in which there is lymphadenopathy, splenomegaly, neutropenia and possibly anaemia and/or thrombocytopenia. Felty syndrome is uncommon (1–5 per cent). Question 10 The patient is initially treated with non-steroidal antiinflammatory agents.These provide disease control for a couple of years, but after this period the disease progresses and joint deformities develop and worsen. Various other agents are tried, including glucocorticoids and penicillamine, but it is found that the most effective agent in this patient is gold.Why are NSAIDs and steroids used in rheumatoid arthritis?

Answer 10 Given that rheumatoid arthritis is an autoimmune condition, treatments that are likely to be successful should have anti-inflammatory and/or immunosuppressant actions.As their name indicates, non-steroidal anti-inflammatory drugs (NSAIDs) damp down the process of inflammation. They achieve this by inhibiting the generation of the various inflammatory mediators that are produced by the arachidonic acid cascade.This also endows them with analgesic properties. In early rheumatoid arthritis, this combination of an analgesic effect and anti-inflammatory action can be quite effective. This is somewhat unusual for an autoimmune disease as they typically require more aggressive immunosuppression than NSAIDs. Glucocorticoids are potent anti-inflammatory agents that are also immunosuppressants. If NSAIDs are ineffective in rheumatoid arthritis, steroids can be invoked. The high doses of steroids that are required in many autoimmune diseases can induce Cushing syndrome and its numerous harmful features. Therefore, it can be desirable to find alternative agents which do not have such diverse adverse effects. One of the most commonly

used is azathioprine. However, in rheumatoid arthritis, it has been found that both gold and penicillamine are effective, although they do not help in other autoimmune diseases. Some patients with rheumatoid arthritis can be managed successfully with NSAIDs. Others have more resistant disease and multiple agents have to be tried before an effective drug is found. Question 11 The patient’s disease is well controlled on gold. Eighteen months after starting this line of treatment, she notices that her ankles and calves have become swollen, together with puffiness of her face and bloating of her abdomen. Examination reveals that the abdominal bloating is due to ascites. Her serum albumin is markedly reduced and her urinary protein excretion is 8 g per 24 hours.What has happened and is it related to her rheumatoid arthritis?

Answer 11 The patient has nephrotic syndrome.This is a renal disease in which there is protein loss through the glomerulus that amounts to at least 3.5 g/day (various figures are quoted around this range and some adopt a pragmatic definition that the nephrotic syndrome occurs when the level of proteinuria is sufficient to cause oedema).There are numerous causes of nephrotic syndrome, one of which is gold therapy. SUMMARY

Rheumatoid arthritis is a multi-system autoimmune disorder of uncertain aetiology, in which the principle manifestation is arthritis. The underlying pathological process revolves around microvascular damage and defective T cell-mediated autoimmunity that involves complex cytokine cascades. The differential diagnosis includes other causes of arthritis. The clinical presentation is frequently characteristic and the investigations aim to exclude other diagnoses, particularly through the use of autoantibodies. Rheumatoid arthritis is a destructive arthropathy. Several characteristic deformities can result and these are due to combinations of joint destruction, subluxation, tendon disruption and derangement of the balance of actions of muscles at joints. The severity of the disease is variable and in more aggressive cases assorted second-line agents can be necessary if NSAIDs fail.These second-line agents are associated with a variety of side-effects, some of them serious.

CASE 39 KEEPING THE PUMP PUMPING A 61-year-old man presents to accident and emergency with a 30-minute history of severe chest pain.The pain was of sudden onset while the man was at rest in his armchair, reading the newspaper.The pain is central and radiates up to the jaw.The patient mentions that he feels sick and generally unwell. On examination, the man appears unwell and frightened and is pale and clammy. His pulse is 112/minute and regular. His blood pressure is 160/90 mmHg. The jugular venous pressure (JVP) is not raised. The apex beat is undisplaced but difficult to palpate; there are no heaves or thrills. The first and second heart sounds are normal. No added sounds or murmurs are heard. Bibasal crackles are present in the lungs, but examination of the respiratory system is otherwise unremarkable. The abdomen and nervous system are normal. The man’s ECG is shown in Figure 39.1. Question 1 What is the diagnosis?

1

11

111

aVR

Answer 1 The man has an acute anterior myocardial infarction. Question 2 What is the underlying pathological process?

Answer 2 Almost all myocardial infarctions are due to underlying atherosclerosis of the coronary arteries.Atherosclerosis is discussed in more detail in Case 1, Intermittent chest pain (p. 2). Question 3 What pathological process is happening in the myocardium?

Answer 3 As a result of the occlusion of its supplying coronary artery, the myocardium becomes ischaemic. If the ischaemia persists, irreversible cell death begins and the

v1

aVL

v2

aVP

v3

v4

v5

v6

Figure 39.1 ECG from patient in Case 39.There is a raised ST segment, indicating an acute myocardial infarctiohn, best seen in leads v2–6 (anterior leads).

152

Part 4 Treatment

process of coagulative necrosis occurs. This will result in an irreversible loss of the affected section of the myocardium, which will be replaced by fibrous scar tissue if the patient survives. The fibrous tissue lacks the contractile properties of the myocardium. Question 4 Once the diagnosis of an acute myocardial infarction is made in this patient, he is given aspirin and thrombolysis.What is the rationale behind these treatments?

Answer 4 Thrombolytic therapy is usually in the form of streptokinase or recombinant tissue plasminogen activator (rTPA). These drugs stimulate the body’s own thrombolytic pathways with the intention of breaking up the thrombus in the coronary artery and restoring circulation.This can limit the size of the infarction and if given

(a)

(b)

early enough can be highly effective. Note that speed is of the essence in the administration of thrombolysis but that there are several contraindications to thrombolysis that must first be excluded. Thrombolysis tends to assume the prominent and glamorous position in the acute treatment of a myocardial infarction (MI), such that the importance of the simpler aspirin is often wrongly underestimated. It should be remembered that in the initial papers that investigated thrombolysis the effect of aspirin alone on mortality was similar to that of thrombolysis. Aspirin inhibits cyclo-oxygenase.This enzyme is vital in the metabolism of arachidonic acid. In platelets, its function is crucial to permit the synthesis of thromboxane A2, a procoagulant agent. Aspirin therefore inhibits platelet-mediated thrombosis. In the context of a myocardial infarction, this can interrupt the development of the atherosclerotic thrombus and halt its extension, thereby reducing the size of the infarcted territory.

Figure 39.2 Heart failure in a patient following a myocardial infarct with cardiomegaly. (a) Distension of the upper lobe veins (arrow). (b) Interstitial lines at the base (arrow – so-called Kerley-B lines).These indicate septal/interstitial oedema.

Figure 39.3 Nuclear medicine stress or myocardial perfusion test (see Figure 1.3).The relatively dark areas (arrows) show where myocardium is under- or not perfused.The images show underperfusion of the posterior or inferior wall of the left ventricle following maximal exercise (the top row in each projection) which remains underperfused at rest (the bottom row in each projection) in keeping with infarction.

Case 39: Keeping the pump pumping Prevailing H1

Question 5 The patient is transferred to the coronary care unit. The introduction of coronary care units was one of the first measures to improve prognosis in acute MI. One of the facilities that they offer is constant ECG monitoring of the patients, supplemented by the provision of nursing staff trained in advance life support.Why is this important in acute MI?

Answer 5 One of the cellular consequences of ischaemia is the failure of Na⫹/K⫹-ATPase. In myocardial tissue, this is important in maintaining the negative electrical potential across the cell membrane. If this pump fails, the cell membrane depolarizes. In an electrically excitable tissue such as the myocardium, this can have serious consequences. Effective myocardial contraction depends upon the co-ordinated propagation of a depolarizing stimulus. Ischaemic, depolarized myocardium is susceptible to depolarizing spontaneously and acting as the focus for the generation of a tachyarrhythmia. Such arrhythmias can be fatal within minutes and in the case of ventricular fibrillation and ventricular tachycardia require immediate treatment with electrical defibrillation.

(a)

(b)

(c)

Figure 39.4 (a) Transverse slice through the left and right ventricles showing the bruised appearance of a recent myocardial infarction, about 2 days old, in the lateral aspect of the left ventricle. (b) Transverse section showing an established infarction, 3–4 days old, which is pale and softened (red arrow).The yellow arrow shows mural thrombus. (c) Vertical slice through a heart in which rupture of the interventricular septum is present.This infarct was around 5–7 days old, a time at which the damaged myocardium has been removed by macrophages, but fibrous scar tissue is incompletely developed, and the myocardium is soft and liable to rupture.

As well as tachyarrhythmias, serious bradycardias, including complete heart block, may occur if the damaged myocardium includes the conducting system. Inferior myocardial infarctions, caused by occlusion of the right coronary artery, are particularly prone to produce bradyarrhythmias because the right coronary artery supplies the sino-atrial and atrioventricular nodes. Question 6 What drugs can be given to stop the complication of tachyarrhythmias?

Answer 6 Beta-blockers. These have a negative chronotropic, calming effect on myocardial contractility and depolarization and can stabilize the electrical activity of ischaemic myocardium. Question 7 As part of his care, the patient has daily blood tests for the first few days. On the second day, he asks one of his doctors what the blood tests are for and is told that they are to help to assess the degree of damage to the patient’s heart.To which blood test in particular is the doctor referring?

Answer 7 In broad terms, the doctor would be referring to cardiac enzymes. Until fairly recently, these involved the complexities of creatinine kinase and its isoforms, aspartate transaminase and lactate dehydrogenase. However, these have been superseded by troponin T. Nevertheless, the principle is the same for each enzyme and that is that damaged myocardium leaks its contents, some of which end up in the peripheral blood. The greater the volume of damaged myocardium, the greater the peak level of the cardiac enzyme(s) and the longer the duration for which it (they) will be elevated. Troponin T has the advantage over its predecessors of being specific for the myocardium and showing its initial rise earlier in the evolution of an MI. Question 8 The patient becomes acquainted with the other patients in his bay, one of whom was admitted only a few hours after he was.The second patient has a history of asthma and is unable to receive beta-blockers.

153

154

Part 4 Treatment

The first patient is quite upset on the third day when the monitoring alarm goes off above his new friend’s bed and, despite the immediate attention of the medical and nursing staff, the second patient dies. The following morning, the patient overhears the medical staff talking about a haemopericardium being found at the autopsy. How does this relate to the second patient’s myocardial infarction, what are the related complications and what treatment can be given to try to prevent it?

Answer 8 An MI results in the death of the infarcted myocardium. The dead tissue is weaker than normal and is vulnerable to rupture.This can affect the free wall of the left ventricle, resulting in rupture of the ventricle and the expulsion of blood into the pericardial sac.The pericardial sac rapidly fills to capacity and compresses the venous return to the heart, resulting in tamponade.This complication is sudden and almost uniformly fatal (very rarely the rupture is locally sealed by the pericardium, leading to a false aneurysm). Other than the free wall of the left ventricle, there can be rupture of the ventricular septum or a papillary muscle (yielding acute mitral regurgitation). Both of these complications impose a serious and potentially fatal haemodynamic disturbance on an already malfunctioning heart. Beta-blockers reduce the incidence of all these types of ruputure due to their negative inotropic effect on the heart. Question 9 As a consequence of his MI, the patient finds himself started on several new drugs. These are explained to him by various staff. In addition to aspirin and betablockers, he is also prescribed an ACE inhibitor. Why?

Answer 9 The loss of myocardial tissue in an MI poses the danger of cardiac failure if sufficient ventricular mass is damaged. This is especially so in anterior MIs which frequently involve a significant volume of the left ventricle. ACE inhibitors have been found to reduce maladaptive changes that occur in the left ventricle in response to an MI, changes that exacerbate the tendency to cardiac failure. As well as being long term, the cardiac failure in an MI may be acute and dramatic, typically if a large volume of myocardium becomes ischaemic and nonfunctioning. Prompt salvage with thrombolysis may help to save some of this myocardium. Question 10 What other complications of MI can occur?

Answer 10 The damage to myocardial tissue induces an acute inflammatory response.This can involve the pericardium, resulting in pericarditis. Aspirin has anti-inflammatory properties and helps to control this inflammation. Related to the acute inflammation is the formation of ventricular thrombus over the infarcted myocardium. The damaged myocardium releases tissue factor which triggers thrombosis.The defective cardiac contractions that may result from the MI can produce local turbulence which is also thrombogenic. This thrombus may be the source of emboli and these can cause complications such as a cerebrovascular accident. Aspirin’s antithrombotic properties may be beneficial. The damaged myocardium will be replaced by fibrous tissue.This can deform, leading to a left ventricular aneurysm.Thrombus may develop in the aneurysm and be a source of emboli. The presence of the aneurysm can impair cardiac function and conduction, leading to cardiac failure and/or arrhythmias.

CASE 40 RADIOLOGIST OR SURGEON? A Sarah McKenzie Case

Sarah was shadowing Dr Mike Costello in the accident and emergency department on a busy Saturday evening when Cheryl, a 42-year-old woman came in distressed with pain in her right eye and a drooping of her eyelid. Both symptoms had developed over the previous 24 hours in a previously fit gym junkie. Her blood pressure on arrival was a little high at 150/95. Mike asked Sarah to examine Cheryl’s eye. Sarah noticed immediately that Cheryl had a drooping of her right eyelid (ptosis) which almost covered her eye. When Sarah raised the eyelid Cheryl complained of blurred and double vision. Sarah remarked that Cheryl’s pupils were asymmetrical, with the right one being larger than the left. Mike looked increasingly concerned and asked Sarah to check on the light reflex. Cheryl’s right pupil failed to constrict when Sarah shone a pen torch into the eye. Question 1 Which anatomical structure do these symptoms and signs originate from?

Answer 1 The muscles of the orbit are supplied by cranial nerves III (oculomotor nerve), IV (trochlear nerve) and VI (abducens nerve).The III nerve supplies the eyelid muscle, papillary muscles and most of the extra-ocular muscles which enable movement of the globe. Question 2 What are the causes of III nerve palsy?

Answer 2 Ischaemic neuropathy (in diabetic patients) Head injury

● ●

● ●



Cerebral artery aneurysm Inflammatory causes – meningitis, multiple sclerosis, sarcoidosis, vasculitis Tumours.

Mike took Sarah aside after performing a detailed neurological examination on the patient and explained that he was worried about a cerebral artery aneurysm causing Cheryl’s symptoms. Cheryl was not a typical arteriopath, had not had a head injury and did not have any other neurological signs to suggest inflammatory or malignant conditions. Sarah accompanied Cheryl to the CT scanner. Dr Sonia Jackson was the radiology registrar on call that night. She had brought her camp bed into the CT reporting room and was getting ready for a torrid night with a succession of head CTs on patients who had fallen over, had a stroke, got drunk or taken an overdose of recreational drugs. She perked up a little when Sarah told her about Cheryl. Sonia thought Cheryl should have a CT angiogram. This took five minutes after Cheryl was placed on the CT table and required an intravenous injection of iodinated contrast. Fifteen minutes later the processed images (Figures 40.1 and 40.2) appeared on Sonia’s computer screen. Within the hour Sonia’s consultant Dr Milton Redman and the neurosurgeon on call, Mr Campbell, were poring over the images. Question 3 What is an aneurysm and what types of aneurysms occur in the brain?

Answer 3 It is a localized dilatation in a blood vessel.This may be asymptomatic, press on adjacent structures such as

Figure 40.1 Processed sagittal and axial CT angiogram showing an aneurysm arising from the origin of the right posterior communicating artery (arrow).The oculomotor nerve passes close to the artery in the interpeduncular cistern as it leaves the midbrain and can therefore be compressed by an expanding aneurysm.

156

Part 4 Treatment

cranial nerves, become thrombosed and embolize distally or rupture resulting in a potentially catastrophic subarachnoid haemorrhage. Berry aneurysms develop because of a congenital defect in the media of the blood vessel at sites of bifurcation – usually in the vicinity of the circle of Willis – presenting in young or middleaged adults. Microaneurysms of Charcot–Bouchard are small aneurysms more distally in the cerebral circulation which present with a cerebral bleed in elderly patients. Rarely one sees post-infection mycotic aneurysms distally in the cerebral circulation. Question 4 What are the other types of aneurysms?

Answer 4 Atheromatous aneurysms in the aorta are an important group because they require treatment before they rupture or close off the origins of their tributaries.They are usually fusiform (spindle-shaped) and are caused by a fairly widespread weakening of the wall secondary to atheroma deposition. Weakening of the aortic media can also occur due to hypertension or inherited disorders of connective tissue, such as Marfan syndrome, and most commonly affects the arch of the aorta.These are not usually true dilated aneurysms but are often referred to as dissecting

(a)

aneurysms because the blood dissects through a tear in the intima to track down between the middle and outer thirds of the media.This can result in a double-barrelled aorta, if the extra track ruptures back into the aorta, or death if it ruptures outwards. The renal and mesenteric vessels can be damaged by arteritis, such as polyarteritis nodosa, producing aneurysms and local ischaemic damage. Syphilis and other infections are rare causes of aneurysms. Dr Redman and Mr Campbell talked at length with Cheryl and her husband who was with her at this stage. They explained that the aneurysm should be treated to stop a potential rupture.They felt that in Cheryl’s case endovascular coiling would be effective in treating the aneurysm. Question 5 What are the options for treating cerebral aneurysms?

Answer 5 Cerebral aneurysms can be treated surgically or with endovascular methods. Surgery involves a craniotomy and placing a clip across the neck of the aneurysm to exclude it from the circulation. Endovascular coiling involves placing a catheter in the femoral artery and

(b)

(c)

(d)

(e)

Figure 40.2 (a) Three-dimensional reprocessed CT image showing saccular aneurysm (arrow). (b) Precoiling angiogram via a catheter (arrowhead) placed in the cavernous internal carotid artery showing the anatomy of the aneurysm (thick arrow). (c) The tip of the microcatheter is in the aneurysm (arrow). (d) The detachable coils (arrow) are delivered into the aneurysm and effectively occlude the aneurysm at the end of the procedure. (e) Metallic coils in the aneurysm sac at the end of the procedure.

Case 40: RadiologistPrevailing or surgeon? H1

passing it through the aorta and carotid artery into the cranium. Microcatheters are used to enter the aneurysm. Detachable platinum coils are then delivered into the aneurysm sac eventually occluding the aneurysm with a combination of thrombus and coil. Eventually the aneurysm shrinks as it becomes excluded from the circulation. Coiling is associated with a more rapid recovery and shorter hospital stay than surgery and may also be associated with fewer potential complications. Not all aneurysms are suitable for coiling.A formally trained interventional neuroradiologist performs coiling Early next morning Sarah reassured Cheryl as she lay nervously in the anaesthetic room of the interventional radiology suite. She was quickly anaesthetized and taken in. Sarah watched Dr Redman catheterize the aneurysm with a microcatheter and then deliver the coils into the sac. A week later, Sarah went to visit Cheryl in the neurosurgical ward. Cheryl’s ptosis had almost resolved and her double vision was improving. Her eye pain and headache had resolved 24 hours after her coiling procedure.

Subarachnoid haemorrhage (ruptured berry aneurysm)

Question 6 What are the main types of cerebral haemorrhage?

Answer 6 The locations can be subdural, extradural, subarachnoid or intracerebral. Subdural haematomas can be acute or chronic and most often occur because of rupture of the veins crossing the convexity of the cerebral hemispheres. This means that the pressure is fairly low compared with an arterial bleed and blood accumulates slowly and often without major symptoms unless the haemorrhage is large. Extradural haematomas (epidural) generally follow skull fracture and especially rupture of the middle meningeal artery.They expand rapidly, compressing the underlying gyri and causing loss of consciousness. Subarachnoid haemorrhages are most commonly due to rupture of a berry aneurysm or vascular malformation.They present with sudden severe headache (sometimes during strenuous activity – such as sexual

Anterior cerebral artery

Haemorrhagic stroke (ruptured microaneurysm) Anterior communicating artery Internal carotid artery

Basilar artery

Vertebral arteries

Thrombotic or embolic stroke

Middle cerebral artery Posterior communicating artery

Posterior cerebral artery

Figure 40.3 This diagram of the cerebral blood supply shows the circle of Willis and compares the sites at which haemorrhagic and thromboembolic strokes occur with the typical berry aneurysms.

157

158

Part 4 Treatment

intercourse), photophobia, neck stiffness and vomiting. Another common cause is head trauma. The blood in the subarachnoid spaces can irritate other vessels, leading to spasm and cerebral infarction.The hydrocephalus which can follow a subarachnoid haemorrhage contributes to raised intracranial pressure.

Intracerebral haemorrhages occur mostly in patients with hypertension who have microaneurysms located in the basal ganglia, pons and cerebellar cortex. Forty per cent will die in the first week. Sarah was exhausted. She had not slept much in the last 48 hours because she was on an attachment to

Extradural potential space

Skull Dura

Subdural space

Pia-arachnoid Subarachnoid potential space

(a)

(b)

Subdural haemorrhage Extradural haemorrhage

(c)

(d)

Intracerebral haemorrhage

Subarachnoid haemorrhage

(e)

(f)

Figure 40.4 Sites of intracranial haemorrhage. (a,b) Normally the dura is densely adherent to the skull, thus trauma to the skull can lead to accumulation of blood between skull and dura (c). There is a relatively wide subdural space, criss-crossed with small vessels which bleed after trauma (d). The pia and arachnoid layers are effectively fused and a subarachnoid haemorrhage, commonly seen with ruptured berry aneurysm, leads to haemorrhage encasing the brain (e). This sometimes tracks along the optic nerves and is visible as a fluid level in the optic discs. Intracerebral haemorrhage behaves as a space-occupying lesion and can cause mid-line shift and coning (f).

CASE 41 ORGAN TRANSPLANTATION

A Sarah McKenzie Case

experience acute medical on-calls. As usual, the medical admissions unit was full, packed with patients suffering from ischaemic heart disease, CVAs and COPD. Interspersed were assorted presentations of malignant tumours, acute or chronic confusion and a scattering of other diseases. It was to one of this last group that Dr Juliet Smith was taking Sarah. The bright yellow of the young female patient’s sclerae was the first thing that Sarah noticed when she entered the cubicle.The striking blue of the patient’s eyes perhaps provided a physiological contrast to the yellow. Kara Ashcroft was 23 years old, employed in the advertising industry and had split up with her boyfriend of one year on Saturday afternoon after an acrimonious row. Distraught once her boyfriend had finally stormed out of her flat and the anger of the argument had given way to contemplation of its consequences, Kara had turned to a bottle of wine in her fridge for whatever comfort the best endeavours of her friends could not provide. By close to midnight, the bottle was empty, as were Kara’s spirits. In a very dark moment, her reason clouded by the alcohol, she opened a new box of paracetamol tablets and swallowed all sixteen of them, plus another half dozen or so that were still in a loose blister panel from a partially finished box. It was noon before Kara woke up on Sunday. Still miserable and far from refreshed despite her long sleep it required a few minutes before she was able to recall what she had done the previous night.Aghast at how she had reacted and certain that although she had seldom felt this low in her life before, she had no desire to die, she felt a sense of considerable relief that she was still alive and experiencing no ill-effects. Breathing a sigh of relief that her drunken misjudgement had not cost her her life, Kara spent a quiet Sunday at home, wondering how she would be moving on from the break up. The fear in Kara’s blue-in-yellow eyes told Sarah exactly what Kara must have thought when she woke up on Monday, just a few hours before Sarah and Dr Smith met her. Bright yellow, jaundiced sclerae would have shone back at her from her bathroom mirror, undeniable portents that her relief of Sunday was misplaced and that Kara was another of the sad list of people who had failed to appreciate how long a paracetamol overdose could take to wreak its malign effects. Sarah was impressed with how Dr Smith handled Kara. She obtained the history of what had happened

efficiently, but without obtrusive haste and still also ascertained Kara’s state of mind at the time she took the overdose. ‘Do you think she meant to kill herself?’ Dr Smith asked of Sarah while they were filling in the labels on the assortment of blood samples that Dr Smith had taken from Kara. ‘No. She basically said as much,’ answered Sarah.‘She was drunk and very upset and got carried away.’ ‘That’s what I think. It’s a so-called parasuicide.They tend to occur in women rather than men, particularly younger women, often after emotionally traumatic events such as this and involve readily accessible, nonviolent methods that don’t require much planning.The doses of the drugs involved typically aren’t that high and there’s no real suicidal intent, like in Kara’s case. Unfortunately, just because there’s not any intent doesn’t mean that the attempt won’t have that effect, particularly where paracetamol’s concerned.’ ‘Is she in that much danger, then?’ ‘Very possibly.’ Dr Smith signed one request form and began filling in another. ‘Obviously we’ve got a lot of blood tests here, but suppose you could only ask for one to see how badly damaged her liver was, which one box would you tick on these forms?’ ‘LFTs,’ replied Sarah, ‘to see how high her enzymes are and how much damage has already occurred.’ ‘That’s close, but there’s actually a better test of acute hepatic function.What we really need to know is how well her liver is functioning now. The enzymes don’t tell you that. I’ll give you a clue, it’s not on the chemical pathology form.’ Sarah assumed a puzzled expression. ‘Oh, the clotting,’ she finally deduced. ‘Exactly.The INR in particular is vital. It’s like an honorary LFT and if you think the liver is up the spout, you should get an INR as well as the traditional LFTs. Clotting factors are one of the first things to suffer if the liver’s struggling and the INR will pick that up. As you can see, we’re also getting an FBC. If the clotting’s deranged, you need to know whether the platelets are adequate and vice versa. Pre-existing anaemia won’t help the situation either. U and Es check out the renal function. Has hepatorenal syndrome developed and how are the kidneys doing now, in case things deteriorate? You need the glucose because the liver has an important role in maintaining blood glucose and you should always have in the

160

Part 4 Treatment

back of your mind the possibility of early diabetes being unmasked by an intercurrent illness. The last thing you need then is a diabetic crisis on top of everything else. ‘Any idea why I’ve requested an aspirin as well as a paracetamol level?’ added Dr Smith.

‘Because people often take both.You can buy both of them over the counter. Sometimes patients might not tell, especially if they really are suicidal.’ ‘Yeah. Or if they forgot, for whatever reason, like they were drunk, or took something like a

Normal/low doses Glucuronidation and sulfation Paracetamol At low doses, the glucuronidation and sulfation pathways handle most of the paracetamol and very little passes down the cytochrome p450 pathway

Inactive metabolite

Cytochrome p450

Conjugation with glutathione N-Acetyl-p-benzoquinoneimine (NAPQI)

Inactive metabolite

The metabolite of the cytochrome p450 pathway, NAPQI, is potentially toxic, but under normal circumstances is deactivated by conjugation with glutathione before it can exert its toxic effects Overdose

Paracetamol Chronic alcohol use induces cytochrome p450, favouring the metabolism of paracetamol by the cytochrome p450 pathway, thereby increasing the toxicity of any given dose

Inactive metabolite

The glucuronidation and sulfation pathways cannot handle the high levels of paracetamol so the cytochrome p450 pathway dominates The high levels of NAPQI deplete the liver of glutathione and neutralization of NAPQI fails

N-Acetyi-p-benzoquinoneimine

Inactive metabolite

Depletion of glutathione

Oxidation

Binding to hepatocyte proteins

Lack of glutathione to deal with other oxidative challenges

Inactivation of sulfhydryl groups in enzymes

Damage to proteins

Damage to DNA and proteins

Enzymes involved in calcium homeostasis are particularly susceptible to NAPQI-related oxidation

Elevated intracellular calcium

Damage to microtubules and microfilaments

Figure 41.1 The metabolism of paracetamol.

Case 41: Organ transplantation Prevailing H1

benzodiazepine that affected their memory. It’s also important to ask the lab to save the blood sample, in case any other drugs enter the equation later.Any other blood test that you’d do?’ ‘I can’t think of any.’ ‘Blood gases. Acidosis is a bad feature. I need to contact the liver unit because she’s presented a good 36 hours after taking the overdose, she’s jaundiced and we should involve them sooner rather than later. Can you put the bloods in the airtube for me, please, then we can have a chat about the usual management of a paracetamol overdose.’ Sarah placed the blood samples and their forms into the appropriate bags, then sent them to the labs via the airtube delivery system. ‘Right, the liver SpR [specialist registrar] is going to come down in a few minutes and take a look at her.You might want to follow this case through as I think you’ll learn a lot from it, Sarah, but while you’re waiting, what do you know about why paracetamol causes liver damage?’ The biochemistry lectures that had dealt with xenometabolism had been some time ago, but they had stuck in Sarah’s mind, especially the parts about paracetamol. ‘Normal doses of paracetamol aren’t a problem.The liver glucuronidates them and they’re inactivated. At higher doses, that pathway can’t cope, so some of the paracetamol gets shunted through cytochrome p450 in hepatocytes.That produces a toxic metabolite which can damage hepatocyte DNA. However, that isn’t normally a problem

(a)

because the liver puts a glutathione group on the toxic metabolite and neutralizes it.’ ‘So what goes wrong in an overdose?’ ‘The amount of the toxic metabolite swamps the liver’s reserves of glutathione, so it doesn’t get inactivated.’ ‘That’s right. So how does that explain the antidote?’ ‘The antidote is acetylcysteine, which contains sulphur atoms and these replenish the liver’s stocks of glutathione.’ ‘Okay. What about Kara’s case could mean she’s especially at risk, perhaps with a lower dose than under other circumstances? Don’t forget that while the normal maximum daily total dose is 4 g, some people can run into trouble at just twice that, 8 g.’

(a)

(b)

Figure 41.2 (a) Axial CT in a patient several weeks after a paracetamol overdose showing the development of portal venous hypertension (in the setting of chronic liver failure) with ascites (black arrow) and recanalization of the umbilical vein (arrowhead).The umbilical vein is normally only patent in fetal life and for a few months after. It carries oxygen from the placenta to the fetus. It reopens in cirrhosis and portal hypertension, shunting blood from the portal circulation to the systemic circulation. (b) Coronal T2 MRI in the same patient as in (a) showing the bright signal of the intraperitoneal fluid (arrows).

(b) Figure 41.3 (a) Liver slice from a patient who died a few days after a paracetamol overdose.The pale areas are the necrotic liver. (b) Low-power photomicrograph showing collapse of the perivenular region (acinar zone 3) with haemorrhage, after liver cell necrosis (arrowhead).The surviving liver cells in the periportal zone 1 areas show a normal trabecular arrangement (thin arrow), surrounding dilated pale-appearing sinusoids.

161

162

Part 4 Treatment

‘The alcohol. It’s an inducing agent so there’s more cytochrome p450 around which means that the threshold at which paracetamol starts going down that pathway is reduced.’ ‘And in somebody who’s an active alcoholic, which Kara doesn’t seem to be, there’s a danger that they’ll have a poor diet which means that they start off with low glutathione reserves, as well as having induced enzymes.You can see what a mess that could be.’ Sarah nodded. An hour later, Dr Uzma Zaman, the hepatology SpR had seen Kara and the labs were starting to return the results of the blood tests.The arterial blood gases were already known and were borderline acidotic. The INR was 2.3. ‘We’re going to take you up to the liver unit, Kara,’ Dr Zaman explained. ‘It’s a special unit we have for dealing with people with damaged livers,’ Dr Zaman had already told Kara what the problem was in relation to the injury caused by the paracetamol. ‘We’ll keep a very close eye on you there and do what’s necessary. Is there anybody you’d like us to call for you?’ ‘My parents, I gave the number when I was first admitted,’ said Kara.‘I’d really like them to be here now.’ ‘Of course. I’ll take care of it. Do you want me to tell them what’s happened?’ ‘I don’t know,’ answered Kara, who was a little tearful.‘I don’t want to worry them, but it’s too late for that and if they don’t know what the problem is, they’ll worry anyway.’ Dr Zaman nodded kindly. ‘I’ll tell them that you’ve had a bad reaction to some medication, which is true and then I can explain things properly when they arrive.’ ‘That would be good, thank you.’ Sarah accompanied Kara up to the liver unit where assorted lines and catheters were inserted and some more blood tests were taken. Sarah noticed how Kara bled a lot around the insertion sites and required a lot more pressure than usual to stop the bleeding. Her jaundice had deepened and she looked far from well. There was also not much urine in the catheter bag. A little over an hour later, Kara’s parents arrived. Understandably, it was a tearful few moments and Dr Zaman let them have them in privacy, before she took Kara’s parents into an office to fill them in with the details, having earlier established Kara’s permission to do this. Sarah waited with Kara during this time. ‘I’ve really screwed up, haven’t I?’ Kara said to Sarah. It seemed to be more of a statement than a question.

Sarah was not sure what to say. Kara’s observation was difficult to challenge in terms of her medical condition, but that was not what Kara needed to hear.‘You’re in the right place now. Everybody knows what they’re doing here.They’ll sort you out, one way or another.’ ‘Can you stay with me, please?’ asked Kara. ‘All the doctors keep changing. First there was Dr Smith, now there’s Dr Zaman who looks like she’ll be around for a bit, but then there was the consultant and that male doctor when I came up. It’s confusing, even though they’ve all been very nice. Now the nurses have changed shifts as well. I just need somebody around who knows what’s going on who’s been here from the start.’ ‘Of course.’ The afternoon slipped into the evening. Charts of vital signs and blood parameters all drew perturbed looks from anybody who studied them with a knowledgeable eye. The expressions were subtle and Kara would not have discerned them, but Sarah was used to doctors’ reactions. By eight o’clock, the number of phone calls and other conversations that centred around Kara had increased in inverse proportion to Kara’s level of alertness. Dr Zaman beckoned Sarah over. Kara’s parents were with her and Sarah had been talking to them for the last two hours, during which time Kara’s participation had waned considerably. ‘She needs a transplant,’ Dr Zaman told Sarah. ‘Her liver’s too far gone for the parvolex to have had a chance and there’s too much damage. All her parameters are going off and she’s getting encephalopathic.’ ‘Just from 12 g of paracetamol?’ ‘That can be enough. I’m going to tell her parents. Would you mind being there? You seem to have developed a rapport with them and I’m going to be busy organizing things to spend much time once I’ve given them the news.They could do with a familiar face.’ ‘Sure.’ Kara’s parents were predictably devastated when they heard the news. Kara was also distraught, although in her drowsy state, it was difficult to tell to what degree. Dr Zaman spent the next half hour making a variety of phone calls, chasing results of various tissue typing analyses and notifying various people.The arrival of an anaesthetist gave Dr Zaman and Sarah the chance to have a talk about the transplant. ‘Normally you’d have more time to work a transplant up,’ began Dr Zaman,‘but Kara doesn’t have that. She needs a new liver right now and she won’t last more

Case 41: Organ transplantation Prevailing H1

than a couple of days without one. As I’m sure you know, the body’s immune system will recognize tissue from another person as foreign and reject it. What do you know about that and how we try to reduce the chances of that happening? I mean the things we do other than immunosuppression?’ Sarah recalled lectures from an immunologist who was not as funny as he thought he was, but nevertheless could convey material competently.‘You have to match basic blood groups. If you don’t, the organ’s rejected almost as soon as the blood supply is restored.’ ‘Yeah. Hyperacute rejection. It’s a disaster and really shouldn’t happen. Sarah’s AB positive, so that’s perhaps the first lucky break she’s had. O negative is the universal donor and people often forget that AB positive is the universal recipient. Go on.’ ‘Then you have to match the HLA types. Somebody said they’re like an ID card as far as transplants are concerned. If the donor organ doesn’t have the right ID, T cells will recognize it and there’ll be a T- and B-cell response.’ ‘That’s acute rejection and happens in a few days after the transplant. You try to minimize it by HLA matching.The better the match, the less likely it is and the less heavy duty the immunosuppression you’ll need. The strange thing is that you can often get away without this step in solid organ transplants. After that?’ ‘Chronic rejection. That’s a more insidious process, maybe due to more minor tissue mismatches and inadequate immunosuppression.The donor organ fails slowly.’ ‘In a nutshell. And we’ve got just hours to sort all of that out for Kara. We got a head start by taking the bloods when she first came up to the unit. It wasn’t exactly a feat of skill to know that a transplant was on

the cards, so we needed to get things rolling as soon as possible. Excuse me.’ Dr Zaman’s attention was summoned by one of her colleagues. She talked to them for a few moments then returned to Sarah. ‘She’s had another break, although it means that somebody else has just had some very bad news.There’s a donor in Durham.The organ harvesting team are just leaving now. It’s not a perfect match, but it’s close enough for Kara and as she’s now at the top of the liver transplant list, she’ll be getting it. They’ll be taking her to theatre before midnight.You might want to go along, if you’re up for it. It’ll be a long operation but it’s not something you see every day and we’re lucky that we actually have theatres here where you can see what’s going on.’ Dr Zaman’s prediction of the start time was accurate to 15 minutes. It was not only daylight but the start of another normal working day before Sarah emerged from the operating theatre. Kara’s own liver had departed several hours ahead of her, swollen, red and congested and looking soft, friable and necrotic, ravaged by the paracetamol.The microscopic slides would be ready the next day, but Sarah’s immediate thoughts were to return to the liver unit to keep her promise to Kara. It was late afternoon before Kara regained some semblance of consciousness. The medical team were starting to speak in encouraging tones about Kara’s prognosis and as she was almost out on her feet, Sarah decided that she would be no further use to anyone if she did not have some sleep. Four days later Kara was looking much better. Her new liver had dealt with the accumulated problems left by her old, destroyed liver and her rehabilitation now centred around recovering from what was in itself major abdominal surgery.

163

PART 5

COMPLEX MANAGEMENT

Case 42: Immobility and its complications

Case 47: Mixed social and medical 166

problems

186

Case 43: A life-threatening rash

170

Case 48: Certifying the cause of death 191

Case 44: A first seizure

174

Case 49: Referring cases to the

Case 45: Febrile and unconscious

178

Case 46: Multi-organ disease

183

coroner



Case 50: Your worst nightmare!

194 197

CASE 42 IMMOBILITY AND ITS COMPLICATIONS A 67-year-old man is referred by his GP to the neurology outpatient department with a history of several weeks of weakness in his limbs. His previous medical history includes type 2 diabetes mellitus for 15 years. He is an ex-smoker, having smoked 15 cigarettes per day from the age of 21 to 55. The patient does not think that the weakness came on suddenly. It is most noticeable in his hands. On examination, the patient’s first 11 cranial nerves are normal. However, he has fasciculations of his tongue. Examination of the limbs reveals wasting of the small muscles of the hands.There are fasciculations in his right biceps, but the muscle bulk is preserved and his right biceps reflex is increased. Tone is elevated in his left forearm, but there is muscle wasting. Both patellar tendon reflexes are hyperreflexic, but his right quadriceps muscle group exhibits wasting. Clonus is elicited at the left ankle, but the left ankle reflex is absent and the left calf seems wasted. The sensory examination is normal, as is co-ordination. Question 1 What is unusual about the pattern of this man’s motor signs?

Answer 1 Not only does the patient have mixed upper motor neurone (UMN) and lower motor neurone (LMN) features (see Case 11, Intermittent neurological signs (p.40)), but these mixed UMN and LMN features are present within the same muscle group. Furthermore, this mixture is found within several muscle groups. For a patient with a single disease process, motor features tend to be either of the UMN or the LMN type, but not both. If both are present, either the patient has more than one disease, or a particular single disease becomes very likely. Question 2 Is diabetes mellitus likely to be the explanation for his neurological features?

Answer 2 No. A sensory neuropathy tends to predominate in diabetes mellitus. In this patient, the features are exclusively motor. In addition, diabetic neuropathy affects

the peripheral nerves only and this patient’s UMN signs indicate an element of CNS disease. Question 3 What single disease can affect both upper and lower motor neurones across several muscle groups, as in this case?

Answer 3 Motor neurone disease (MND).While it is important to be wary of dual pathology, or an unusual presentation of a systemic disease such as vasculitis, in a patient in whom the combination of UMN and LMN features is seen in three or more muscle groups, in the absence of sensory features, motor neurone disease is very likely. Question 4 What patterns of features can be encountered in this disease?

Answer 4 Three main patterns are seen, depending on where in the motor nervous system the disease process dominates. Overlap can occur. ●





Amyotrophic lateral sclerosis dominates in the corticospinal tract, giving mainly UMN features. Progressive muscular atrophy affects the anterior horn cells, yielding LMN signs. Bulbar palsy focuses on the cranial nerves and the muscles of the head and throat.

Note that the extraocular muscles are virtually never affected in MND. Question 5 What is the basic pathology of the disease?

Answer 5 As the name implies, the disease shows loss of motor neurones in the corticospinal tract and the anterior horn of the spinal cord. These pathways become atrophic and smaller. The pathogenesis is uncertain. Most cases are acquired. Inherited cases tend to have an earlier age of onset and a much longer survival. Current theories

Case 42: Immobility and its complications

Pyramidal cells in primary motor cortex Final output of the brain to the motor neurones in the spinal cord Receive projections from basal ganglia, cerebellum, higher motor regions and other higher regions Also known as upper motor neurones. Much of their basal activity is inhibitory and serves to suppress primitive reflexes that operate at a subcortical and spinal level and are unwanted in the presence of higher motor skills

Basal ganglia Receive projections from various sensory pathways and brain regions and assist with the development of motor functions. Project to the primary motor cortex to influence the output of this region. Precise functions remain to be elucidated Disease states are characterized by increased tone (rigidity), abnormal movements (e.g. tremor, chorea, athetosis) and bradykinesia, but not paralysis of voluntary movement.

Other sensory fibres Assorted somatosensory fibres such as pressure, pain, fine touch and crude touch, project to the cerebral cortex and the cerebellum and are integrated to assist in the development of optimal motor output signals Muscle spindle Measures the degree of stretch in a muscle and tendon. Serves to help to maintain the resting state of tone of the muscle and is essential to the spinal stretch reflex. Provides crucial proprioreceptive information for co-indination

Sensory data going to brain

Cerebellum Receives sensory information from proprioreceptors and other somatosensory fibres. Also receives input from vestibular system. Sends fibres to and receives them from the primary motor cortex. Integrates data from these various sources to coordinate movement and modify the activity of the primary motor cortex. Disease states are characterized by impaired coordination, either of basic motor functions such as balance and eye movements and/or complex skilled movements (including speech)

Skeletal muscle fibre

Gamma motor neurone Innervates the muscle spindle and serves to modify the size of the stretch reflex. A form of lower motor neurone. Increased gamma motor neurone activity stretches the muscle spindle, increasing its sensitivity and therefore increasing the size of any reflex caused by stretching of the muscle spindle Figure 42.1 The nervous pathways that interact to control motor function.

Alpha motor neurone Final common output pathway from the nervous system to muscle fibres. All higher motor information converges on the alpha motor neurone to be relayed to the skeletal muscle fibre to produce movement

167

168

Part 5 Complex management

revolve around excitotoxicity, in which excitatory amino acid neurotransmitters are believed to mediate cell death by causing an excessive influx of calcium ions through ion channels. Free radicals are also implicated. Question 6 What is the usual prognosis?

Answer 6 Motor neurone disease is typically relentlessly progressive. Patients tend to die within three years. Question 7 The patient is given the diagnosis. Eighteen months later, he is admitted to hospital from a nursing home suffering from a fever. Examination discloses that he has 4 ⫻ 3 cm ulcers over both heels and a 8 ⫻ 6 cm ulcer over his sacrum.All three ulcers are deep.That over his right heel exposes tendons. Deep soft tissues are observed in the sacral lesion.At this point in his disease, the patient has been immobile for three months and has difficulty in swallowing. He has marked wasting of all four limbs.What is the diagnosis of his skin lesions?

Answer 7 The patient has pressure sores. The heels and sacrum are typical locations for pressure sores and the patient’s condition places him at great risk of these lesions (see Figure 42.2). Question 8 Why is the patient at risk of this complication?

Answer 8 Pressure sores arise in patients who have decreased mobility. Constant pressure on weight-bearing areas, typically the sacrum and heels in somebody lying in bed, can lead to breakdown of the skin. If the problem persists, the ulcer becomes deeper. In extreme cases, the ulcer can extend down to bone. The development of pressure sores becomes more likely if certain risk factors are present.These relate to wound healing. Question 9 What factors are necessary for good wound healing?

Answer 9 Wound healing can be considered to be analogous to a construction site project. Raw materials are necessary to form the new structure. Adequate transportation networks must be in place to bring these raw materials to the construction site.The damaged structure must be cleared away. Vandals must be stopped from entering the site and disrupting the process.The problem which caused the damage in the first place must be prevented from recurring. In wound healing, this analogy equates to adequate nutrition to provide the energy and proteins necessary for generating new tissue. Good blood flow is essential to deliver nutrients and oxygen to the tissue, as well as for transporting breakdown products away. The immune system is necessary for preventing infection of the wound and for clearing away debris. Question 10 Figure 42.2 Poor blood supply, oedema of the lower limbs, poor nutrition and infection may all contribute to the failure to heal of a pressure sore in a bed-ridden patient.

Given the factors necessary for wound healing, why might this patient be at particular risk of pressure sores?

Case 42: Immobility and its complications

Wound gap a. Fibrin clot

Wound gap b. Day 1–2: cellular infiltrate, temporary matrix, wound contraction, epithelial migration, clot dissolution

c. Day 3–4: surface intact, new basement membrane, definitive matrix

d. Day 5: scar

Figure 42.3 Bedsores illustrate the principle of healing by secondary intention, which occurs when much tissue has been lost and leads to appreciable scar formation. By contrast a surgical incision, in which little damage is caused to dermal and subcutaneous tissues and in which the sides can be brought together by suturing, heals with virtually no scarring.

Answer 10 The patient has a history of smoking and diabetes mellitus. Both of these conditions can produce microvascular insufficiency and this can impair would healing, especially at a distal site such as the heel. The patient’s MND-related dysphagia will cause problems with maintaining an adequate nutritional intake. As well as being the cause of the immobility which underlies the pressure sores, the patient’s MND has also produced muscle wasting. This loss of muscle padding renders the skin more susceptible to the pressure effects related to the underlying bone. Immobility can also cause problems with personal hygiene, including those related to the excretion of urine and faeces. Prolonged contact of the skin to these aµgents can facilitate the development of pressure sores.

Question 11 What steps can be taken to reduce the chance of developing pressure sores?

Answer 11 Prevention is the key strategy in handling pressure sores. A vital step is recognizing which patients are at risk and this is derived from an understanding of the basic pathology of wound healing. Patients at risk can be nursed on special mattresses which redistribute the weight-bearing load by varying inflation and deflation of different compartments of the mattress. Help in moving and turning the patient frequently is important, as is assisting the patient in maintaining their personal hygiene. Diet should be adequate and provided in a manner that the patient can effectively eat and absorb. Scrupulous surveillance of pressure areas is also essential.

169

CASE 43 A LIFE-THREATENING RASH A 53-year-old woman presents to accident and emergency with a rapidly progressive, painful skin rash.The rash commenced the previous day with widespread erythema that initially affected the patient’s arms and legs. As the patient had been doing some gardening during the sunny day before, she initially attributed it to the sun and irritation by plant sap. However, the rash rapidly spread and became tender. Shortly after this, blisters began to form and, a few hours prior to her arrival at hospital, her skin started to slough off from some of the affected areas. By the time of her attendance at accident and emergency, the extent of this sloughing had become much worse. Her previous medical history is largely unremarkable, although she had begun taking antibiotics for a urinary tract infection two days before the rash appeared. On examination, the patient is tachycardic and appears dehydrated. Large areas of the superficial layers of her skin have been lost. In addition, she has numerous blisters and widespread erythema. A few more discrete lesions are revealed by close inspection and these consist of concentric rings of erythema that resemble a target. Question 1 What is the likely diagnosis?

Answer 1 Toxic epidermal necrolysis. The targetoid lesions are a characteristic feature of toxic epidermal necrolysis, as is the widespread erythema and skin loss.

adhesion of the basal layer of the epidermis to the underlying dermis. Breakage of this adhesion and loss of cohesion between keratinocytes, together with local oedema also induced by the inflammation permits the epidermis to separate from the dermis and form a blister. As the process continues, the inflammation completes the destruction of the keratinocytes that form the blister and the dead epidermis sloughs off, being no longer adequately anchored to the underlying dermis. Question 3 How could a person’s immune system come to mount such a powerful response against their own skin?

Answer 3 As far as the magnitude of the response is concerned, the function of the immune system is to destroy anything it recognizes as foreign. Therefore, while the effects of this are potentially catastrophic in toxic epidermal necrolysis, the degree of the immune reaction is consistent with the role of the immune system, albeit that the target identification is aberrant. The mechanisms by which the immune system’s ability to separate self from foreign is deranged in toxic epidermal necrolysis are not established in precise detail. However, it is believed that there is some sort of initiating trigger, typically either an infection or a drug, which either causes cross-reactivity between the trigger and the antigenic properties of the skin, or modifies the antigenicity of the skin such that it now appears foreign to the immune system. In the case of drug-induced

Question 2 A skin biopsy is performed and confirms the clinical impression.What basic process is happening in the skin in this disease?

Answer 2 In toxic epidermal necrolysis, the epidermis is targeted by a lymphocyte-based autoimmune reaction. The direct cytotoxic action of T lymphocytes, in conjunction with the effects of cytokines such as tumour necrosis factor, destroys the keratinocytes of the epidermis. The initial stage of erythema reflects the beginning of this inflammatory reaction. The formation of blisters denotes damage to the keratinocytes, in particular the

(a)

(b)

Figure 43.1 (a) Low-power view of a skin biopsy that displays toxic epidermal necrolysis.The epidermis has separated from the dermis.There is little remaining inflammation, possibly because the immune system has destroyed its target and moved on. (b) Higher power view to demonstrate the epidermal necrosis. Most of the keratinocytes are ghost cells which have lost their nuclei.

Case 43: A life-threatening rash

toxic epidermal necrolysis, certain metabolites of the drug can act as haptens on epidermal molecules and mislead the immune system. Question 4 Toxic epidermal necrolysis is rare, yet some of the viral infections that can cause it are not. Similarly, many of the drugs that can precipitate the condition are in common use. How could toxic epidermal necrolysis be only a rare complication of common events?

Answer 4 The precise details for this process are also not fully elucidated, but in general terms, the phenomenon relates to genetic variation. Different HLA phenotypes have different susceptibilities to autoimmune disease, for example, the strong association between HLA-B27 and ankylosing spondylyitis, or the tendency for people

Keratinocytes: waterproof, resist minor trauma Interdigitating antigen presenting cells in prickle cell layer detect pathogens and migrate to lymph nodes to present antigen to lymphocytes Basal layer contains stem cells which regenerate after trauma. Melanocytes produce protective melanin pigment Intra- epithelial and circulating lymphocytes interact with other immune cells to resist pathogens Sensory nerves detect dangerous stimuli Pilosebaceous units, sweat glands and skin capillaries contribute to temperature control Mast cells and macrophages in extracellular space respond to trauma and invading pathogens

Figure 43.2 Skin has many roles.

with organ-specific autoimmune diseases to be HLA DR3- or DR4-positive. In toxic epidermal necrolysis, certain HLA phenotypes and other aspects of immune function that demonstrate genetic polymorphism may render the individual more vulnerable to developing an aberrant immune response against the epidermis, should an appropriate precipitating event occur. In parallel to genetic variation in the immune system is the polymorphism of xenometabolism. Different people will metabolize some drugs at different rates and by different pathways. This leads to the accumulation of different metabolites at different concentrations. Some of these metabolites may be able to act as haptens that could trigger toxic epidermal necrolysis. The element of coincidence is also important.A person may have the immune constituents that render them susceptible to toxic epidermal necrolysis, but never meet a trigger that will precipitate the process, so

171

172

Part 5 Complex management

Primary barriers Eye: tears

Secondary barriers Mucosa-associated lymphoid tissue:

Ear: waxy secretions Nasopharynx: ciliated epithelium mucous secretion Mouth: saliva Trachea and bronchi: ‘mucociliary escalator’, lgA from MALT tissue Lung: Intra-alveolar macrophages

• Pharyngeal lymphoid tissue (‘Waldeyer’s ring’ includes tonsils) • MALT in bronchial tree, intestines (particularly pronounced in Peyers patches in terminal ileum), bladder. • Inducible MALT in stomach and salivary glands

Stomach: acid, pepsin Intestines: bile, pancreatic enzymes, lgA from MALT tissue. Colon contains commensal bacteria.

Urethra: urinary stream; valves prevent retrograde flow.

Vagina: lactobacilli, slightly acid secretion Skin: waterproof, robust

Lymph nodes: main groups in cervical, supraclavicular, axillary, groin, para-aortic and mediastinal regions. Each organ has associated lymph nodes.

Figure 43.3 Natural barriers to infection: the importance of skin and other epithelial surfaces as protective barriers is only really appreciated once they have been breached.

the disease never happens. Similarly, a person might have the metabolic pathways that allow them to generate the harmful haptens, but their immune system may be resistant to being fooled by them and toxic epidermal necrolysis never supervenes. Question 5 Returning to the patient, what problems is she likely to face as a consequence of large parts of her epidermis being lost?

Answer 5 Toxic epidermal necrolysis is a life-threatening condition that emphasizes some of the functions of the skin. The skin is a waterproof barrier.When it is disrupted over a large area, the volume of water lost through the skin is greatly increased because this waterproofing is compromised and the underlying tissues become exposed to the air.Thus, regulating fluid balance can be challenging.

Case 43: A life-threatening rash

The skin is also a barrier to infection and can be considered to be the outermost element of the innate immune system. When the skin is breached, microorganisms have a means of access to the body that would not normally be available to them. In toxic epidermal necrolysis, this breach is large. As well as protecting against water loss, the skin is important in reducing heat loss (as well as facilitating heat loss when necessary). This thermoregulatory capacity of the skin is severely disrupted in toxic epidermal necrolysis and the patient can have difficulty in maintaining their core temperature. Exposure of large areas of the subepidermal tissues is painful and providing adequate analgesia is vital. Question 6 What other diseases could be present that would further complicate the management of a patient with toxic epidermal necrolysis?

Answer 6 The problem of managing a patient’s fluid balance becomes even greater if they have co-existing

cardiovascular disease. Such patients are less able to tolerate fluid overload and the balance between ensuring that the patient does not become fluid depleted, yet does not suffer cardiac failure from fluid overload can be delicate. Toxic epidermal necrolysis places the body under considerable stress. Any stressful intercurrent illness can derange previously stable control of diabetes mellitus. Patients with type 2 diabetes may temporarily require insulin and those with type 1 diabetes may require significant changes to their normal dose. Sliding scale regimes usually provide the best solution to this problem. Addison’s disease is less common than diabetes mellitus and sometimes goes undiagnosed for some time. However, a patient who is coping reasonably well with subclinical Addison’s disease, or who is a known patient controlled on oral replacement therapy may suffer an Addisonian crisis when the stress of the new illness overwhelms their failing adrenal corticosteroid system.

173

CASE 44 A FIRST SEIZURE A 29-year-old man is walking in the park with his friends when he suddenly collapses and is witnessed to lose control of his bowels and begin generalized shaking of all of his limbs.This episode lasts for three minutes, during which time he is unresponsive.After the episode stops spontaneously, the man is in a deep sleep. An ambulance is called and the man is taken to the accident and emergency department of the local hospital. On arrival at accident and emergency, it transpires that the man has no history of seizures and has been previously fit and well, although he does mention that he has had a dry cough for the past few days and has felt short of breath on exertion, as well as having headaches. He had attributed this to a ‘bad cold with a touch of bronchitis’. No abnormality is found on examination. The patient’s full blood count, urea and electrolytes and liver function tests are normal. The chest X-ray reveals subtle shadowing in the lung fields radiating out from both hila in a ‘batwing’ distribution. An urgent brain CT scan is performed and shows an enhancing mass in the right frontal lobe. Extension of the scan beyond the originally intended region does not discover any other mass lesions. A neurosurgical and oncological opinion is obtained and the patient undergoes a biopsy of the cerebral lesion. The histopathology report gives a diagnosis of a diffuse large B cell lymphoma. A chest opinion has also been sought and it has been determined that while the patient has normal oxygen saturations at rest, he is susceptible to marked desaturation on exertion. A bronchoscopy is undertaken and includes a brochoalveolar lavage (BAL). Question 1 What abnormality is the BAL likely to show?

(a)

(b)

Answer 1 The diagnosis of Pneumocystis carinii pneumonia (PCP) is the anticipated finding in the fluid obtained from the BAL. The clinical and radiological picture of the patient’s respiratory problem is in itself suggestive of PCP. Desaturation on exertion is characteristic, in the appropriate clinical setting, as is the ‘batwing’ pattern on chest X-ray (although other conditions, such as uraemia, can have a similar distribution of radiological changes).The cough in PCP is typically dry. However, Pneumocystis carinii is not a normal cause of pneumonia and requires an appropriate context in which to suspect it. Question 2 Which group of patients is susceptible to PCP?

Answer 2 Patients who have defective cell-mediated immunity. Question 3 Are the any reasons to suspect that the patient has defective cell-mediated immunity (other than the presence of PCP)?

Answer 3 The patient has a cerebral lymphoma. According to the CT, this seems to be a primary CNS lymphoma as there is no CT evidence of lymphoma outside the brain. Primary CNS lymphoma is a characteristic finding in a particular condition in which there is defective cellmediated immunity.

Figure 44.1 (a) Chest X-ray from a 37year-old male patient with HIV and Mycobacterium avium intracellulare (MAI) infection (arrow). (b) Chest X-ray from a 27-year-old male patient with HIV showing diffuse bilateral lung infiltrates – Pneumocystis carinii pneumonia (PCP).

Case 44: A first seizure

(a)

(b)

(d)

(c)

(e)

(f)

Figure 44.2 (a) H&E and (b) Grocott show Pneumocystis carinii (PCP) and (c) shows Mycobacterium avium intracellulare (MAI).These are two commonly encountered organisms in HIV/AIDS patients with pneumonia. However, tuberculosis is the major global cause of death in HIV/AIDS. (d) Cryptococcus and (e,f) Toxoplasmosis are two of the most common infective intracerebral lesions found in HIV/AIDS.

Question 4 In which disease is a primary CNS lymphoma a characteristic finding?

Answer 4 In a patient who has human immunodeficiency virus (HIV) infection, a primary CNS lymphoma is an ‘AIDSdefining illness’, as is PCP. The progression from the presentation, through the selection of the specific investigations, to the diagnosis of acquired immune deficiency syndrome (AIDS) in this patient may seem abrupt, but there are certain important points. The patient has no previous history of seizures.While approximately 3 per cent of the population are said to experience a seizure at some point in their life, the new onset of primary epilepsy in an adult should be viewed with caution until an underlying cause for seizures is excluded. A common secondary cause of a seizure is a mass lesion. In isolation, the patient’s history of a headache could have numerous causes. However, when combined with a seizure, the emphasis shifts to excluding a mass lesion. Under ordinary circumstances, the patient’s interpretation of his dry cough could well be plausible. However, the chest X-ray findings are not congruent and as discussed above, do raise a certain possibility. When the existence of a CNS mass lesion and a respiratory problem that could be PCP are combined,

HIV/AIDS enters the differential.The biopsy finding of a lymphoma greatly strengthens this suspicion. Question 5 How does the HIV virus cause immunosuppression?

Answer 5 The HIV virus infects and kills the CD4 subset of T lymphocytes.These are the helper T cells and have a central function in the regulation of many aspects of the immune system. If the CD4 T cell population is depleted, the immune system’s ability to organize its response against various types of infection is lost. Therefore, while the other individual components of the immune system may remain functional, they cannot operate in a co-ordinated fashion and this is enough to hamper the efficacy of the immune response. Question 6 To what sorts of infection are people with defective T cell-mediated immunity susceptible and why?

Answer 6 The T cell response is particularly useful for dealing with intracellular organisms, such as viruses, mycobacteria and some fungi.This relates to the ability of cytotoxic T cells to destroy infected cells. However, this destructive ability of cytotoxic T cells is a weapon that must be tightly regulated to prevent it from activating inappropri-

175

176

Part 5 Complex management

Highly specific immunoglobulin gp120

Plasma cell

Memory B cell

Differentiation involves hypermutation

Viral and cell membranes fuse, releasing viral capsid into host cell

Ig with reasonable 'fit' for antigen B cell

FDC

Whole antigen retained on the FDC surface

Co-stimulatory molecules and cytokine secrection

Co-stimulatory molecules and cytokine secrection

CD4+ T cell (helper)

Th1 cells secrete IL-2 (CD25) and IFNγ TCR MHC II

CD 4

APC CD8+ T cell

Memory T cell

HIV retrovirus with surface gp120 molecule, which binds CD4 on T helper (Th cells and other antigen presenting cells (APC) such as macrophages

MHC II

HIV infects CD4+ cells. Integrated into host DNA, it lies dormant until proliferation of infected cells is induced, e.g. by infection with Pneumocystis carinii or Toxoplasma gondii. Loss of CD4+ helper cells results in a reduction in cell-mediated immunity, allowing organisms which are usually controlled or destroyed by this arm of the immune system (e.g.TB) to survive. Tumours, particularly those linked to oncogenic viruses, e.g. Kaposi sarcoma, non-Hodgkin’s lymphoma are all more common in HIV/AIDS and relate to decreased immune surveillance and immunodysregulation.

Cytokine secretion Killing/induction of apoptosis Figure 44.3 Normal interactions between CD4+ T helper (Th) cells, CD8+ effector T cells, B cells and antigen-presenting cells, such as macrophages are interfered with by HIV infection.The cells co-ordinate the acquired cell-mediated and humoral immune responses.

ately. T helper cells are important in giving cytotoxic T cells permission to strike.Therefore, if the helper T cells are crippled, the cytotoxic response is also hamstrung. Question 7 Why is the immune response to bacterial infection better preserved and are there any problems with this aspect of the immune system in HIV/AIDS?

Answer 7 As extracellular organisms, bacteria are ideal targets for antibodies and therefore fall within the province of the B lymphocyte–plasma cell system. B lymphocytes can be stimulated directly by their cognate antigen without the need for T cells to assist them and thus bacterial infections can still be addressed if T cell function is defective. However, the B cell response is more

Case 44: A first seizure

(a)

(b)

(c)

(d)

(e)

Figure 44.4 MRI scans from HIV-positive patients. (a) Enhancing left occipital mass (arrow) in a 53-year-old patient with cerebral lymphoma. (b,c) HIV encephalitis in a 37-year-old man with cortical atrophy (b, arrow) and ‘bright’ inflammatory white matter changes (c, arrow). (d) From 47-year-old man with HIV and cerebral cryptococcosis (arrow). (e) Cerebral toxoplasmosis in a 42-year-old man.

complex. Activated helper T cells can also activate B cells, thereby augmenting the B cell response. Furthermore, B cells are dependent on T cells to switch their immunoglobulin class from IgM to IgG or IgA. The availability of appropriate immunoglobulin subclasses is vital to provide an optimal response. T cells also assist B cells in hypermutating their immunoglobulin to provide the best specificity for the target antigen, as well as allowing the generation of memory B cells. Therefore, if there is a problem with T helper cells, B cell performance is suboptimal. In addition to the impairment of the B cell response in HIV/AIDS, there can be a polyclonal increase in immunoglobulins. This also reflects dysregulation of B cell activity. Question 8 What term is employed for the infections that characterize HIV/AIDS and other diseases of immunocompromisation?

Answer 8 Opportunistic infections.These are infections by organisms that are normally not able to cause an infection in a healthy person. However, when the immune system becomes defective, they are able to take advantage of this weakness. Question 9 Why are certain types of tumour more common in HIV/AIDS?

Answer 9 The immune system has a role in suppressing neoplasia. Neoplastic cells can express altered proteins on their cell surfaces as a result of the mutations that have

transformed them into neoplastic cells. The immune system, especially T cells and NK cells, can recognize these proteins and attempt to remove the cells. High-grade B cell lymphomas are among the most common malignancies in HIV/AIDS. As mentioned above, the regulation of B cell function is already disordered by the depletion of the helper T cell population. This situation becomes exacerbated by the ability of the Epstein–Barr virus (EBV) to intercalate itself into the DNA of B cells and overdrive their proliferation. Loss of the T cell response to EBV in HIV/AIDS permits infected B cells to proliferate aberrantly (the B cells may have been infected many years prior to the onset of HIV/AIDS and have behaved normally due to adequate regulation by the helper T cell system during this interval). In some cases, this is accompanied by the accrual of mutations that culminate in malignant transformation. Interestingly, one of the other characteristic malignant tumours of HIV/AIDS, Kaposi’s sarcoma, is intimately related to human herpes virus type 8. Furthermore, cervical carcinoma, which is dependent on the human papilloma virus, is an AIDS-defining illness in a patient known to have HIV. Question 10 What specific treatment is given in HIV/AIDS?

Answer 10 Highly active antiretroviral therapy (HAART). HAART aims to reduce the viral load. This allows the CD4 T lymphocyte population to increase, restoring immunocompetence. This can be very helpful in tackling both opportunistic infections and opportunistic malignancies, although specific antimicrobial and oncological therapy will still be required.

177

CASE 45 FEBRILE AND UNCONSCIOUS A 20-year-old woman is brought into accident and emergency having been found unconscious by a friend in her room at the local university halls of residence. The friend reports that the patient has no chronic medical problems or previous serious illnesses but had been suffering from a cold the previous day.The patient is not known to be taking any medication, has no history of recreational drug use or alcohol abuse. On examination, the patient is unconscious, with a Glasgow Coma Score of 3/15. Her temperature is 39.3°C. The pulse is 160/min and regular. The blood pressure is 75/45 mmHg. No focal neurological signs are present. There is a widespread purpuric, non-blanching rash.Vaginal examination is normal and in particular, no tampons are present. No needle track marks or signs of

Investigations Arterial blood gases (obtained on room air) pH 7.12 (7.35–7.45) pO2 6.9 kPa (11.2–12.6 kPa) pCO2 6.4 kPa (4.7–6.0 kPa) HCO3– 14 mmol/L (19–24 mmol/L) Full blood count Hb (female) 9.2 g/dL WCC 24 ⫻ 109/L Neutrophils 21 ⫻ 109/L Platelets 33 ⫻ 109/L MCV 81 fL MHC 31.3 pg MCHC 34.9 g/dL

trauma are found. The patient appears well nourished. There is no evidence of faecal or urinary incontinence. The oxygen saturations on room air are 85 per cent, improving to only 90 per cent with high flow oxygen by mask. Urinary catheterization reveals a residual volume of just 20 mL. Urine output over the next 30 minutes is 5 mL. It is noted that the patient oozes blood persistently from the venepuncture wounds and develops a bruise at the site of the arterial blood gas (ABG) sampling despite reasonable pressure. Assorted emergency investigations are obtained. Question 1 What process do the results of the full blood count and clotting tests indicate and what is its basic mechanism?

Answer 1 The clotting analysis exhibits prolongation of the international normalized ratio (INR), activated partial thromboplastin time ratio (APTTR) and thrombin time

(11.5–15.5 g/dL) (4–11 ⫻ 109/L) (150–450 ⫻ 109/L) (75–95 fL) (28–33 pg) (32–36 g/dL)

Clotting INR 4.2 (0.9–1.1) APTTR 3.4 (0.9–1.1) TT 19 s (14–16 s) Fibrin degradation 35 mg/mL (⬍10 mg/mL) products Biochemistry Na⫹ K⫹ Urea Creatinine Glucose

137 mmol/L 5.6 mmol/L 10.4 mmol/L 250 µmol/L 3.9 mmol/L

Chest X-ray

See Figure 45.1

(135–145 mmol/L) (3.5–5 mmol/L) (2.5–6.7 mmol/L) (70–170 µmol/L) (3.5–5.5 mmol/L)

Figure 45.1 Typical portable ICU chest X-ray in a young patient with acute respiratory distress syndrome (ARDS) showing an endotracheal tube in the distal trachea (arrow head), a nasogastric tube with the tip placed in the stomach (arrow) and widespread ‘airspace shadowing’ or ‘consolidation’.These ill-defined areas of opacity develop as fluid, pus (pneumonia), blood (Goodpasture’s syndrome) or tumour (bronchioloalveolar carcinoma) fill the alveoli.

Case 45: Febrile and unconscious

(TT), denoting a bleeding disorder that affects the whole of the coagulation cascade. The elevated fibrin degradation products indicate widespread activation of the coagulation system in that they demonstrate that there has been a significant increase in the quantity of fibrinogen that has been activated and converted to fibrin, then degraded by fibrinolytic processes. The full blood count reveals thrombocytopenia, implying that the platelet component of the clotting system is also deficient. Further evidence of deranged clotting is present in the form of the patient’s failure to clot at her venepuncture and ABG wounds.

The combination of widespread prolongation of clotting assays, thrombocytopenia and raised FDPs indicates disseminated intravascular coagulation (DIC). DIC is a condition in which there is an aberrant, generalized activation of the clotting and fibrinolytic systems on a systemic scale, usually at the level of small blood vessels. Small microthrombi are formed and are broken down by the fibrinolytic system throughout the circulation. The process occurs on a sufficient level to deplete the reserves of the coagulation and/or fibrinolytic systems. Typically, the coagulation system succumbs to failure first, resulting in an overall bleeding disorder.

Clinical setting Infection

Obstetric complications

Malignant tumours

Massive tissue damage

Gram-negative organisms

Pre-eclamptic toxaemia

e.g. pancreas, lung, prostate

e.g. trauma, burns

Meningococci

Placental abruption Amniotic fluid embolism

Mechanism Activation of clotting⫹⫹⫹⫹

Activation of fibrinolysis⫹⫹⫹⫹

Clinical effects COAGULATION TENDENCY Microinfarction throughout body

BLEEDING TENDENCY Haemorrhage into skin, intestines, brain and other organs

Sites commonly affected Kidney: Brain: Heart: Lungs: Adrenals: Liver: Spleen: Stomach and intestines:

Acute renal failure Coma, convulsions and death Petechial haemorrhages over endothelium and pericardial surface Intra-alveolar haemorrhage and ARDS Medullary haemorrhage Liver failure, exacerbates clotting factor deficiency Typically softened Widespread petechial haemorrhage into mucosal layer, oozing blood loss into lumen

Death from multi-organ failure due to a combination of haemorrhage and infarction

Figure 45.2 Disseminated intravascular coagulation (DIC): clinical settings and effects.

179

180

Part 5 Complex management

DIC has many causes which share the property of being forms of a severe, systemic insult. Question 2 What does the information indicate about the patient’s renal function and what is the likely cause?

Answer 2 The patient has acute renal failure. The low volume of urine in the bladder, given the lack of evidence to suggest that the patient had recently emptied her bladder, is the first indication of a low urine output.The formal measurement confirms this. As a general rule, the minimum acceptable urine output for an adult is 30–35 mL/hour. This may be refined to 0.5 mL/kg/hour, which is a useful guide for patients whose weight is significantly different from the 70 kg standard model. The hyperkalaemia, elevated urea and creatinine are biochemical features of acute renal failure and reflect the failure of the kidney to excrete these substances adequately from the blood. The cause of the renal failure is inadequate renal perfusion, secondary to marked hypotension. This is also known as prerenal acute renal failure. In the absence of an adequate blood flow and perfusion pressure, glomerular filtration fails. The low blood flow is also harmful to the tubular epithelium and thus a previously normal kidney is unable to function. Question 3

Answer 3 The blood gases show hypoxia, hypercapnia and an acidosis.This is type II respiratory failure. Question 4 What derangement has occurred in the acid–base balance?

Answer 4 The acidosis is complex as the bicarbonate level is also low. In a pure respiratory acidosis, bicarbonate is normal or elevated, the latter occurring due to compensatory retention of bicarbonate by the kidney. However, in either a pure metabolic acidosis or a mixed metabolic and respiratory acidosis, bicarbonate levels are reduced as they are consumed by the excess hydrogen ions of the acid. In this case, the patient has a mixed respiratory and metabolic acidosis. The respiratory component of the acidosis is a consequence of the impaired gas transfer leading to retention of carbon dioxide.This patient has two reasons to have a metabolic acidosis. First, acute renal failure is accompanied by a metabolic acidosis because the kidney is the normal route for excreting hydrogen ions. Second, the patient’s severe hypotension will lead to significant organ hypoperfusion and a switch to anaerobic metabolism, with an accumulation of lactic acid. Question 5

What process has occurred in the lungs and what basic mechanism underlies it?

What is the diagnosis and what process underlies the complications above, together with the hypotension?

(b)

(a)

(c)

Figure 45.3 (a) Purpuric skin rash in a baby with meningococcal meningitis. (b) Early and (c) late meningococcal rash.The spots fail to blanch when viewed through a glass tumbler pressed against the skin. Most inflammatory rashes are caused by local vasodilatation but in disseminated intravascular coagulation (DIC)-associated rashes, such as that seen in meningococcal septicaemia, there is damage to capillary walls due to microthrombus formation, followed by haemorrhage into the skin. (Photographs reproduced with the kind permission of the Meningitis Research Foundation; www.meningitis.org.)

Case 45: Febrile and unconscious

Answer 5 The clinical presentation is very characteristic of meningococcal septicaemia. The causative organism, Neisseria meningitidis (meningococcus), can also produce meningitis, but meningococcal septicaemia is often present in the absence of meningitis and vice versa. Meningococcus wreaks much of its havoc in meningococcal septicaemia by the production of an endotoxin.The endotoxin damages endothelium, leading to increased

LTB4

permeability and thrombosis.Thus, the significant quantities of endotoxin that are spread throughout the vasculature have a route for triggering widespread foci of coagulation and initiating DIC. Endotoxin also stimulates the production of large quantities of assorted cytokines by macrophages, including interkeukin 1 (IL-1), IL-6 and tumour necrosis factor (TNF). Considerable activation of complement also occurs and further drives an acute inflammatory response, thereby contributing to vasodilatation and

C5a

PAF

Neutrophil aggregation

O2 free radicals

Arachidonic acid metabolites e.g. thromboxane

Endothelial damage ↑Capillary permeability Pulmonary vasoconstriction

Oedema

Lysosomal enzymes e.g. proteases

Epithelial damage

Digestion of collagen framework

Surfactant loss Desquamation Atelectasis and hyaline membrane formation

POOR VENTILATION AND GAS TRANSFER

Vasoconstriction Neutrophils attracted by chemical mediators released secondary to shock secrete thromboxane and other vasoconstrictors

Atelectasis (collapse of airspaces) Caused by deficient surfactant secretion by damaged alveolar cells

Pulmonary oedema Outflow of fluid from vessels made leaky by inflammatory mediators and direct endothelial cell damage by lysosomal causes interstitial and alveolar oedema

Hyaline membranes Amorphous pink material lining alveolar spaces, composed of desquamated pneumocytes, degenerate macrophages, fibrin and plasma constituents PULMONARY VASCULATURE

AIR SPACES

Figure 45.4 Pathogenesis of adult respiratory distress syndrome (ARDS). An initiating event causes shock. Acute inflammatory mediators damage vascular endothelium, the alveolar walls and lining epithelium and cause pulmonary vasoconstriction, oedema, collapse and the formation of membranes over the alveolar surface.

181

182

Part 5 Complex management

hypotension. TNF stimulates endothelial cells to produce nitric oxide. Nitric oxide is a powerful physiological vasodilator that is important in normal vascular regulation. If nitric oxide levels are inappropriately elevated throughout the vasculature, as can occur if driven by meningococcal endotoxin, hypotension and shock result. Nitric oxide-induced shock and hypotension are especially difficult to treat because the nitric oxide acts directly at the final step of the vasodilator pathway. Inhibitors of vasodilatation are generally unsuccessful because they interrupt the vasodilator pathway before nitric oxide has become involved. Instead, pressor agents are needed to drive the vasoconstrictor mechanisms. Inhibitors of nitric oxide synthetase are in development. Caution will be necessary because nitric oxide is important in cerebral neurotransmission and cerebral blood flow autoregulation and disturbing these processes in a brain already suffering from the effects of septicaemic shock is not ideal. However, isoforms of the enzyme exist and may permit organ specificity. Question 6 Why was specific reference made to the absence of a tampon?

Answer 6 Retained tampons can act as a focus for Staphylococcus aureus which then secretes an endotoxin that can cause the serious toxic shock syndrome.

SUMMARY

Although this case may seem to have many complex, independent elements, they all originate from the basic process of infection and the production of a toxin by the organism.The toxin has systemic access and operates at a systemic level. By inducing an inappropriately targeted, excessive, acute inflammatory response, the endotoxin induces profound vasodilatation and shock. As is typical of these processes, acute renal failure and acute respiratory distress syndrome (ARDS) result. DIC is a well-known companion to shock and endotoxin-mediated diseases and also supervenes. Knowledge of the underlying mechanism shows that treatment should be aimed at removing the cause, which is the presence of the endotoxin that is itself secondary to the meningococcal septicaemia. Appropriate antibiotics are required to address this. Supportive care is also essential to counter the established effects of the endotoxin. This can be challenging but understanding how the organs are damaged and why they are failing permits optimal corrective measures to be taken.

CASE 46 MULTI-ORGAN DISEASE A 59-year-old man presents to his GP with a onemonth history of swelling of his ankles, urinary frequency and polydipsia. Further questioning also elicits a history of a reduced libido and shortness of breath on moderate exertion where this had not previously been a problem. On examination, the patient seems to have a greyish tinge to his skin, although does not appear anaemic or cyanosed.There is mild pitting oedema of the ankles.The pulse is 88 beats per minute and irregularly irregular.The blood pressure is 135/84 mmHg. The jugular venous pressure (JVP) is not raised. The apex beat shows mild lateral displacement, but is normal in quality. No heaves or thrills are present. The first and second heart sound are normal and there are no murmurs or added sounds. Examination of the lungs reveals bilateral basal crackles, but is otherwise normal.Abdominal examination is unremarkable. Examination of the external genitalia reveals testes that seem to show a mild degree of atrophy. No scrotal masses are present. Neurological examination is unremarkable. The GP orders a variety of blood tests.

Investigations Full blood count Urea and electrolytes Bilirubin GGT ALT Alk phos Albumin Glucose

Normal Normal 16 µg/L 134 IU/L 79 IU/L 90 IU/L 37 g/L 8 mmol/L (fasting venous)

A chest X-ray reveals a limited degree of upper lobe blood diversion and a mild increase in the width of the cardiac silhouette. Question 1 What rhythm would the ECG be expected to demonstrate?

Answer 1 Atrial fibrillation. The irregularly irregular pulse is typical. In the case of this patient, no additional abnormalities were present.

Question 2 What additional cardiac process is occurring?

Answer 2 Cardiac failure of a relatively mild degree.The patient has exertional dyspnoea, coupled with peripheral oedema and evidence of pulmonary oedema on clinical examination and chest X-ray. Given that the patient’s pulse rate is in the normal range, the atrial fibrillation alone may well not be sufficient to explain the impaired cardiac function. Question 3 What endocrine abnormality(ies) does this patient have?

Answer 3 The elevated fasting blood glucose indicates diabetes mellitus.This would also explain the polyuria and polydipsia due to an osmotic diuresis induced by glycosuria. There is also likely to be an element of hypogonadism. There has been a change in the patient’s libido and there seems to be some degree of testicular atrophy. Further investigation would reveal that the patient had a low blood follicle-stimulating hormone (FSH), luteinizing hormone (LH) and testosterone. Question 4 What is the diagnosis?

Answer 4 The patient has haemochromatosis. He has abnormal liver function tests, cardiac disease, diabetes mellitus, pituitary disease (hypogonadism secondary to decreased levels of the FSH and LH from the pituitary) and characteristic skin pigmentation changes. Question 5 What is haemochromatosis?

Answer 5 Haemochromatosis is an autosomal recessive disease in which there is an abnormal increase in iron absorption from the gut. Question 6 How could the diagnosis be confirmed?

184

Part 5 Complex management

Crypt enterocyte

The daughter cells of the crypt enterocyte move up through the crypt into the villus where they will be responsible for absorption of nutrients from the gut. They carry with them the modifications of function generated in the crypt enterocyte. Thus, if body iron stores are high, the expression of the iron absorbing protein gene will be reduced in order to decrease iron absorption. Conversely, if iron stores are low, as detected by the HFE protein, then expression of the iron absorbing protein will be increased

HFE protein allows the crypt enterocyte to determine body iron stores through transferrin levels and modulate its DNA accordingly HFE protein coupled to transferrin receptor

Fe Fe Fe Fe Fe

Transferrrin binds to a transferrin receptor on a crypt enterocyte Transferrrin carrying previously absorbed iron in the blood

Fe Fe Fe Fe Fe

Brush border

Villus enterocyte Iron absorbing protein

DNA carrying the modification of its expression determined in the crypt enterocyte

Transferrin receptor

Fe Fe Fe Fe Fe

Figure 46.1 Control of absorption of iron from the gut.

(a)

(b)

Figure 46.2 (a) Axial CT in a haemochromatosis patient with nodular cirrhosis (arrow head) with multiple small nodules scattered through the liver and ascites (thin arrow). (b) Axial T2 MRI of the liver showing two hepatocellular carcinomas (thin arrows) in a haemochromatosis liver – a complication of cirrhosis. Note the nodular margin of the liver (arrow head) a feature of cirrhosis.

Answer 6 Genetic studies can be performed and are the most useful, but measurement of iron transport and storage levels in the blood is also helpful and quicker. Liver biopsy can reveal the presence of excess iron, as well as assessing other histological parameters and permitting measurement of the total iron content. The gene for haemochromatosis is located on chromosome 6p21.3, although alternative loci have been described for variants of the disease.The protein is called HFE.There is a linkage with the HLA types A3 and B14.

excess iron is present in the free form. Free iron is actually toxic and causes damage because it induces the generation of free radicals. The tissue damages triggers a chronic inflammatory response that includes fibrosis. Different organs will show different degrees of susceptibility to haemochromatosis, due in part to their roles in normal iron metabolism. As the body’s main metabolic centre and iron reservoir, the liver is especially vulnerable. However, the heart, pancreas, skin and pituitary are also at particular risk. Iron deposition and the ensuing chronic inflammation and fibrosis damage these organs, leading to impaired organ function.

Question 7

Question 8

How does haemochromatosis cause disease?

What patterns of change can occur in the liver?

Answer 7 The elevated absorption of iron from the gut overwhelms the body’s storage capacity for iron.This means that the

Answer 8 The liver has surprisingly few basic patterns of response to injury, although these are often coloured by assorted

Case 46: Multi-organ disease

Answer 9 Even if the diagnosis has been confirmed through other means, liver biopsy remains important in haemochromatosis in order to evaluate the extent of the liver damage. The distinction between fibrosis without cirrhosis and cirrhosis is important. Question 10 Given that haemochromatosis is a disease of iron overload, what treatment would seem appropriate?

Figure 46.3 Cirrhotic liver showing bright blue positivity with Perl’s stain for haemosiderin.This is grade 4 (of 4) haemosiderosis. Most of the iron is within liver parenchymal cells, suggesting a genetic aetiology, rather than in Kupffer cells, where surplus dietary iron or transfusional iron overload is first apparent.

nuances for specific diseases. In haemochromatosis, the liver changes at presentation tend to fall in the spectrum of chronic hepatitis, active chronic hepatitis, cirrhosis and hepatocellular carcinoma. As a general rule, anything that causes active chronic hepatitis can cause cirrhosis and anything that can cause cirrhosis can cause hepatocellular carcinoma, but the tendency with which cirrhosis can lead to hepatocellular carcinoma does vary. Question 9 Is liver biopsy important if the diagnosis has already been made?

Answer 10 It is not currently possible to correct the abnormality in the bowel that causes the increased absorption of iron. However, it is possible to remove the excess iron. Regular venesection is a simple mechanism for extracting iron from the body and avoids the use of chelating agents and their attendant side-effects.Venesection can produce quite marked improvements in organ function, but cannot reverse established cirrhosis. Question 11 Why do women who have haemochromatosis tend to present later than men?

Answer 11 Women of reproductive age have natural blood loss in the form of menstruation.This can be sufficient to hold the disease at bay until after the menopause.

185

CASE 47 MIXED SOCIAL AND MEDICAL PROBLEMS Gavin Solaros is a 33-year-old futures trader in the City of London.The element of risk appeals to him and he is well paid for his nerve and success. His lifestyle is lavish. Gourmet feasts are accompanied by the finest wines and spirits. One evening, Gavin polishes off a bottle of claret with a delicious plate of roast guinea fowl. Suddenly he is seized by a searing pain in his epigastrium, which radiates through to his back. He is nauseated and vomits – the great wine is wasted! He doubles up but no change of position relieves the agony. Gavin summons his GP, who notes that the abdomen is extremely tender, with peritonism. Bowel sounds are absent. He has a pyrexia of 38.3°C, blood pressure of 100/60 mmHg and tachycardia of 118 beats per minute, sinus rhythm. Gavin is urgently admitted to hospital. Question 1 List and comment on your differential diagnosis.

Answer 1 Perforated duodenal or gastric peptic ulcer: abdominal X-ray would show gas under the diaphragm if the free wall is perforated; perforation into the pancreas would cause a moderate rise in pancreatic amylase and back pain. Acute pancreatitis: he is rather young. 80–95 per cent of cases have a history of gallstones or alcohol abuse. Look for pyrexia, tachycardia, mild jaundice, bleeding into flank or around umbilicus. Serum amylase >1000 IU/L is diagnostic. Acute appendicitis: unusual symptoms and signs unless appendix is long and retrocaecal. Biliary colic, acute cholecystitis: look for jaundice, dark urine. Rather young. Pain is usually colicky. May precede pancreatitis. Ureteric colic: usually a griping pain, possibly accompanied by loin pain on affected side if pyelonephritis is present, may have haematuria or dysuria. Meckel’s diverticulitis with perforation – rare! Meckel’s diverticulum occurs in 2 per cent of the population, but few contain ectopic pancreatic or gastric mucosa which increase the risk of perforation. Intestinal obstruction by torsion, intussusception, Crohn’s stricture, infarction: pain would not usually radiate to the back.

















At this age, diverticulitis, carcinomatous obstruction or perforation or abdominal aortic aneurysm are unlikely.

In hospital, Gavin’s blood and urine are tested. An abdominal ultrasound and CT scan show a few small gallstones in the gallbladder, but no stone obstructs the common bile duct (CBD).The pancreas is swollen. He settles on non-morphine-derived painkillers and intravenous fluids. The test results return.

Investigations Hb WCC Platelets Urea Creatinine Calcium Glucose Blood gases Amylase

11.3 g/dL 13.5 ⫻ 109/dL 200 ⫻ 1012/dL High normal High normal Low normal Slight increase Normal Slight acidosis 1340 IU/L

A diagnosis of acute pancreatitis is made. He is reassured that his attack could have been far worse. Question 2 What is the mechanism underlying acute pancreatitis? What are the long-term complications?

Answer 2 In 60 per cent of cases, it is due to gallstone obstruction of the CBD and pancreatic duct, with bile reflux into the pancreatic duct. Twenty per cent of cases are due to alcohol, possibly a directly toxic effect. Alcohol causes extremely viscid mucous secretions which can result in protein plugs that obstruct small and large pancreatic ducts. Obstruction causes inflammation and autodigestion of the pancreas by its wide range of enzymes. Lipases digest adjacent fat, proteases the pancreas itself and elastases demolish the walls of blood vessels. Severe bleeding into the pancreas and retroperitoneal tissues can result and this may track into the flanks and around

Case 47: Mixed social and medical problems

the umbilicus to cause bruising (Cullen’s and Grey–Turner’s signs, respectively). Acute pancreatitis is life-threatening. Some patients spend months in intensive care. Adverse features (Ranson’s criteria) are: ● ● ● ● ● ● ● ● ●

very high white cell count low haemoglobin raised urea raised liver enzymes inflammatory ascites raised glucose metabolic acidosis low blood pO2 levels age more than 55 years Long-term complications include:







pancreatic pseudocyst: fluid and blood accumulate in the destroyed pancreatic tissue. chronic pancreatitis: acute does not necessarily precede chronic pancreatitis. diabetes mellitus due to damage to the islets of Langerhans.

but resents the constant questioning about his alcohol intake. His colleagues comment that he has become withdrawn. Sometimes his breath smells of alcohol mid-morning. He is defensive and aggressive when questioned, and then suddenly dissolves into tears. Embarrassed, they shuffle him out of the building and home in a taxi before their employers see him. His GP tells him that he is depressed, which may be linked to his continued alcohol consumption, and adds, ‘Can I ask you some questions, called a CAGE test?’ ‘Just give me some pills and leave me alone,’ snarls Gavin rudely.The GP prescribes antidepressants. Once he has gone, Gavin turns once more to his cellar and comforts himself with a deliciously crisp bottle of Pouilly Fumé with … no, he doesn’t feel like eating. He rounds off his evening with a glass of vintage port. Question 3 What psychiatric manifestations of alcohol dependence are seen in Gavin? What is the CAGE test, and does he score positively?

Gavin returns to work, shaken, 12 weeks later. He has been advised to avoid alcohol. Unusually, he finds himself anxious. He finds that alcohol improves his performance. He starts taking a hip flask containing vodka to work. His trading decisions are erratic. He has regular check-ups with his GP,

(a)

(b) Figure 47.1 CT of acute pancreatitis with the tail of the pancreas replaced by fluid (arrow head) pancreatic necrosis and gas in a pancreatic abscess (thin arrow).

Figure 47.2 (a) Chronic alcohol-related pancreatitis with calcifications in the pancreatic duct (arrow). (b) The appearances are mirrored in this pathological specimen showing the pancreas and duodenum.

187

188

Part 5 Complex management

Answer 3 Inability to concentrate Depression Emotional lability.

● ● ●

The CAGE test is a four-question test developed by Dr John Ewing: ●



● ●

Have you ever felt you should cut down on your alcohol intake? Have you been annoyed by someone criticizing your alcohol intake? Have you ever felt guilty about your drinking? Have you ever had to take a drink first thing in the morning (‘eye-opener’) to recover from drinking the night before or to steady your nerves?

Gavin has another bout of severe abdominal pain with fever. Back in hospital, the CT scan shows calcified areas in his pancreas. He is diagnosed with chronic pancreatitis, treated with analgesics and has his fluid balance restored. After four weeks he is discharged and advised to avoid alcohol at all costs.

Answer 4 Yes, alcohol is the main association, and gallstones are also implicated in recurrent acute pancreatitis. When Gavin finally returns to work, he is summoned to the chief executive’s office. Serious trading misdemeanours have been identified. With a shock Gavin recalled that he had meant to cover up some major losses about three months ago. He is sacked. After a year Gavin can no longer meet his mortgage repayments and his penthouse suite is repossessed. He moves to a small flat in the suburbs. He cannot get another city job and has brief stints at jobs with easy access to alcohol, losing each within weeks for unreliability. Another bout of chronic pancreatitis occurs about three years after the first episode, this time taking many weeks to recover. Gavin is now approaching 40 and is greying and haggard, with early clawing of his hands (Dupuytren’s contractures). He is now homeless. He begs for money and buys beer or spirits. He suffers painful attacks of gout in his fingers and his right big toe. Question 5

Question 4

What is gout and why are alcohol drinkers at increased risk?

Is chronic pancreatitis related to alcohol?

General: • Social exclusion • Malnutrition • Accidental trauma to self and others • Fetal alcohol syndrome (maternal alcohol use) Oesophagus: squamous cell carcinoma Intestines: bleeding due to raised INR

2

Stomach: superficial gastritis Liver-related: • Steatohepatitis • Cirrhosis • Hepatocellular carcinoma • Acute or chronic liver failure • Spontaneous bacterial peritonitis, other infection • Portal hypertension-related problems, e.g. bleeding Varices

1

4

Brain: • Encephalopathy • Depression ⫾ suicide • Dementia • Psychosis Heart: dilated cardiomyopathy, atrial fibrillation

Peripheral nerves: distal neuropathy

Figure 47.3 The diagram reinforces the fact that alcohol does not only damage the liver. It is likely that alcohol may play a less direct role in other diseases.

Case 47: Mixed social and medical problems

Answer 5 Uric acid is a product of purine metabolism. High uric acid levels in the blood, from over-ingestion, over-production or reduced excretion of uric acid, cause deposition of monosodium urate crystals in joints, typically the big toe. Gouty joints are exquisitely painful and are hot, red and very swollen. Attacks last about 2–3 days. Beer and some spirits contain high levels of purines, as do bacon, beans, liver and shellfish. Alcohol stimulates diuresis, raising the blood uric acid concentration. Male sex is important and genetic factors may be contributory. Gavin throws himself under a bus, but only breaks his right humerus. He is admitted to hospital once more. The bones will not unite. ‘You are so malnourished,’ a kindly night nurse explains to him.‘You must try to eat properly.We are giving you extra vitamins.’ Question 6 What can cause malnutrition in chronic alcoholics?

Answer 6 Chronic pancreatitis: pancreatic acinar destruction and fibrous replacement can cause pancreatic enzyme deficiency and malabsorption. Alcoholics often neglect food because the alcohol can be metabolized to give energy. Vitamin deficiencies develop, particularly the water-soluble B vitamins, which are poorly stored in the body. Thiamine (vitamin B1) deficiency is most common. Money is spent on alcohol rather than food. Loss of employment exacerbates this element.







Question 7 Which nutrients are most important in bone healing?

Answer 7 Vitamin C Zinc Protein Oxygen Vitamin D and calcium.

● ● ● ● ●

Gavin becomes breathless during his stay in hospital. The junior doctor has been reading about alcohol-related conditions and considers a diagnosis of alcoholic cardiomyopathy.Then Gavin coughs up purulent sputum – he has developed a chest infection with Pseudomonas aeruginosa, resistant to all first-line antibiotics.

Question 8 What is the significance of this infection?

Answer 8 Pseudomonas aeruginosa is a typical example of a hospitalacquired infection, and is often resistant to the standard first-line antibiotics for chest infections. Alcoholics are particularly susceptible to infection by encapsulated bacteria. Question 9 Does alcohol cause a cardiomyopathy?

Answer 9 There are three main types of cardiomyopathy – hypertrophic, dilated and restrictive.Alcohol is one of several causes of dilated cardiomyopathy. At last Gavin is told he can go home. Nobody checks that he has a home to go to. He wanders the streets. A policeman takes him to a night shelter, where he is fed and housed. Gavin has difficulty with simple physical tasks like doing up buttons. After a few years he becomes very confused and forgetful and invents stories to explain away the gaps in his memory. One day, over the course of a few hours, he begins to stagger, with eyes rolling – he seems drunk.The experienced hostel manager recognizes Wernicke–Korsakoff’s type dementia. He takes Gavin to hospital. There is little that the psychiatric department can offer him, other than vitamin B injections. Question 10 Can alcohol cause the symptoms displayed by Gavin?

Answer 10 Gavin’s fumbling indicates a sensorimotor neuropathy, which often affects the feet and hands. Alcohol may be directly toxic or act via vitamin deficiencies (particularly B12, thiamine and vitamin E). Malnutrition of B vitamins (especially thiamine) causes cortical atrophy with damage to the hypothalamus and mamillary bodies, important for processing memories. Wernicke’s encephalopathy comprises paralysis of eye muscles, nystagmus, confusion and major problems with balance and co-ordination, with a strikingly wide-based,

189

190

Part 5 Complex management

irregular and staggering gait. Patients are often misdiagnosed as being drunk, so delaying treatment.Wernicke’s encephalopathy may rapidly progress to coma and death if vitamin B1 is not infused immediately – the brain damage already incurred cannot be repaired.Twenty per cent of patients with Wernicke’s encephalopathy die; 85 per cent of Wernicke’s encephalopathy survivors will

(a)

(c)

develop Korsakoff’s psychosis (amnesic-confabulatory syndrome), caused by thiamine deficiency. Patients fill the gaps in their memories by inventing stories. They often show personality alterations. One winter’s night, Gavin falls asleep on an icy bench outside the city firm where he once was a rising star. He never awakens.

(b)

(d)

Figure 47.4 (a) Coronal CT in a patient with a head injury after a drinking binge showing a large subdural haematoma (arrowhead) associated with mass effect, midline shift and subfalcine herniation (arrow). (b) Cerebral atrophy in a 35-year-old man with chronic alcohol abuse. Note the depth and prominence of the cerebral sulci (arrow). (c) Hand X-rays of a patient with gout with erosive changes in the joints (arrows), joint deformities and preservation of bone density. (d) Chest X-ray showing congestive cardiomyopathy in a 50-year-old male patient with chronic alcohol abuse . Note the massive heart and the plethoric lungs.

CASE 48 CERTIFYING THE CAUSE OF DEATH This book, Pathology in Clinical Practice, would not be complete without some practice in deciding how to certify the cause of death. It is the duty of doctors to provide the ‘Medical Certificate of Cause of Death’ to bereaved relatives and to know when to refer cases to the coroner, so let us use some of the scenarios in the book to illustrate the key points. Question 1 Look back at Case 1, Intermittent chest pain (p.2) and imagine that the man progresses to suffer from unstable angina, is seen regularly by his doctor and then suddenly collapses and dies while watching a crucial cup match at his local football club.There are no suspicious circumstances.What would you write as cause of death on the death certificate? A copy of the certificates used in the UK is shown as Figure 48.1.This is closely similar to those of other countries, but some allow for four lines of sequential conditions under I.

Answer 1 He has a clearly diagnosed condition, unstable angina, that can result in sudden death and no reasons to suspect any other cause so the death certificate would show: I (a) Acute myocardial ischaemia (b) Coronary artery atheroma Question 2 What are death certificates used for?

Answer 2 The death certificate is needed by the relatives so that they can register the death.What is written as the cause of death and how the death is explained to them by the healthcare professionals involved can help them come to terms with the loss, so it is important to take time to do this well. The data are recorded by the General

Figure 48.1 A specimen copy of the death certificate that is completed by doctors in England and Wales.

192

Part 5 Complex management

Register Office (in England and Wales), collated and analysed by the Office of National Statistics and used by researchers and health planners to assess health needs and the impact of interventions. Question 3 Let’s try another case. Look back at Case 12, Permanent neurological signs (p. 44) and imagine that this woman did not return home, because she was unable to care for herself, and was looked after in a nursing home. Here she became depressed, ate little and rarely moved out of bed. She developed a productive cough but had indicated in her Advance Decision to Refuse Treatment (ADRT) that she did not want to receive antibiotics. She died 48 hours later. Her notes indicate that she had a malignant breast lump removed three years ago and was not thought to have any recurrence.What would you write on her death certificate?

Answer 3 I (a) Bronchopneumonia (b) Left cerebral infarction (c) Cerebrovascular disease II Breast cancer The sequence of events here is that the final cause of death is her chest infection, which translates into medical terminology as bronchopneumonia.That is unlikely

to have killed her if she had not already had a ‘stroke’. Her ‘stroke’ was an ischaemic one (rather than a haemorrhagic one) and would have resulted in necrosis of cerebral tissue (cerebral infarction) with the underlying cause being narrowed and atheromatous arteries (cerebrovascular disease). Section II of the cause of death is for ‘Other significant conditions contributing to the death but not related to the disease or condition causing it’. This can be a very tricky decision. Has the breast cancer actually contributed to the death? Might she have been less depressed and more keen to accept antibiotics if she had not already had a malignant tumour and possibly feared that it might recur or spread? You have to use your judgement, but it is generally acceptable to indicate serious conditions under II so that the epidemiologists can use it if they wish. Question 4 How is the death certificate data collated and analysed?

Answer 4 The World Health Organization co-ordinates the production of coding systems suitable for use across the world that allow comparisons to be made over time and between different countries.The relevant one for death certificates is the International Statistical Classification of

I a: road traffic accident I b: multiple stab wounds II: War (Romans vs Gauls)

Figure 48.2 Be as precise as possible in selecting the actual cause of death for section I, and include contributing factors in section II.

Case 48: Certifying the casuse of death

Diseases and Related Health Problems, 10th revision known affectionately as ‘ICD-10’. The diagnoses reported on adult death certificates are coded automatically using software, a complex set of rules and the codes contained in the three volumes of ICD-10. It is the underlying cause of death that is coded and this should occupy the lowest completed line of part I. If you have difficulty organizing the sequence of events in part I, look at the figures you have put in the ‘approximate interval between onset and death’ box. The ‘underlying cause’ should have the longest duration.

urethral obstruction. That could have caused repeated infective episodes and/or raised back pressure affecting both kidneys. This would be supported by the finding of abnormal blood urea and electrolytes which occurs in chronic renal failure. His final cause of death will be acute cardiac failure, which can involve any combination of the cardiovascular consequences of acute or chronic renal failure (i.e. retention of fluid, hypertension, arrythmias due to raised potassium, pericarditis due to uraemia). So the death certificate would show:

Question 5

I (a) Acute cardiac failure (b) chronic renal failure (c) benign prostatic hyperplasia.

By now you will have appreciated that understanding the basic mechanisms of disease is the key to putting together the sequence of events that led to death, so let us just reinforce that with three more cases that you have already encountered. 1. Case 3, Difficulty in passing urine (p. 9). Imagine that this man has lived another decade but has become increasingly confused and is brought to accident and emergency with severe cardiac failure and is found to have pericarditis, and markedly raised blood urea and abnormal electrolytes. He dies the following morning. What do you put as the cause of death? 2. Case 31, Investigation after death (p. 118).This case has had an autopsy and you need to indicate in the upper part of the certificate whether you have taken the autopsy information into account. If not, you need to keep notes of the autopsy outcome so that you can provide further information when contacted. What are you going to give as the cause of death after attending the autopsy? 3. Case 23, Complex blood results (p. 90).This man with myeloma is treated for the next three years with intermittent chemotherapy to reduce the malignant plasma cell load.At the age of 72 years he has become a little breathless with features of mild heart failure and then drops dead half way round his favourite golf course. How do you complete the death certificate?

Answer 5a He is known to have had benign prostatic hyperplasia a decade ago and this is a condition which is slowly progressive so he is likely to have developed increasing

Answer 5b The autopsy has demonstrated metastases in most vital organs but no acute terminal event (such as massive haemorrhage from the ulcerated gut lesions).Thus, it is acceptable to put: I (a) Multi-organ failure (b) multiple metastases (c) malignant melanoma. Answer 5c If you are his GP treating his mild heart failure and expected to complete the death certificate it would be worth checking with the specialist looking after his myeloma whether there was any suggestion of cardiac amyloid and what his ECGs had looked like. This is a case where an autopsy might provide useful information about the effectiveness of his myeloma treatment and precise cause of his cardiac problems; but since this is natural death with a variety of possible causes, it does not require referral to the coroner. A possible cause of death would be: I (a) Acute cardiac failure (b) cardiac amyloid (c) myeloma. Without an autopsy, an acceptable alternative could be: I (a) Acute myocardial ischaemia (b) atherosclerosis II Myeloma.

193

CASE 49 REFERRING CASES TO THE CORONER Certifying the cause of death is closely similar in most countries around the world. The law will differ, however, on when you should not complete the death certificate but must refer the death for further investigation. This section explains the position for England and Wales at the time of publication. If you are practising in another country, make sure that you understand the local law. Question 1 The doctor who has been in attendance during the deceased’s last illness is required to provide a medical certificate of death if they are able to.What are the circumstances in which a doctor should not complete the death certificate?

Answer 1 You should not issue the certificate if you were not in attendance during the deceased’s last illness. Refer it to the doctor who was. You may complete the certificate if you are referring the case to the coroner, provided that you clearly indicate that you have referred it to the coroner for further action. Generally, though, it is better not to complete the certificate if you think it should be a coroner’s case. It can cause confusion and distress to the relatives if they try to register the death. Question 2 When should you refer cases to the coroner?

Answer 2 Currently there is a common law duty in England and Wales on all people to refer relevant cases to the coroner. The guidelines on what to refer are indicative rather than prescriptive and this is going to change with the introduction of legislation that will create a statutory duty on doctors and other public service personnel to report deaths to the coroner. The details are not finalized, but the list is likely to include: ● ●





Deaths resulting from self-harm and neglect. Deaths resulting from neglect or abuse when there is an established duty of care by a public authority, other organizations and individuals. Deaths occurring during or shortly after a period of detention (e.g. in police custody, by military authorities or under the Mental Health Act). Death caused or contributed to by the police’s conduct.

Deaths related to employment. Death resulting from lack of care or appropriate treatment, defective treatment and adverse reaction to prescribed medicine. Death of a child (‘where the death of that child was not anticipated as a significant possibility 24 hours before death’). Deaths where a violent crime is suspected. Sudden and/or accidental death. A death that is the subject of significant concern or suspicion. Where the death has not been certified (because no doctor has attended the patient recently). A death that may have been caused or contributed to by a specified disease or condition.This is an innovation which, under new legislation, would involve the chief coroner specifying certain conditions as reportable. Possible examples are severe acute respiratory syndrome (SARS), tuberculosis, deep vein thrombosis associated with air travel, avian flu. Deaths associated with childbirth or termination of pregnancy.

● ●



● ● ●







For most doctors, that can be summarized as: unexpected unknown unnatural suspicious.

● ● ● ●

Percentage of registered deaths 50 45 40 35 30 25 20 15 10 5 0 1920

1930

1938

1950

1960

1970

1980

1990

2000 2006

Figure 49.1 Deaths reported to coroners as a percentage of registered deaths, England and Wales. (Redrawn from data in Coroners’ Annual Statistics 2006 available at www.dca.gov.uk/ statistics/coroners.htm. Crown copyright material is reproduced with the permission of the Controller of HMSO and the Queen’s Printer for Scotland.)

Case 49: Referring cases to the coroner

Using that list as a guide, look back at Case 18, Tachycardia and tachypnoea (p.67) and decide whether you would refer to the coroner in each of the following scenarios:

asthma and is susceptible to chest infections and prescribes antibiotics and bed rest. She dies later that day. (c) Instead of calling the ambulance, she takes some painkillers and decides to try to sleep off her jet lag. She doesn’t wake up.

(a) She is admitted to hospital, commences appropriate treatment but suffers a further massive pulmonary embolism, which kills her. (b) Instead of calling the ambulance, she requests a GP home visit. Her GP knows that she suffers from

Answer 3a There is no need to refer to the coroner unless, in the future, the chief coroner stipulates that transatlantic flight-related deaths should be referred. Pulmonary

Question 3

Accident or misadventure 31%

Suicide 13% Industrial disease 12%

Industrial disease 2%

Suicide 9%

Accident or misadventure 40%

Natural causes 28%

Natural causes 23%

All other 9% Drug related 3%

Open verdicts 9%

Drug related 1% Open verdicts 8%

Males (n⫽18 855)

All other 12%

Females (n⫽8692)

Figure 49.2 Verdicts returned at inquest by sex, England and Wales, 2006. (Redrawn from data in Coroners’ Annual Statistics 2006 available at www.dca.gov.uk/statistics/coroners.htm. Crown copyright material is reproduced with the permission of the Controller of HMSO and the Queen’s Printer for Scotland.)

embolism is a natural death, the diagnosis has been established and it is not suspicious. She has been correctly treated, however, bereaved relatives may question the care when faced with a sudden unexpected death and they may suggest referral.Then it would be wise to agree. Answer 3b It is the GP who is the doctor in attendance and he or she must refer to the coroner because the death is unexpected. Even if the GP believes that the cause of death is a chest infection, a healthy 51-year-old would not be expected to die suddenly. Answer 3c This must be referred to the coroner and those involved need to investigate the full range of possible causes, including the unnatural and suspicious ones.

This requires the coroner’s officers to question the relatives and a pathologist to perform an autopsy. The information from these two sources is then discussed so that a certificate is issued or an inquest is held. In this case, the autopsy would show a natural cause of death (pulmonary embolism) and no inquest is required. Question 4 From the previous case (Certifying the cause of death, p. 191) look again at Question 5a and decide whether it should be referred to the coroner.

Answer 4 The confused 82-year-old does not sound to have received adequate care over the last few months or years but it is difficult to judge without knowing more

195

196

Part 5 Complex management

of his social history. If in doubt, phone the coroner’s officer for advice. Ultimately it is your responsibility to decide whether to refer, but taking advice is prudent. Question 5 You have signed the death certificate on one of your patients and the relatives are planning a cremation so you are asked to complete the cremation form.What is the crucial thing that you have to do before signing a cremation certificate that you do not have to do before signing a death certificate?

Answer 5 You must see the body.The death certificate requires you to indicate whether you have seen the deceased after death, whether another medical practitioner has or

whether no practitioner has, but does not require you to see it. For a cremation form, you must personally see and examine the body.The nature of the examination is not specified and simple inspection is sufficient if the person is long dead and has been dealt with by funeral directors or other health professionals. If you are the first to see the body, it is wise to turn it over to spot the knife in the back.

CASE 50 YOUR WORST NIGHTMARE!

A Sarah McKenzie Case

Sarah McKenzie had not spent five years at medical school without learning to ignore the swarm of low-flying rumours that buzzed around as the misleading harbingers of the next exam to lurk on the horizon, but neither had she made the mistake of underestimating the resilience of medical students in garnering intelligence ahead of the latest confrontation between student and faculty.With her finals now upon her, she had every reason to believe that the talk of the pathology department unleashing a new, secret weapon was true.There was to be an additional paper and in the absence of facts, the students had started to speculate. Rather than waste valuable revision time trying to ascertain the future, Sarah endeavoured to devote her energy to learning about those subjects she knew from her training were relevant. On the day of the first exam, one of Sarah’s friends, Kelly, had, according to Kelly’s own assessment, made an absolute mess of the pathology paper. Hours of attempts at reassurance had done little to ameliorate the situation and had predictably degenerated into a long and tearful discussion on the part of Kelly about what could come up in the infamous new extra paper, which would be upon them the following morning. By the time Kelly had been soothed by sympathetic listening and a revision session in which Sarah had tested her, from textbooks and lecture notes, on all manner of subjects, it was just after one in the morning before Sarah returned to her bed. There had been occasions before when Sarah had turned over an exam paper and found that the first question centred on a topic with which she was not particularly comfortable, or was frankly unfamiliar to her. A couple of times she had experienced the first cold pangs of fear when the second question had also fallen outside the boundaries of her expectation, but as she made her way through the pages of the dreaded pathology paper, there was no familiar point of anchorage for her, no place at which she could set down her pen and begin the challenge from a secure base. The icy sensation that gripped her was much worse than a mere chill of alarm and she could see her aspirations for the future, for her career, crumbling into fine dust that would be blown away with her next breath. Not one single disease did she recognize in the paper. Sarah looked around the examination hall, as candidates do when, in desperation, they try to seek some

comfort in the fact that most of their colleagues are suffering similar difficulties.The confusion and distress on the faces of the rest of the candidates mirrored her own. As the final test, Sarah, moved her gaze to her friend Thomas, the person that the rest of the year would look to as the top of the class. Thomas’s face bore what seemed to Sarah to be an incongruous smile, an expression that a chess player might adopt when he appreciates the ingenuity of an opponent’s move, yet perceives in the same instance of this admiration that he can reply to it, defeat its cleverness and ultimately smash it from the board. Sarah returned her attention to the paper. Other than Thomas, everybody had the same problem. She did not allow herself to be distracted by speculation as to whether or not the medical school would really be prepared to fail almost the entire year. Keeping calm and focusing on technique could yet go a long way. Deciding to adopt a methodical approach, Sarah began with the first part of question 1 as it was at least something she could do, even if the reference to the condition of Jacobs–Tate syndrome in the preceding preamble suggested that much of the question would be beyond her, given that she had never heard of Jacobs–Tate syndrome. ‘What is the GABA-A receptor?’ ‘The GABA-A receptor is a CNS neurotransmitter receptor for gamma-aminobutyric acid,’ wrote Sarah. ‘The binding of GABA opens a chloride channel, which has an inhibitory effect on the neurone.’ ‘What is the mechanism of action of benzodiazepines?’ Again, this was safe territory. Sarah liked pharmacology because it allowed her to correlate normal physiology and biochemistry with functional effects and rewarded the effort put in to learning those normal processes. ‘Benzodiazepines bind to the GABA-A receptor and allosterically modify it so that its frequency of opening is increased when GABA binds.This increases the effect of GABA on the receptor.’ At least Sarah had a couple of marks to her name now, although she had no idea how talk of the GABA-A receptor related to the main subject of the question, Jacobs–Tate syndrome. ‘What legal recreational drug exerts some of its effects via an action on the GABA-A receptor?’

198

Part 5 Complex management

Sarah was moving outside her central base of knowledge, but she remembered Thomas once saying something about alcohol affecting the GABA-A receptor and gave this as her answer. ‘What are the typical effects of this drug on CNS function?’ ‘Alcohol is a CNS depressant,’ answered Sarah, employing the tactic of beginning with a broad answer. ‘It impairs co-ordination, reaction times, judgement and reduces inhibitions.’ This aspect of her answer did not require any specialist medical knowledge. ‘Through actions on which part of the brain in particular are the effects of this drug on higher function mediated?’ Sarah nearly wrote cerebellum, but then read the question properly and noted the word ‘higher’. She recalled some functional neuroanatomy and opted for ‘frontal lobes’. Jacobs–Tate syndrome was no nearer to making any sense to her, but now that Sarah had recovered from her initial dismay on skimming the paper, she could see that she had a fighting chance of squeezing marks out of it. ‘How does Cyanococcobacillus jacobstateii survive phagocytosis?’ Sarah’s immediate thoughts on reading this would not have borne repetition in polite company. Her next thoughts were that with this question her source of marks from the Jacobs–Tate syndrome SAQ had dried up. However, the mention of surviving phagocytosis triggered a memory from the time she had been rescued from one of Professor Roland Pauls’ interminable rambles by one of the Professor’s registrars. Working on the basis that writing nothing would score nothing, Sarah chose to use her knowledge of TB. ‘The organism is able to prevent fusion of the lysosome with the phagosome,’ she offered. ‘What form of inflammatory response might result as a consequence?’ Continuing with her TB analogy, Sarah wrote ‘Granulomatous’. ‘Name one other sexually transmitted disease that can generate this type of inflammatory response.’ The more important thing for Sarah here was not that she scored another mark by answering syphilis, but that she inferred from the question that Cyanococcobacillus jacobstateii was transmitted sexually. ‘What characteristic, but non-specific clinical feature does this produce in the vaginal wall or testes of individuals infected with Cyanococcobacillus jacobstateii?’

Thinking again of TB, Sarah associated granulomas with fibrosis and the ability to produce a mass effect, so she answered ‘mass or nodule’. ‘With what other diagnosis may this cause confusion, especially in the male?’ Sarah reasoned that the differential diagnosis of a mass, especially in the testis, had to include a malignant tumour. ‘How do colonies of Cyanococcobacillus jacobstateii migrate from the genital tract to home to the olfactory bulb?’ Sarah did not have a clue as to the specifics, but her understanding of parallel concepts in embryology and the function of inflammatory cells suggested to her that Cyanococcobacillus jacobstateii possessed cell adhesion molecules which recognized targets that were specific to the olfactory bulb. ‘What clinical features can occur in the non-secretory CNS phase of Jacobs–Tate syndrome?’ Until a few minutes ago, Sarah had never heard of Jacobs–Tate syndrome and it was a further revelation to her that the disease had phases. Her first reaction was to give up on the question, but she glanced at her watch and saw that she was on schedule for the paper and so had time to think. ‘It’s an infection, causes granulomas, presumably fibrosis and is inside somebody’s head,’ mused Sarah. ‘And it likes the olfactory bulb. So these must be the reasonable features in that case.’ Giving the general clinical features of a chronic intracranial space-occupying lesion, Sarah wrote ‘headache and seizures’. Focusing on the specific site, she added ‘impaired sense of smell’. ‘What clinical features are attributable to the two neurotoxins ethano-benzodiazapoid A and ethano-benzodiazapoid B that are produced by Cyanococcobacillus jacobstateii in the secretory CNS phase of Jacobs–Tate syndrome?’ Now Sarah realized the link between the opening parts of the question and Jacobs–Tate syndrome. She recapitulated her earlier answer regarding the effects of alcohol and added in ‘drowsiness, increased sleep and anxiolytic effect’, to reflect the actions of benzodiazepines. ‘Variant strains of Cyanococcobacillus jacobstateii have reduced or absent synthesis of the two benzodiazapoid neurotoxins.Why could this be a disadvantage in the spread of these variant organisms between individuals?’ Sarah read over the answers she had already given, on the basis that everything she knew about Jacobs–Tate

Case 50: Your worst nightmare!

syndrome was in them. After a minute, she worked out that as the disease was sexually transmitted, anything that increased sexual activity, or higher risk sexual activity without barrier contraception, would facilitate spread of the organism. As Cyanococcobacillus jacobstateii preferentially targeted the frontal lobes with neurotoxins that had actions which looked like they induced a state resembling a degree of drunkenness, variants of the organism which were defective in the synthesis of these neurotoxins would lose the advantages of the effects of the toxins on promoting sexual behaviour. Realizing that the effects of the neurotoxins were concentrated on the frontal lobes by simple anatomical proximity of the focus of bacterial infection, she decided to delete the effects on co-ordination from her answer to the previous question. Feeling rather pleased with herself for having kept her head and worked through the question, Sarah’s concentration was disturbed by the sound of Kelly shouting abuse at one of the invigilators, a mild-mannered gentleman who worked as an office assistant in the medical school registry. Kelly appeared to be under the misapprehension that the poor man was personally responsible for setting the paper. His attempts to calm Kelly down were ineffective and the student became more and more agitated and aggressive.The reaching out of a hand by the man to soothe her was met by Kelly ramming her fountain pen into his neck with a degree of anatomical precision for the carotid artery that might have gathered marks if the subject of the exam had been surface anatomy.

Before Sarah knew what had happened, she and Kelly were sprinting headlong from the examination hall and through the nearby park. Exactly how or why she had come to accompany Kelly on her flight was unknown to Sarah, she was simply aware that one moment she had been sitting in her chair and the next she was several dozen metres away and that it made perfect sense, even though it simultaneously should not. For a few seconds, the high-pitched, intrusive and irritating noise sounded like an alert of some sort to Sarah, not really like a police siren, but something similar, perhaps a call to arms or battle stations. After a few more seconds, she had woken up sufficiently to realize it was her alarm clock. Sarah tried to make sense of her bizarre dream while she brushed her teeth. She remained convinced that she had never heard of Jacobs–Tate syndrome by the time she had made breakfast, but was equally persuaded that all the answers she had given in the paper she had imagined were in themselves correct. An internet search failed to reveal any references to Jacobs–Tate syndrome, Cyanococcobacillus jacobstateii or ethano-benzodiazapoids, leaving Sarah to conclude that while it was all the product of her nightmarish fantasy, she had been able to invent a disease merely through a knowledge of some principles of pathology and perhaps more importantly for the rest of her finals, that a knowledge of its pathology had permitted her to make sense of a disease of which she had otherwise never heard.

199